[河北] 切换
河北省学业规划中心 系统讲解

logo

专属河北报考与数据查询一站式平台

  • 1.普通高校招生有哪些形式?
  • 2.普通高校招生的主要环节有哪些?
  • 3.普通高考文化总成绩是如何组成的?
  • 4.我省高考实施的"3 + 1 + 2”模式和传统的文理分科有何区别?
  • 5.招生计划是如何编制的?什么是历史科目组合、物理科目组合?
  • 6.什么是强基计划?
  • 7.强基计划试点高校有哪些?
  • 8.强基计划如何招生?
  • 9.什么是强基计划招生“十严禁”?
  • 10.普通高校是怎样分类的?
  • 11.公办院校、民办院校与独立学院有哪些区别?
  • 12.什么是本硕、本硕博和本博连读?
  • 13.什么是本科学术互认课程(ISEC )?
  • 14.哪些高校(专业)对考生有特殊要求?
  • 15.普通体育招生、高水平运动队招生、体育单招有什么区别?
  • 16.体育单招如何招生?
  • 17.高水平运动队如何招生?
  • 18.2024年起高水平运动队招生将有什么变化?
  • 19.招收高水平运动队的试点院校和项目有哪些?
  • 20.艺术类专业考试形式有哪些?
  • 21.独立设置艺术院校(含参照执行院校)及相关专业有哪些?
  • 22.艺术类专业招生与高水平艺术团招生有什么区别?
  • 23.高水平艺术团招生的院校有哪些?
  • 24.什么是“定向就业招生”?
  • 25.什么是国家专项计划?
  • 26.什么是高校专项计划?
  • 27.什么是地方专项计划?
  • 28.什么是公费师范生?
  • 29.什么是优师专项计划?
  • 30.什么是免费医学定向计划?
  • 31.什么是对口升学考试?
  • 32.对口升学考试与普通高校招生考试能否兼报?
  • 33.什么是高职单招?怎样报考?
  • 34.保送生有哪些条件?
  • 35.什么是高校招生“阳光工程”?
  • 36.考生档案有哪些内容和作用?
  • 37.香港、澳门的高校如何招生?
  • 38.高考各阶段考生密码有哪些重置办法?
  • 39.考生如何查看自己的高考报名及体检信息?
  • 1.哪些人员需要参加高考报名?
  • 2.哪些人员不能参加高考报名?
  • 3.考生在我省参加高考报名须具备什么条件?
  • 4.外省进城务工人员随迁子女在我省参加高考报名应具备哪些条件?
  • 5.考生报考“三个专项”计划,应具备哪些条件?
  • 6.考生报考免费医学定向计划应具备哪些条件?
  • 7.考生报名时需要提交哪些材料?
  • 8.今年高考报名时间和地点是怎样规定的?
  • 9.报名工作流程是怎样的?
  • 10.考生报名时如何选择考试类型和报考类别?
  • 11.报考艺术类考生如何选择艺术类别?
  • 12.考生可以选择哪些外语语种?
  • 13.享受优惠加分政策的考生如何申请加分?
  • 14.符合优先录取条件的考生如何申请?
  • 15.报考普通体育类专业有哪些具体要求?
  • 16.我省对于考生兼报是如何规定的?
  • 17.报名和参加各种专业(专门)考试及文化统一考试时是否需要携带本人居民身份证?
  • 18.身份证、户口簿、学考及学籍的姓名、身份证号、民族、出生年月等内容不一致的如何办理报名手续?
  • 19.在外省就读回我省参加高考的考生,普通高中学生学业水平考试和综合素质评价信息如何办理?
  • 20.考生号的作用是什么?
  • 21.准考证的作用是什么?
  • 22.考生个人信息表的使用范围有哪些?
  • 23.考生为什么要签订《河北省普通高校招生考试考生诚信承诺书》?
  • 24.如何确定考试科目和交纳考试费?
  • 25.报考选择性考试科目有什么要求?
  • 1.怎样进行思想政治品德考核?
  • 2.哪些情况属于思想政治品德考核不合格?
  • 3.报考军队院校的考生怎样进行政治考核?
  • 4.报考公安院校的考生怎样进行政治考察?
  • 5.报考司法类高校提前录取专业的考生怎样进行政治考察?
  • 6.报考军队飞行学员的考生怎样进行政治考核?
  • 7.报考北京电子科技学院的考生怎样进行政审?
  • 8.综合素质评价如何使用?
  • 1.考生为什么要体检?
  • 2.考生体检应注意什么?
  • 3.高考体检有哪些项目?
  • 4.高考体检是怎样进行的?
  • 5.对体检结果有异议的考生怎样申请复查?
  • 6.《普通高等学校招生体检工作指导意见》的具体内容是什么?
  • 7.报考军队院校军检是如何进行的?
  • 8.军队院校招收学员体格检查标准具体内容是什么?
  • 9.报考公安院校面试体检和体能测评是如何安排的?
  • 10.公安院校招生对面试体检和体能测评有哪些要求?
  • 11.司法类高校提前录取专业的面试、体能测试是如何安排的?
  • 12.司法类高校提前录取专业对身体条件有哪些要求?
  • 13.报考军队飞行学员体检是如何安排的?
  • 14.报考民航飞行学员面试体检是如何安排的?
  • 15.普通高校为什么对新生进行身体复查?
  • 16.新旧视力对照表
  • 17.血压原用计量单位与法定计量单位换算表
  • 1.普通高校招生文化考试时间如何安排?
  • 2.普通高校招生文化课考试科目是如何设置的?
  • 3.思想政治、地理、化学、生物4门再选科目考试成绩 是如何进行等级赋分的?
  • 4.中等职业学校对口升学考试科目及考试时间如何安排?
  • 5.考点、考场是如何安排的?
  • 6.普通高考外语听力考试如何进行?
  • 7.残疾考生如何申请高考合理便利?
  • 8.艺术类专业统考是如何安排的?
  • 9.考生如何查询艺术类专业统考成绩?
  • 10.艺术校际联考是如何安排的?
  • 11.院校艺术类专业校考是如何安排的?
  • 12.我省对艺术类专业校考的相关规定有哪些?
  • 13.普通体育类专业考试有哪些要求?
  • 14.普通体育专业考试是怎样安排测试顺序的?
  • 15.河北体育学院少数民族传统体育项目测试是怎样安排的?
  • 16.对口专业考试是如何安排的?
  • 17.考生如何参加外语口试?
  • ​ 18.《考场规则》的主要内容有哪些?考生应遵守如下考场规则
  • 19.考生如何配合考试工作人员进行安全检查?
  • 20.考生考前需做好哪些心理准备?
  • 21.考生考前应注意哪些主要事项?
  • 22.考生遇有特殊情况应如何处理?
  • 23.答题过程中考生需要注意哪些事项?
  • 24.针对网上评卷,考生答题时要注意哪些问题?
  • 25.网上评卷有哪些流程?
  • 26.网上评卷有何优点?
  • 27.考生如何查询高考成绩?
  • 1.考生填报志愿的作用是什么?
  • 2.填报志愿的时间是如何安排的?
  • 3.考生填报志愿方式和流程是怎样的?
  • 4.考生在网上填报志愿时应注意哪些事项?
  • 5.高校专业(类)对考生选考科目有哪些要求?
  • 6.如何查询招生院校对选考科目的要求?
  • 7.2023年各录取批次及志愿是如何设置的?
  • 8.什么是征集计划?
  • 9.填报平行志愿有哪些建议?
  • 10.对平行志愿的认识误区有哪些?
  • 11.考生填报志愿的主要依据和参考资料有哪些?
  • 12.什么是志愿填报辅助系统?
  • 13.志愿填报辅助系统操作流程及注意事项有哪些?
  • 14.什么是河北省高考志愿填报智能参考系统?
  • 15.志愿填报系统、志愿填报辅助系统和志愿填报智能参考系统有什么区别?
  • 16.填报志愿时考生如何对待优惠加分政策?
  • 17.艺术类专业考生填报志愿应注意哪些问题?
  • 18.怎样填报艺术类平行志愿?
  • 19.体育类考生填报志愿有哪些注意事项?
  • 20.艺术类、体育类考生是否可以兼报普通类专业?
  • 21.填报军队院校志愿应注意哪些事项?
  • 22.填报招飞计划应注意哪些事项?
  • 23.填报公安院校志愿应注意哪些事项?
  • 24.如何填报定向就业招生志愿?
  • 25.高水平运动队招生如何填报志愿?
  • 26.高水平艺术团招生如何填报志愿?
  • 27.国家专项计划招生如何填报志愿?
  • 28.高校专项计划招生如何填报志愿?
  • 29.地方专项计划招生如何填报志愿?
  • 30.如何填报公费师范生(含优师专项)志愿?
  • 31.如何填报预科班志愿?
  • ​32.志愿填报截止后还可以补报志愿吗?
  • 33.未填报集中志愿的考生,是否可以填报该批(段)的征集志愿?
  • 34.如何对待“冷门”和“热门”专业?
  • 35.填报志愿时如何兼顾个性因素?
  • 36.河北招生考试信息服务网网上咨询活动何时开始?
  • 1.什么是“学校负责、招办监督”的录取原则?
  • 2.各批各类录取控制分数线是如何划定的?
  • 3.投档原则是什么?
  • 4.高校招生录取规则是什么?
  • 5.各批次(段)在录取时是否相互影响?
  • 6.网上录取基本流程是什么?
  • 7.平行志愿投档是如何进行的?
  • 8.征集志愿是如何投档的?
  • 9.什么是投档线?
  • 10.国家确定的优惠加分项目有哪些?分值是多少?
  • 11.河北省确定的优惠加分项目有哪些?分值是多少?
  • 12.投档录取时如何使用优惠加分?
  • 13.优先录取的规定有哪些?
  • 14.军队院校如何录取?
  • 15.公安院校如何录取?
  • 16.司法类院校提前录取专业如何录取?
  • 17.艺术类专业如何录取?
  • 18.普通体育类专业如何录取?
  • 19.高水平艺术团招生如何录取?
  • 20.高水平运动队招生如何录取?
  • 21.定向就业招生如何录取?
  • 22.国家专项计划招生如何录取?
  • 23.高校专项计划招生如何录取?
  • 24.地方专项计划招生如何录取?
  • 25.国家专项计划、高校专项计划、地方专项计划录取考生放弃入学资格或退学有什么后果?
  • 26.少数民族本科预科班如何录取?
  • 27.边防军人子女预科班如何录取?
  • 28.残疾考生报考普通高校录取有什么规定?
  • 29.单独招收残障考生的院校有哪些?
  • 30.控制分数线上考生未被录取的主要原因有哪些?
  • 31.如何准确查询录取结果?
  • 32.如何识别和防范招生诈骗行为?
  • 33.新生入学报到前应做哪些准备?
  • 34.考生如何申请国家助学贷款?
  • 35.高校如何进行新生入学复查?
  • 36.新生入学时如何购买合格的学生公寓床上用品?
  • 1.如何认定与处理考生的违规行为?
  • 2.考试工作人员违纪舞弊应受何种处理?
  • 3.对教育考试机构作出的违规处理决定不服的如何申诉?
  • 4.发现考生违规行为如何举报?
  • 5.违规招生的学校应受何种处理?
  • 6.在艺术、体育、对口升学和高职单招等各类单招中违规的考生如何处理?
  • 7.《中华人民共和国刑法》涉及考试作弊入刑的主要内容是什么?
  • 8.《最高人民法院最高人民检察院关于办理组织考试作弊等刑事案件适用法律若干问题的解释》涉及高考作弊入刑的主要内容是什么?
  • 9.《中国共产党纪律处分条例》涉及考试招生违规处理的主要内容是什么?
  • 10.《中华人民共和国教育法》涉及考试招生违规处理的主要内容是什么?

            河北省 2023年普通高校

        招生工作新视点

1.强化报名资格审查。对接教育部高校学信息和全省高学籍信息 实时比对考生学籍信息;联合公安部门对考生户籍和居住证信息逐一核查,严防高考移民”;与卫健、人社、退役军人等部门建立联审机制,强化优惠加分、优先录取等优惠资格审核,严防弄虚作假。

2.严厉打击违规行为。将防范手机等高科技作弊及群体性、有组织舞弊作为重点。考前联合公安、保密、无线电管理、通信管理等部门持续开展考试环境综合治理专项活动。考生入场全部采用人脸识别技术, 严防替考。强化安检措施, 严格执行国家教育考试入场安检规范, 严防手机等作弊器材入场。所有考场、 通道、卫生间等区域实现5G信号屏蔽全覆盖。考试期间强化监考巡考,实现全程监控、全程录像。考后严格执行考场视频回放制度,严查违规违纪行为。

3.严格三个专项计划报考资格。《教育部关于做好2023年普通高校招生工作的通知》(教学[2023]J1号)规定,从2023年起,往年被国家专项计划、高校专项计划、地方专项计划录取后放弃入学资格或退学的考生,不再具有专项计划报考资格。

4.加强涉考培训咨询机构治理。根据教育部部署,各地教育行政部门会同网信、科技、公安、文化和旅游、市场监管、体育等部门,加强对社会培训机构或个人开展涉考培训咨询的规范治理,严厉打击涉及虚假宣传、价格欺诈、组织或参与考试作弊、干扰破坏考试招生秩序等违规违法行为。

5.深化艺术类专业考试招生改革。按照教育部《关于进一步加强和改进普通高等学校艺术类专业考试招生工作的指导意见》精神,我省到2024 年,基本建立以统一高考为基础 、省级专业考试为主体,依据高考文化成绩、专业考试成绩,参考学生综合素质评价,分类考试、综合评价、多元录取的高校艺术类专业考试招生制度,形成促进公平、科学选才、监督有力的艺术人才选拔评价体系。

6.改革高校高水平艺术团招生办法。自2024 年起,高校高水平艺术团不再从高校招生环节选拔,由相关高校从在校生中遴选培养。

7.深化高校高水平运动队考试招生改革。自2024年起,进一步完善和规范高校高水平运动队考试招生工作,通过优化招生项目范围, 严格报考条件和资格审核,改进考试评价方式,提高文化成绩要求,完善招生录取机制,选拔培养德智体美劳全面发展且具有较高体育竞技水平的学生,为奥运会、世界大学生运动会等重大体育比赛和国家竞技体育后备人才培养体系提供人才支撑。


重要日程备忘

11.png22.png







wx.png

1.普通高校招生有哪些形式?

普通高校招生主要有统一考试招生和单独考试招生两种形式。

统一考试招生是指普通高校招生统一文化考试(含普通高中学业水平选择性考试)招生。

单独考试招生是指经教育部或省级教育行政部门批准的普通高校单独组织的考试招生。目前,主要有运动训练、武术与民族传统体育专业考试招生(简称体育单招),少年班、职教师资班、残障、消防等单独考试招生和高职院校单独考试招生(简称高职单招)。

f43213efbba25e9e7f3e98deb60214fd.png

2.普通高校招生的主要环节有哪些?

普通高校招生包括报名、思想政治品德考核、体检、确定选考科目、考试、填报志愿、录取、入学复查等环节。

报名。报名是基础工作,考生完成报名才能取得考试资格或录取备案资格。考生报名时填写的信息,用于建立考生电子档案、编排考场和录取备案等。

思想政治品德考核。主要是考核考生本人的现实表现。由考生所在学校或单位对考生的政治态度、思想品德作出全面鉴定,是高校录取的重要依据之一。

体检。即身体健康状况检查,由县级以上招生考试机构和卫生健康行政部门组织实施,在指定的二级甲等以上医院或相应的医疗单位进行。根据身体检查结果,考生可以选择适合自己身体状况的专业(类)。招生高校录取时,根据本校的专业要求,选择符合条件的考生。

确定选考科目。选考科目为思想政治、历史、地理、物理、化学、生物6 门, 考生须从历史、物理 2 门首选科目中选择 1 门,再从思想 政治、地理、化学、生物 4 门再选科目中选择 2 门参加考试。普通高中在校学生应在相应科目学业水平合格性考试合格的基础上报考选择性考试科目。

考试。检验考生对知识掌握和运用能力的评价方式,考试成绩将作为高校招生录取的重要依据。

填报志愿。考生通过填报志愿表达自己的升学愿望,志愿信息是高校录取新生的重要依据。

录取。高校依据本校录取规则和考生志愿,择优选拔德智体美劳全面发展的新生。

入学复查。新生入学报到时,各高校将依照《普通高等学校学生管理规定》对新生进行身份复核、体检复查。同时,对特殊类型招生录取的新生,高校将开展入学专业复测和复核。具体要求请向录取高校咨询。

f43213efbba25e9e7f3e98deb60214fd.png

3.普通高考文化总成绩是如何组成的?

考生普通高考文化总成绩由统一高考的语文、数学、外语3 门成绩和考生选择的 3 门选考科目成绩组成,满分为750分。统一高考科目的语文、数学、外语每门满分为150分, 均按原始成绩计入考生高考文化总成绩。选考科目每门满分为100分,其中首选科目历史、物理使用原始成绩计入考生高考文化总成绩,再选科目思想政治、地理、化学、生物按等级赋分后计入考生高考文化总成绩。

f43213efbba25e9e7f3e98deb60214fd.png

4.我省高考实施的“3 + 1 + 2”模式和传统的文理分科有何区别?  

"3 + 1 + 2" 模式和传统的文理分科有着本质上的区别。一是目标导向不同。 " 3 + 1 + 2" 模式既体现了物理、历史学科的基础性作用,突出了高校不同学科专业选才的要求,也更加注重学生的全面发展,提高学生的综合素质。二是选择科目组合不同。

"3 + 1 + 2" 模式,学生可根据个人爱好、兴趣、特长和拟报考学校与专业的招生要求以及高中学校的办学条件, 在 12 种组合中自主选择,增大了考生的选择面。而传统文理分科仅有 2 种固定的组合供考生选择。三是考试内容不同。" 3 + 1 + 2" 模式中,学生参加全国统一高考的语文、数学、外语3个科目考试时不分文理, 所有考生的考试试卷完全一致, 而传统文理分科的数学考试科目,试卷的内容和难度是有区分的。

f43213efbba25e9e7f3e98deb60214fd.png

5.招生计划是如何编制的?什么是历史科目组合、物理科目组合?

2023年我省普通高校招生普通类、体育类按历史科目组合、物理科目组合分别编制招生计划;艺术类不区分历史科目组合、物理科目组合,统一编制招生计划。

历史科目组合和物理科目组合以首选科目为界定标准,首选科目选择历史的为历史科目组合,首选科目选择物理的为物理科目组合。

f43213efbba25e9e7f3e98deb60214fd.png

6.什么是强基计划?

为深入贯彻习近平新时代中国特色社会主义思想,落实全国教育大会精神,服务国家重大战略需求,加强基础学科拔尖创新人才选拔培养,教育部在深入调研、总结高校自主招生和上海等地高考综合改革试点经验的基础上,制定出台了《关在部分高校开展基础学科招生改革试点工作的意见》(也称强基计划),在确保公平公正的前提下,积极探索多维度考核评价模式,逐步建立基础学科拔尖创新人才选拔培养的有效机制。

立足服务国家战略需求。强基计划主要选拔有志服务国家重大战略需求且综合素质优秀或基础学科拔尖的学生,突出基础学科的支撑引领作用,由有关高校结合自身办学特色,重点在数学、物理、化学、生物及历史、哲学、古文字学等相关专业招生。

遴选部分高校开展试点。起步阶段,遴选部分一流大学建设高校开展试点。相关高校可向教育部申请并提交相关专业的招生和人才培养一体化方案。教育部组织专家综合各方面因素, 研究确定强基计划招生高校、专业及规模。

改革招生选拔模式。探索建立基于统一高考的多维度考核评价学生的招生模式。考生参加统一高考和高校考核后,高校根据考生高考成绩、高校综合考核结果及综合素质评价情况等按比例合成考生综合成绩(其中高考成绩所占比例不得低于 85 %) , 按考生综合成绩由高到低进行录取。对于极少数在相关学科领域具有突出才能和表现的考生,有关高校可制定破格入围高校考核的条件和办法,并提前向社会公布,考生须参加统一高考。

创新人才培养模式。高校对通过强基计划录取的学生单独制订培养方案,采取导师制、小班化等培养模式。建立激励机制,增强学生的荣誉感和使命感。畅通成长发展通道, 对学业优秀的学生,高校可在免试推荐研究生、直博、公派留学、奖学金等方面予以优先安排。探索建立本博衔接的培养模式。推进科教协同育人,探索建立结合重大科研任务的人才培养机制。通过强基计划录取的学生入校后原则上不得转到相关学科之外的专业就读。

健全公平公正保障机制。严格高校考核,笔试、面试均须安  排在国家教育考试标准化考点进行,面试采取专家、考生双随机抽签的方式,全 程录音录像。完善信息公开公示,加 强违规违纪查处。将强基计划招生及人才培养工作纳入巡视和督导的工作范围,建立动态准入退出机制。

报考强基计划的考生,按有关试点高校招生简章的要求在教育部阳光高考平台报考,无须在我省志愿填报系统填报强基计划志愿,录取结果由高校确定。我省根据高校确定的拟录取考生名单,在本科提前批录取开始前办理录取备案手续。强基计划考生可正常填报我省其他类高考志愿,如被强基计划录取,将不能参加其他各类型投档和录取。

f43213efbba25e9e7f3e98deb60214fd.png

7.强基计划试点高校有哪些?

2022 年强基计划试点高校有北京大学、中国人民大学、清华大学、北京航空航天大学、北京理工大学、中国农业大学、北京师范大学、中央民族大学、南开大学、天津大学、大连理工大学、东北大学、吉林大学、哈尔滨工业大学、复旦大学、同济大学、上海交通大学、华东师范大学、南京大学、东南大学、浙江大学、中国科学技术大学、厦门大学、山东大学、中国海洋大学、武汉大学、华中科技大学、湖南大学、中南大学、中山大学、华南理工大学、四川大学、重庆大学、电子科技大学、西安交通大学、西北工业大学、西北农林科技大学、兰州大学、国防科技大学。2023 年强基计划试点高校名单,可登录教育部阳光高考平台网站查询或直接向相关招生院校咨询。

f43213efbba25e9e7f3e98deb60214fd.png

8.强基计划如何招生?

有关高校应根据本校的办学定位、学科特色等,制定强基计划招生简章,内容包括领导机构、招生专业及计划、报考条件及方式、入围高校考核的办法、考核程序及办法、学生综合素质评价使用办法、综合成绩折算办法及录取规则、监督机制、咨询及申诉渠道等。招生简章报经教育部核准备案后提前向社会公布。

符合生源所在地当年高考报名条件以及强基计划招生学校报考条件的考生,由本人提出申请,按高校招生简章要求进行网上报名。

报名考生均须参加全国统一高考。对于以高考成绩入围高校考核的,有关高校在各省(区、市)本科一批录取最低控制分数线(合并录取批次省份应单独划定相应分数线,同)上, 按照在生源所在省份强基计划招生名额的一定倍数,以考生高考成绩从高到低确定参加高校考核名单。对符合高校破格入围条件的考生,考生高考成绩应达到高校招生简章确定的要求,且原则上不得低于各省(区、市)本科一批录取最低控制分数线。有关高校应按照教育部规定确定入围高校考核的考生名单并公示入围标准,对入围考生组织高校考核(含笔试、面试)和体育测试,其中体育测试结果作为录取的重要参考。高校考核是国家教育考试的组成部分,由招生高校负责组织实施。有关高校要严格执行教育部关于特殊类型考试招生工作相关规定,合理确定高校考核的内容和形式。积极探索通过笔试、面试、实践操作等方式,考查学生分析问题、解决问题的能力和创新思维,增强选才的科学性。充分运用学生综合素质档案,全面、深入地考察学生的能力和素养。高校考核的笔试、面试应安排在国家教育考试标准化考点进行, 面试采取专家、考生双随机抽签的方式全程录音录像。

有关高校将考生高考成绩、高校综合考核结果及综合素质评价情况等按比例合成考生综合成绩(其中高考成绩所占比例不得低于 85 % ) 并根据考生填报志愿,按综合成绩由高到低确定录取名单, 提交生源所在省级招办办理录取手续。破格录取的考生,按照高校招生简章公布办法进行录取。被录取考生不再参加后续高考志愿录取。有关高校须及时确定录取考生名单并公示录取标准。

具体招生程序及要求以各有关高校公布的招生简章为准。

f43213efbba25e9e7f3e98deb60214fd.png

9.什么是强基计划招生“十严禁”?

高校不得发布未经教育部备案的强基计划招生简章或进行虚假招生宣传;不得以任何形式组织与强基计划招生挂钩的冬令营、夏令营及考核工作,或委托个人或中介组织开展报名、考核等有关工作;高校招生工作人员、专家评委不得参与社会机构组织的各类培训、辅导活动;不得以新生高额奖学金”“入校后重新选择专业等方式进行恶性生源竞争或向考生违规承诺录取;未经批准不得突破强基计划的招生计划录取;不得在发放新生录取通知书或新生入学报到环节更改考生录取专业;通过强基计划录取的学生入校后原则上不得转到相关学科之外的专业就读;省级高校招生委员会和省级教育行政部门不得擅自扩大强基计划招生高校范围或出台与国家招生政策相抵触的招生办法;省级招生考试机构不得为不符合要求的考生或违反规定程序办理录取手续;有关中学等不得出具与事实不符的考生推荐材料、证明材料等或在考生综合素质档案中虚构事实或故意隐瞒事实。

f43213efbba25e9e7f3e98deb60214fd.png

10.普通高校是怎样分类的?

目前,我国具有普通高等学历教育资格的高校有 2700 多所,站在考生填报志愿的角度对高校进行简单分类如下:

按办学层次,可分为本科院校和专科院校等; 按办学类型(办学性质),可分为普通或成人高校、公办或民办院校或独立学院、高等专科学校或高等职业技术学校等;按隶属关系,可分为教育部直属高校、中央其他部门所属高校、省(区、市)所属高校以及行业所属高校等;此外,按照高校主体专业设置,还可分为综合院校、工科院校、农业院校、林业院校、医药院校、师范院校、语言院校、财经院校、政法院校、体育院校、艺术院校、民族院校、军事院校等。

f43213efbba25e9e7f3e98deb60214fd.png

11.公办院校、民办院校与独立学院有哪些区别?

公办院校是指由政府举办的高校;民办院校是指企业事业组织、社会团体及其他社会组织和公民个人利用非国家财政性教育经费,面向社会举办的高校;独立学院则是指实施本科以上学历教育的普通高校与国家机构以外的社会组织或者个人合作,利用非国家财政性经费举办的实施本科学历教育的高校,属民办性质, 其中一些独立学院与校本部共享教学资源。个别高校的举办主体(办学性质)可能会发生变化,考生在填报志愿前,要登录高校官方网站认真查看高校招生章程或向高校咨询。

根据《中华人民共和国民办教育促进法》的规定,民办院校的受教育者在升学、就业、社会优待以及参加先进评选等方与同级同类公办院校的受教育者同等权利。

f43213efbba25e9e7f3e98deb60214fd.png

12.什么是本硕、本硕博和本博连读?

本硕连读专业是指学生完成本科学业要求,可直接攻读硕士研究生;本硕博连读专业, 即本科 、硕士研究生、博士研究生连续攻读,中间没有间隔;本博连读专业,即学生获得学士学位后,直接攻读博士学位,中间不再授予硕士学位,一旦不能完成博士毕业答辩,则只能获得学士学位。

f43213efbba25e9e7f3e98deb60214fd.png

13.什么是本科学术互认课程(ISEC )?

本科学术互认课程 (International Scholarly Exchange Curri­culum以下简称 ISEC)是由国家留学基金管理委员会(直属于教育部的非营利性事业法人机构,China Scholarship Council简称 CSC)依托服务国家国际教育交流合作的丰富资源,着眼于全方位服务国内院校的教育国际化工作,联合国内外院校和国际教育专家研发的集合通用型本科学术互认课程方案、国际标准师资培育平台以及国际化教育资源交流路径为一体的多元化、融通型国际化人才培养服务体系,是我国高等教育领域引进国外优质教育资源和促进国内外院校教育交流的有益尝试。

本科学术互认课程采用中外方高校双向学分互认的创新型人才培养模式,学生就读的国内阶段部分课程可被境外合作院校认可,学 生在国外学习阶段的部分课程可被 ISEC 项目中方高校审核认定。本科阶段专业总计学制一般为四年,除国内正常的四年学制形式外,也可以采用国内、境外两个学习阶段的模式。其中,国内阶段不能少于两年。学生如在境外完成本科最后阶段的学业,满足境外院校对授予学位所规定的学习时间和学业表现要求的,可申请境外大学学士学位。不出国留学的学生,或在境外交流一段时间,回到国内院校完成学业,其课程仍将采用本科学术互认课程的国际化课程。学生顺利完成学业,经毕业资格审查合格后颁发国内院校的学历和学位证书。本科学术互认课程办学信息详见相关高校官方网站或向高校咨询。

f43213efbba25e9e7f3e98deb60214fd.png

14.哪些高校(专业)对考生有特殊要求?

一些高校(专业)在录取时,对考生思想政治品德、身体健康状况、专业特长等方面有特殊要求。主要包括:外语专业、体育专业、艺术专业、军队院校(含武警)、公安院校、司法院校等。这里简单介绍报考上述院校(专业) 的基本要求, 在本书以后的相关章节中还有更详细的介绍。

报考外语专业和有外语口试要求专业的,如高校招生有特殊要求,须按要求参加外语口试。

报考体育专业的, 体育 专业考试。包括体育教育和社会体育指导与管理专业(以下简称普通体育专业)、体育单招、少数民族传统体育项目。

报考高水平运动队的,须是国家二级及以上运动员且参加由招生高校组织(部分项目实施全国统考)的专业测试。

报考艺术专业的,须参加艺术专业考试(部分无须专业考试,直接按高考文化课成绩录取的艺术统考未涉及的校考专业除外)。可授予艺术学学位的非艺术类专业,如高校有要求,也须参加相应的专业测试。

报考高水平艺术团的,须参加招生高校组织的专业测试。

报考用蒙语授课专业的考生,应具有一定的蒙语基础,须加试蒙语文。

报考军队院校的,须是普通高中毕业(应、往届毕业生均可),普通高中学业水平考试合格性考试体育与健康科目成绩为“合格”, 且须参加政治考核、面试和军检。

报考飞行学员的,须本着自愿的原则,符合飞行员体检要求和心理选拔测评要求,并进行相应的政治考核。

报考公安、司法院校(专业)的,应按要求参加公安或司法院校组织的(政治)考察、体检(或体能测试)。报考公安院校(专业)须普通高中毕业。

报考少年班的,应是德智体美劳全面发展、智力超常、学习成绩优异,须按高校招生简章要求申请并参加相关测试。

报考需要提前进行面试、体检的空中乘务等专业的,应按高校要求提前进行面试和体检。

f43213efbba25e9e7f3e98deb60214fd.png

15.普通体育招生、高水平运动队招生、体育单招有什么区别?

报考普通体育专业的考生,须参加全省统一组织的体育专业考试。文化课考试参加全国统考。其中报考河北体育学院少数民族传统体育项目的,须是少数民族考生,且须参加河北体育学院单独组织的专业测试,但不要求参加全省统一组织的体育专业考试。

报考高水平运动队,应是高级中等教育学校毕业,获得国家二级运动员(含)以上证书且高中阶段在省级(含)以上比赛中获得集体项目前六名的主力队员或个人项目前三名者;或是具有高级中等教育学校毕业同等学力,获得国家一级运动员(含)以上证书者或近三年内在全国或国际集体项目比赛中获得前八名的主力队员。报考高水平运动队的考生,须在规定时间参加招生院校组织的专业测试(部分项目实施全国统考)。获得高水平运动队资格的考生,还要参加文化课全国统一高考。获得一级运动员、运动健将、国际健将及武术武英级(或以上)称号之一的考生, 可申请参加国家单独组织的统一文化课考试,也可选择参加全国统一招生考试。入学后进入非体育专业学习。

  报考体育单招,应具备运动训练、武术与民族传统体育专业招生所列项目二级运动员(含)以上技术等级称号。考生须参加体育专项考试(全国统考和分区统考方式),参加省级招生考试机构组织的文化考试。

f43213efbba25e9e7f3e98deb60214fd.png

16.体育单招如何招生?

体育单招实行文化考试和体育专项考试相结合的办法。文化考试由生源地省级招生考试机构组织实施,体育专项考试分项目采用全国统考和分区统考方式,由国家体育总局委托高校组织实施。

报考的考生应当符合以下条件:符合2023年高考报名条件;具备下述所列项目的二级运动员(含)以上运动技术等级称号。

报考体育单招的考生应参加其户口所在地招办统一组织的高考报名,报考考试类型为“统考”。考生还须依据各高校招生简章要求,在规定时间内登录“中国运动文化教育网”(网址:http://www.ydyeducation.com ) 或“体教联盟" APP体育单招管理系统进行注册(验证考生报考资格)并报名。具体时间:

冬季项目:

注册时间为 2022 12 1 12 : 00 12 31 12 : 00

报名时间为 2022 12 22 12 : 00 12 31 12 : 00

夏季项目(含武术与民族传统体育专业项目,下同): 注册时间为 2023 2 1 12 : 00 3 10 12: 00

报名时间为 2023 3 1 12 : 00 3 10 12: 00

2023 年报名考生的等级证书审批日期:冬季项目为 2013 11 日至 2022 12 31 日;夏季项目为 2013 1 1 日至2023 3 10 日。

运动训练、武术与民族传统体育专业招生文化考试时间:

2023 4 15 日至 16 日。体育专项考试时间:冬季项目考试时间为 2023 1 4 日至 5 10 日;夏季项目考试时间为 2023 3 20 日至 5 10 日。

考生若已报名运动训练、武术与民族传统体育专业志愿并被录取,不得放弃录取资格,同时不再参加普通高考及高校高水平运动队的录取。

达到免试条件的运动员(含退役运动员)在履行相关手续后,可进入相关高校学习深造。具体招生事宜可向招生院校咨询。

举办运动训练专业的招生院校有:

北京体育大学、上海体育学院、武汉体育学院、西安体育学院、成都体育学院、沈阳体育学院、首都体育学院、天津体育学院、河北体育学院、吉林体育学院、哈尔滨体育学院、南京体育学院、山东体育学院、广州体育学院、北京师范大学、河北师范大学、山西大学、中北大学、山西师范大学、内蒙古师范大学、内蒙古民族大学、辽宁师范大学、沈阳师范大学、大连大学、大连理工大学(盘锦校区)、辽宁工程技术大学、吉林大学、东北师范大学、吉林师范大学、延边大学、北华大学、长春师范大学、哈尔滨师范大学、华东师范大学、苏州大学、中国矿业大学、江苏师范大学、浙江大学、宁波大学、安徽师范大学、合肥师范学院、福建师范大学、集美大学、华东交通大学、南昌大学、江西师范大学、井冈山大学、赣南师范大学、宜春学院、中国海洋大学、山东理工大学、烟台大学、聊城大学、曲阜师范大学、河南大学、郑州大学、河南师范大学、洛阳师范学院、南阳师范学院、黄河科技学院、华中师范大学、江汉大学、中南大学、湖南师范大学、湖南工业大学、衡阳师范学院、华南理工大学、华南师范大学、深圳大学、嘉应学院、广西师范大学、海南师范大学、西南大学、重庆师范大学、西华师范大学、贵州师范大学、云南师范大学、陕西师范大学、陕西理工学院、西北师范大学、天水师范学院、青海师范大学、青海民族大学、宁夏大学、新疆师范大学、石河子大学、张家口学院、内蒙古大学、湖南人文科技学院、广西大学、西藏民族大学、同济大学、暨南大学、河北工程大学、廊坊师范学院、华中科技大学、重庆文理学院、邯郸学院、南京工业大学、武汉体育学院体育科技学院、西安电子科技大学、华侨大学、太原理工大学、齐齐哈尔大学、巢湖学院、湖南科技大学、昆明理工大学、东北大学、河海大学、东北电力大学、江西财经大学、湖北师范大学、武汉商学院、广东工业大学、广州商学院、长江师范学院、宁夏师范学院等117 所。

举办武术与民族传统体育专业的招生院校有:

北京体育大学、上海体育学院、武汉体育学院、西安体育学院、成都体育学院、沈阳体育学院、首都体育学院、天津体育学院、河北体育学院、吉林体育学院、哈尔滨体育学院、南京体育学院、山东体育学院、广州体育学院、河北师范大学、山西师范大学、晋中学院、内蒙古民族大学、沈阳师范大学、东北师范大学、哈尔滨师范大学、苏州大学、扬州大学、江苏师范大学、浙江大学、杭州师范大学、阜阳师范大学、集美大学、江西师范大学、山东师范大学、鲁东大学、菏泽学院、河南大学、郑州大学、河南理工大学、洛阳师范学院、商丘师范学院、黄河科技学院、湖南师范大学、吉首大学、广西师范大学、海南师范大学、贵州师范大学、云南师范大学、云南民族大学、西北师范大学、宁夏大学、青海师范大学、青海民族大学、山西大学、西华师范学院、三亚学院、呼和浩特民族学院、邯郸学院、沧州师范学 院、武汉体育学院体育科技学院、湖南工业大学、长江师范学院等58所。

运动训练专业所设项目为:

冬季项目:速度滑冰、短道速滑、花样滑冰、冰球、冰壶、越野滑雪、高山滑雪、跳台滑雪、自由式滑雪(雪上技巧、空中技巧)、单板滑雪(平行大回转、U型场地技巧)、冬季两项。

夏季项目:射击、射箭、场地自行车、公路自行车、山地自行车、BMX小轮车、击剑、现代五项、铁人三项、马术、帆船、赛艇、皮划艇静水、皮划艇激流回旋、冲浪、蹼泳、滑水、摩托艇、举重、自由式摔跤、古典式摔跤、柔道、拳击、眙拳道、空手道、田径、游泳、公开水域游泳、跳水、水球、花样游泳、体操、艺术体操、蹦床、技巧、手球、曲棍球、棒球、垒球、足球(十一人制)、篮球、排球、沙滩排球、乒乓球、羽毛球、网球、橄榄球、高尔夫球、围棋、象棋、国际象棋、登山、攀岩。

其中,篮球项目考生使用篮球或三人篮球运动员技术等级证书均可报名,按照篮球项目考试方法与评分标准参加篮球专项考试;橄榄球项目考生使用七人制或十五人制运动员技术等级证书均可报名,按照七人制橄榄球考试方法与评分标准参加橄榄球专项考试。

武术与民族传统体育专业所设项目为:武术套路、武术散打、中国式摔跤。

f43213efbba25e9e7f3e98deb60214fd.png

17.高水平运动队如何招生?

考生根据高水平运动队招生试点高校确定的报名条件,申请参加学校组织的运动队对应项目专业测试;试点高校确定并公示合格考生名单及享受高考文化课成绩优惠的相应录取要求;按照本校公布的办法,择优录取专业测试合格、高考文化成绩达到相应录取要求的考生。

专业测试分项目采取全国统考、高校联考和高校校考相结合的组织方式。足球、乒乓球、羽毛球、游泳、田径、武术、跆拳道、击剑、棒球、沙滩排球、冰雪等11个项目实施全国统考,与运动训练、武术与民族传统体育专业招生的体育专项考试统一组织,统一采用国家体育总局审定的《普通高等学校运动训练、武术与民族传统体育专业体育专项考试方法与评分标准》进行评分,各地各高校不再组织相关项目的省级统考、校考。其他项目暂由试点高校独立或联合组织。

试点高校要按照本校发展定位和人才培养要求,合理确定本校运动队录取考生高考文化课成绩最低要求方案,一般不低于生源省份本科第二批次录取控制分数线;对于少数体育测试成绩特别突出的考生,高校可适度降低文化成绩录取要求,但不得低于生源省份本科第二批次录取控制分数线的65% (对于合并本科批次省份的情况,高校可参照艺术类专业文化课划线有关要求予以适当提高),高校公示的此类考生人数不得超过学校当年高水平运动队招生计划的30 %。试点高校面向一级运动员等组织的部分运动员单独招生工作,文化课考试与运动训练专业单独招生文化课考试统一组织,高水平运动队录取此类考生的文化课最低线不得低于运动训练、武术与民族传统体育专业统一划定的文化课最低线,高校确定并公示的此类考生拟录取人数不得超过本校当年运动队招生计划的20%

试点高校按照教育部、国家体育总局关于进一步完善和规范高校高水平运动队(以下简称运动队)考试招生工作的有关要求,进一步明晰运动队招生试点工作定位,在奥运会、世界大学生运动会项目(包括足球、篮球、排球项目等)范围内,按照教育部评估确定的项目,结合学校实际,根据本校运动队建设规划,确定运动队招生项目和招生计划。对于不具备相关师资、设备、场地等组队条件、退队率超过20%的高校,和非奥运会或世界大学生运动会项目、未设运动员技术等级标准、生源严重不足且连续两年录取数为零的相关项目,不再安排运动队招生。本校运动训练、武术与民族传统体育专业已涉及的运动项目,原则上不安排运动队招生。要大力加强从在校生中选拔培养运动队成员。各地各试点高校要健全管理制度, 严格政策执行,畅通监督举报渠道,加大违规查处力度,确保公平公正。高校要深入落实招生信息公开各项要求,加强招生政策公开、考生资格信息公开、录取结果公开,主动接受社会监督。

f43213efbba25e9e7f3e98deb60214fd.png

18.2024年起高水平运动队招生将有什么变化?

优化招生项目。招生高校在奥运会、世界大学生运动会项目(包括足球、篮球、排球项目等)范围内,确定运动队招生项目和招生计划。对于不具备相关师资、设备、场地等组队条件、学生入校后退队率超过20%的高校,以及非奥运会或世界大学生运动会项目、未设运动员技术等级标准、生源严重不足且连续两年录取数为零的相关项目,不再安排高水平运动队招生。原则上连续三届奥运会、世界大学生运动会没有学生参赛的项目,不再安排高水平运动队招生。

严格报考条件。从2024年起,符合生源省份高考报名条件, 获得国家一级运动员(含)以上技术等级称号者方可以报考高水平运动队。从2027年起,符合生源省份高考报名条件,获得国家一级运动员(含)以上技术等级称号且近三年在国家体育总局、教育部规定的全国性比赛中获得前八名者方可以报考高水平运动队。有关高校可进一步提高报考本校高水平运动队的具体条件要求。

高校还会加强考生报考资格审核,加强对考生运动员技术等级证书的核验,同时对考生的基本报考信息、运动员技术等级证书及参加赛事名次等予以公示,接受社会监督。

改进考试方式。高校高水平运动队招生采取“文化考试+ 专业测试”相结合的考试评价方式。2024年起,文化考试成绩全部使用全国统一高考文化课考试成绩,专业测试全部纳入全国统考并由国家体育总局牵头组织实施,高校不再组织相关专业测试。

提高成绩要求。2024年起, 招收高水平运动队的“世界一流大学建设高校”,对考生的高考成绩要求须达到生源省份本科录取最低控制分数线;其他高校对考生的高考成绩要求须达到生源省份本科录取最低控制分数线的 80%。对于体育专业成绩突出、具有特殊培养潜质的考生,允许高校探索建立文化课成绩破格录取机制。

明确专业范围。2024年起,高水平运动队录取学生中,高考文化课成绩不低于招生高校相关专业在生源省份录取分数线下 20 分的学生,可申请就读相应的普通专业;其余学生限定就读体育学类专业,原则上不得转到其他类专业就读。

除此之外,高校还将加强高水平运动队学生入校培养管理,严格组织新生入学复查,要求学生认真履行参加训练和比赛的义务,实施严格的学业标准。

f43213efbba25e9e7f3e98deb60214fd.png

19.招收高水平运动队的试点院校和项目有哪些?

2022 年招收高水平运动队的院校(含项目)名单

序号 省份 学校名称       招生项目

1 北京 北京大学          田径、篮球、足球、乒乓球、羽毛球、游泳

2 北京 中国人民大学       田径、篮球、排球、足球、网球、武术

北京 清华大学         田径、篮球、排球、游泳、击剑、足球、射击

4 北京 北京交通大学      田径、排球、羽毛球、跆拳道

5 北京 北京科技大学       田径、篮球、足球、羽毛球、跆拳道

6 北京 中国石油大学(北京)  田径、羽毛球、游泳

7 北京 北京邮电大学       乒乓球、武术

8 北京 华北电力大学       田径、篮球

9 北京 北京化工大学       篮球、网球

10 北京 中国农业大学       田径、足球

11 北京 北京林业大学       田径、足球

12 北京 北京中医药大学     武术、足球

13 北京 北京师范大学       篮球、足球、排球

14 北京 北京外国语大学     游泳

15 北京 对外经济贸易大学    足球

16 北京 中央财经大学       排球

17 北京 中国政法大学       排球、乒乓球、羽毛球、足球

18 北京 中央民族大学       网球、排球

19 北京 北京理工大学       田径、足球、武术

20 北京 北京航空航天大学    田径、排球、足球、羽毛球

21 北京 北京联合大学       足球

22 北京 北京工业大学      篮球、足球、排球、游泳、羽毛球

23 北京 北方工业大学        棒球

24 北京 首都经济贸易大学          篮球、游泳

25 北京 北京石油化工学院          田径

26 北京 中国地质大学(北京)  田径、眙拳道

27 天津 南开大学        田径、排球

28 天津 天津大学        田径、篮球、游泳

29 天津 中国民航大学       篮球、足球

30 天津 天津科技大学       乒乓球

31 天津 天津理工大学       武术、跆拳道

32 天津 天津医科大学       游泳

33 天津 天津外国语大学     网球、击剑

34 天津 天津财经大学       田径、篮球

35 河北 河北农业大学       田径、排球

36 河北 河北地质大学       足球、田径

37 山西 太原理工大学       田径、篮球、足球

38 山西 中北大学          排球

39 山西 山西大同大学       足球

40 山西 山西医科大学       足球

41 山西 山西财经大学       田径、篮球

42 山西 太原师范学院       游泳、足球

43 内蒙古 内蒙古大学      足球

44 内蒙古 内蒙古科技大学    田径、篮球、排球、足球

45 内蒙古 内蒙古工业大学    排球、足球

46 内蒙古 内蒙古农业大学    田径、篮球、足球

47 辽宁 大连理工大学       田径、篮球

48 辽宁 东北大学        田径、篮球

49 辽宁 辽宁大学        足球、篮球

50 辽宁 沈阳农业大学       排球

51 辽宁 沈阳理工大学       足球

52 辽宁 沈阳工业大学       足球

53 辽宁 沈阳建筑大学       游泳、网球

54 辽宁 辽宁石油化工大学         羽毛球、击剑

55 辽宁 沈阳化工大学       排球、网球

56 辽宁 东北财经大学       田径、篮球

57 辽宁 大连海事大学       足球、游泳

58 吉林 吉林大学        田径、排球、足球、乒乓球

59 吉林 东北师范大学       冰雪

60 吉林 延边大学        足球

61 吉林 北华大学        足球

62 吉林 吉林农业大学       篮球、足球

63 吉林 东北电力大学       冰雪、足球、田径、网球

64 吉林 吉林化工学院       足球

65 吉林 长春工程学院       篮球、足球

66 黑龙江 哈尔滨工业大学    冰雪、篮球

67 黑龙江 哈尔滨工程大学    田径、篮球、排球

68 黑龙江 东北石油大学     田径

69 黑龙江 东北农业大学     田径

70 黑龙江 哈尔滨师范大学    冰雪、足球

71 上海 复旦大学        田径、排球、游泳、武术、足球

72 上海 同济大学        田径、游泳、羽毛球

73 上海 上海交通大学      田径、篮球、游泳、乒乓球、网球、羽毛球、足球

74 上海 华东理工大学      田径、篮球、乒乓球、网球、武术

75 上海 东华大学        田径、足球

76 上海 华东师范大学       田径、篮球、排球

77 上海 上海外国语大学     棒球、排球

78 上海 上海财经大学       游泳、网球

79 上海 上海大学        田径、排球、网球、武术、足球

80 上海 上海理工大学        田径、足球

81 上海 上海海事大学        游泳、武术、足球

82 上海 上海工程技术大学     足球、篮球

83 上海 上海海洋大学        田径

84 上海 上海中医药大学       排球、武术

85 上海 华东政法大学        棒球、足球

86 上海 上海立信会计金融学院  击剑、足球

87 上海 上海电力大学       击剑、足球

88 上海 上海对外经贸大学          冰雪

89 江苏 南京大学          田径、篮球、排球、足球

90 江苏 东南大学         田径、排球、游泳、乒乓球、篮球、羽毛球

91 江苏 中国矿业大学       篮球、网球、足球

92 江苏 河海大学          足球、乒乓球

93 江苏 江南大学        足球

94 江苏 南京农业大学       排球、武术、网球、足球

95 江苏 中国药科大学       排球

96 江苏 南京理工大学       田径、篮球、足球

97 江苏 南京航空航天大学          田径、篮球、足球、乒乓球

98 江苏 苏州大学        田径、游泳、篮球、足球

99 江苏 扬州大学        田径、乒乓球、篮球

100 江苏 江苏大学         排球、沙滩排球、足球

101 江苏 南京邮电大学      篮球、足球

102 江苏 南京工业大学      田径、乒乓球

103 江苏 南京师范大学     田径、足球、排球

104 江苏 南京财经大学     田径、篮球、羽毛球

105 江苏 南通大学       田径、击剑、乒乓球

106 江苏 南京信息工程大学       排球、乒乓球、跆拳道、足球

107 江苏 苏州科技大学     羽毛球

108 浙江 浙江大学       田径、篮球、网球、排球

109 浙江 宁波大学       篮球、足球

110 浙江 浙江工业大学     田径、羽毛球、足球

111 浙江 浙江理工大学     田径、篮球、足球

112 浙江 浙江中医药大学    武术、足球

113 浙江 浙江师范大学     田径、篮球、足球

114 浙江 浙江工商大学     篮球、足球

115 浙江 浙江财经大学     游泳、乒乓球、足球

116 安徽 合肥工业大学     篮球、足球

117 安徽 安徽大学       羽毛球

118 安徽 安徽农业大学     足球

119 安徽 安徽建筑大学     乒乓球、击剑

120 安徽 合肥学院       击剑

121 福建 厦门大学       足球、篮球、棒球、武术

122 福建 福州大学       游泳、篮球、足球

123 福建 厦门理工学院     篮球

124 江西 南昌大学       篮球、排球、网球

125 江西 华东交通大学     武术

126 江西 南昌航空大学     羽毛球

127 江西 井冈山大学      篮球、足球

128 江西 江西科技师范大学       足球

129 山东 山东大学       田径、排球、篮球、足球、游泳

130 山东 中国海洋大学     田径、排球

131 山东 青岛大学       田径、足球

132 山东 山东科技大学     篮球、田径

133 山东 聊城大学       田径

134 山东 青岛科技大学     足球

135 山东 济南大学       足球、网球

136 山东 山东农业大学     田径、篮球、武术

137 山东 山东中医药大学    乒乓球、武术

138 山东 山东师范大学     田径、排球、乒乓球、足球

139 山东 鲁东大学       田径、篮球

140 山东 山东财经大学     排球、足球、乒乓球

141 山东 齐鲁工业大学     足球

142 山东 中国石油大学(华东) 田径、篮球、排球、乒乓球

143 山东 青岛理工大学     足球

144 河南 郑州大学       田径、篮球、乒乓球、足球

145 河南 河南理工大学     篮球、乒乓球、足球

146 河南 河南财经政法大学    田径、武术、足球

147 湖北 武汉大学        田径、排球、乒乓球、羽毛球、足球

148 湖北 江汉大学       足球

149 湖北 华中科技大学     田径、篮球、乒乓球、武术

150 湖北 武汉理工大学     田径、篮球、网球、武术

151 湖北 中国地质大学(武汉)田径、游泳、羽毛球、篮球

152 湖北 华中农业大学     羽毛球、田径

153 湖北 华中师范大学     羽毛球、游泳

154 湖北 湖北中医药大学    武术、足球

155 湖北 湖北大学       田径、足球、武术、乒乓球

156 湖北 三峡大学       武术、足球、篮球

157 湖北 武汉科技大学     田径、篮球、足球

158 湖北 湖北工业大学     篮球、乒乓球

159 湖北 中南民族大学     足球

160 湖北 武汉轻工大学     篮球、足球、羽毛球

161 湖北 湖北经济学院     田径、排球、足球

162 湖南 中南大学       田径、篮球、排球、乒乓球

163 湖南 湖南大学       田径、篮球

164 湖南 湘潭大学       田径、篮球、羽毛球、足球

165 湖南 长沙理工大学     田径、足球

166 湖南 南华大学       排球

167 湖南 湖南工业大学     田径、篮球、排球

168 湖南 湖南工商大学     网球

169 湖南 湖南农业大学     足球

170 湖南 湖南师范大学     篮球、排球、乒乓球、足球

171 湖南 湘南学院       篮球、足球

172 湖南 中南林业科技大学       田径、排球

173 湖南 湖南科技大学     足球

174 广东 中山大学       田径、排球、击剑

175 广东 华南理工大学     田径、篮球、游泳、乒乓 球足球

176 广东 暨南大学       田径、游泳、羽毛球、网球、武术

177 广东 深圳大学       乒乓球

178 广东 广东工业大学     篮球、足球、乒乓球、网球

179 广东 华南农业大学     羽毛球、足球、跆拳道

180 广东 华南师范大学     游泳、乒乓球、武术

181 广东 广州大学       篮球、足球

182 广东 韩山师范学院     田径、足球

183 广东 广东财经大学     田径、游泳

184 广东 汕头大学       篮球

185 广西 广西大学       篮球、游泳、乒乓球、羽毛球

186 广西 桂林电子科技大学       网球

187 广西 广西师范大学     足球

188 广西 广西民族大学     足球、篮球

189 广西 广西中医药大学    羽毛球

190 广西 南宁师范大学     足球、田径、篮球

191 海南 海南大学       田径、篮球、排球、足球

192 海南 海南热带海洋学院      足球

193 重庆 重庆大学       篮球、网球、足球

194 重庆 重庆师范大学     田径、篮球、足球

195 重庆 重庆工商大学     排球、足球

196 重庆 西南政法大学     田径、排球、篮球

197 重庆 重庆科技学院     足球

198 重庆 重庆文理学院     篮球、足球

199 四川 四川大学       田径、排球、足球、游泳、网球

200 四川 西南交通大学     田径、篮球、排球

201 四川 电子科技大学     田径、篮球、足球、游泳

202 四川 西南财经大学     篮球、网球、足球

203 四川 西华大学       足球、跆拳道、乒乓球

204 四川 成都中医药大学    武术

205 四川 西南石油大学     网球

206 四川 吉利学院       足球

207 四川 西南科技大学     足球

208 四川 成都大学       足球

209 贵州 贵州大学       田径、篮球、乒乓球、足球

210 贵州 贵州师范大学     足球

211 云南 云南大学       田径、足球

212 云南 云南农业大学     篮球、足球

213 云南 昆明理工大学     排球

214 云南 云南财经大学     篮球、网球、足球、排球

215 陕西 西安交通大学     田径、篮球、足球、游泳、乒乓球

216 陕西 长安大学       足球

217 陕西 西安电子科技大学      游泳

218 陕西 西北农林科技大学    网球

219 陕西 陕西师范大学     武术

220 陕西西北工业大学     田径、篮球、排球、乒乓球、羽毛球

221 陕西 西安理工大学     田径、游泳

222 陕西 西安建筑科技大学       田径、足球、网球、游泳

223 陕西 西安工业大学     篮球、足球、田径

224 陕西 西安邮电大学     田径

225 陕西 西安财经大学     足球

226 陕西 西北大学       足球、冰雪

227 甘肃 兰州大学       田径、排球、武术、足球

228 甘肃 西北民族大学     田径、足球

229 甘肃 兰州交通大学     田径、乒乓球、足球

230 甘肃 甘肃政法大学     足球、冰雪

231 甘肃 西北师范大学     足球

232 青海 青海师范大学     篮球

233 宁夏 宁夏大学       足球

234 新疆 新疆大学       田径、篮球、足球

2023年招收高水平运动队的院校(含项目)名单如有调整,以教育部相关文件为准,考生也可向相关招生院校咨询。

f43213efbba25e9e7f3e98deb60214fd.png

20.艺术类专业考试形式有哪些?

艺术类专业考试分三种形式:一是河北省统一组织的专业考试(简称统考),二是高校联合组织的专业考试(简称校际联考),三是高校单独组织的专业考试(简称校考)。其中,高考报名时选报声乐类、器乐类、舞蹈类、美术类等艺术统考类别的考生,也可参加校际联考和校考专业的考试和录取;选报”不参加艺术统考的校考类(含校际联考)”的考生,不能参加艺术统考的考试和录取,也不能参加对统考成绩有要求的艺术校考专业的录取。

(1 ) 统考

我省对美术类、音乐类(分声乐类、器乐类)、舞蹈类专业实行统考。

①美术类专业统考。考生报考以下对美术技能有专业考核要求且符合教育部艺术类专业设置规定的专业,均应参加美术类专业统考。

本科专业:戏剧影视美术设计、动画、美术学、绘画、雕塑、摄影、中国画、实验艺术、跨媒体艺术、文物保护与修复、漫画、纤维艺术、艺术设计学、视觉传达设计、环境设计、产品设计、服装与服饰设计、公共艺术、工艺美术、数字媒体艺术、陶瓷艺术设计、新媒体艺术、包装设计、环境艺术设计、美术、公共艺术设计、游戏创意设计、展示艺术设计、数字影像设计、时尚品设计、舞台艺术设计、科技艺术、美术教育、珠宝首饰设计与工艺等专业。

专科专业:艺术设计、视觉传达设计、数字媒体艺术设计、产品艺术设计、服装与服饰设计、环境艺术设计、书画艺术、公共艺术设计、游戏艺术设计、展示艺术设计、美容美体艺术、工艺美术品设计、广告艺术设计、室内艺术设计、家具艺术设计、动漫设计、人物形象设计、摄影与摄像艺术、雕刻艺术设计、皮具艺术设计、包装艺术设计、陶瓷设计与工艺、首饰设计与工艺、玉器设计与工艺、刺绣设计与工艺、雕塑设计、服装陈列与展示设计、舞台艺术设计与制作、民族美术、民族服装与饰品、民族传统技艺、影视动画、影视多媒体技术等专业。

其中摄影、动画、戏剧影视美术设计等本科专业,动漫设计、游戏艺术设计、人物形象设计、美容美体艺术、摄影与摄像艺术、舞台艺术设计与制作、影视多媒体技术和影视动画等专科专业, 一般应按美术类招生。如高校招生有特殊需要, 上述专业不按美术类招生的, 须在招生简章中公布。

美术类专业统考由河北师范大学具体组织实施,考点设在各市。

②音乐类专业统考(分声乐统考和器乐统考)。考生报考以下对声乐或器乐专项技能有专业考核要求且符合教育部艺术类专业设置规定的专业,均应参加相应的音乐类专业统考。

本科专业:音乐表演、音乐学、作曲与作曲技术理论、流行音乐、音乐治疗、音乐教育、录音艺术等专业。

专科专业:音乐表演、现代流行音乐、录音技术与艺术、音乐制作、钢琴伴奏、钢琴调律、作曲技术等专业。

其中录音艺术等本科专业,录音技术与艺术等专科专业,一般应按音乐类招生。如高校招生有特殊需要,上述专业不按音乐类招生的,须在招生简章中公布。

音乐类专业统考由河北经贸大学具体组织实施。

③舞蹈类专业统考。考生报考以下对舞蹈专项技能有专业考核要求且符合教育部艺术类专业设置规定的专业,均应参加舞蹈类专业统考。

本科专业:舞蹈表演、舞蹈学、舞蹈编导、舞蹈教育、舞蹈表演与编导等专业。

专科专业:舞蹈表演、舞蹈编导等专业。舞蹈类专业统考由保定学院具体组织实施。

以上①至③所包含的专业如与教育部要求不一致,执行教育部规定。

2)校际联考

戏剧与影视学类、书法学、播音与主持艺术、服装表演类、航空服务艺术与管理等五类专业实行校际联考。2023年各类校际联考承办院校分别为:河北大学承办戏剧与影视学类、书法学测试;河北地质大学承办播音与主持艺术测试;河北科技大学承办服装表演类测试;河北民族师范学院承办航空服务艺术与管理测试。各类别校际联考考试安排、成绩发布等事项详见相关承办院校发布的报考公告。

河北省普通高校招生艺术类专业校际联考各类别涉及专业如下:

①戏剧与影视学类。本科专业:戏剧学、电影学、戏剧影视文学、广播电视编导、戏剧影视导演、影视摄影与制作、影视技术、戏剧教育、音乐剧、影视编导;专科专业:广播影视节目制作、影视编导。

②书法学。本科专业:书法学;专科专业:书画艺术。

③播音与主持艺术。本科专业:播音与主持艺术、播音与主持;专科专业:播音与主持。

④服装表演类.本科专业:服装与服饰设计(服装设计与表演);专科专业:时尚表演与传播。

⑤航空服务艺术与管理。本科专业:航空服务艺术与管理。

3)校考

①我省省级统考已涉及的专业,高校一般应直接使用统考成绩作为考生的专业考试成绩。确有必要补充考核的艺术类本科专业,高校应面向省级统考合格生源(其中美术类本科专业须我省美术类专业统考本科合格)组织校考。省级统考未涉及的艺术类专业,高校可组织校考。

②对于可授予艺术学学士学位的艺术教育、服装设计与工程、风景园林、文化产业管理等4个非艺术学门类专业,高校若对考生有艺术专业基础要求,须在招生章程中明确告知考生应参加的专业考试科类,我省不统一组织此类专业统考。

③高校应在艺术类专业招生章程中明确告知考生须参加河北省艺术统考的类别,及时发布艺术类专业子科类对应关系。对于我省统考类别涉及的艺术类专业,如高校有特殊选拔要求,不需要我省相应类别专业统考合格直接使用校考成绩录取的,须在招生章程中明确向社会公布。

④高校组织校考的专业,其考试时间、地点由高校负责公布,考生参加院校组织的校考需自行向院校索取或登录高校网站查询招生简章,按招生简章和院校要求参加专业考试。

⑤考生报考我省艺术统考涉及的校考专业,须相应类别统考合格(其中美术类本科专业须我省美术类专业统考本科合格),如无法确定所报考专业是否属于艺术类省级统考涉及的专业,应先报名参加相应类别省级统考。

f43213efbba25e9e7f3e98deb60214fd.png

21.独立设置艺术院校(含参照执行院校)及相关专业有哪些?

独立设置本科艺术院校有:中央戏剧学院、中央美术学院、中央音乐学院、中国音乐学院、北京电影学院、北京舞蹈学院、中国戏曲学院、天津音乐学院、天津美术学院、鲁迅美术学院、沈阳音乐学院、吉林艺术学院、上海音乐学院、上海戏剧学院、南京艺术学院、中国美术学院、景德镇陶瓷大学(原景德镇陶瓷学院)、山东艺术学院、山东工艺美术学院、武汉音乐学院、广州美术学院、星海音乐学院、广西艺术学院、四川美术学院、云南艺术学院、西安音乐学院、新疆艺术学院、湖北美术学院等 28 所高校。

参照独立设置艺术院校招生的院校及专业有:清华大学(下属美术学院)、中国传媒大学、北京服装学院、天津工业大学、哈尔滨音乐学院、东华大学、上海视觉艺术学院(原复旦大学上海视觉艺术学院)、上海大学(下属上海美术学院、上海电影学院、音乐学院)、江南大学等9所高校的艺术类本科专业;北京印刷学院(限视觉传达设计、数字媒体艺术、动画、绘画等4个本科专业)、内蒙古艺术学院(原内蒙古大学艺术学院,限音乐表演、表演、音乐学等3个具有蒙古族特色的本科专业)、苏州大学(限视觉传达设计、环境设计、产品设计、服装与服饰设计等4个专业)、浙江传媒学院(限播音与主持艺术、广播电视编导、摄影、录音艺术、影视摄影与制作等5个专业)、浙江理工大学(限服装与服饰设计、视觉传达设计、环境设计、产品设计、数字媒体艺术等5个专业)、浙江音乐学院(限音乐学、音乐表演、作曲与作曲技术理论、舞蹈表演、舞蹈学、舞蹈编导、表演、艺术与科技等8个专业)、武汉设计工程学院(下属成龙影视传媒学院,限 表演、播音与主持艺术、戏剧影视美术设计等3个专业)等7所高校的部分艺术类本科专业。

可组织校考的院校及专业有:河北美术学院(仅可组织校考,且限书法学、雕塑、艺术与科技、动画、数字媒体艺术、广播电视编导、服装与服饰设计、戏剧影视美术设计、影视摄影与制作、表演、播音与主持艺术等11个本科专业)、中央民族大学(可组织校考,但不得单独划定文化线,音乐表演专业须安排分省计划)。

以上院校名单及专业如有调整,以教育部有关规定为准。

f43213efbba25e9e7f3e98deb60214fd.png

22.艺术类专业招生与高水平艺术团招生有什么区别?

艺术类专业招生和高水平艺术团招生是两种不同的招生形式。

艺术类专业考生报考的是高等艺术院校或者普通高校中的艺术类专业,如音乐、美术等专业。文化课和专业成绩均单独划线,填报艺术专业志愿, 入学后进入艺术专业学习。

高水平艺术团招生专业是普通高校非艺术本科专业,如建筑、通信、中文等。具备高水平艺术团招生资格的考生,录取时按照有关规定享受一定分数的优惠,入学后进入非艺术专业学习,按照高校的要求参加艺术活动。

按照国家有关规定,艺术团招生主要招收艺术团首席表演者或对幼功要求高的相关专业项目的艺术团成员。已开设有艺术类专业的试点高校不再单独招收相应专业项目的高水平艺术团考生,美术、书法、播音与主持专业不得纳入艺术团招生范围。

2024年起,高校高水平艺术团不再从高校招生环节选拔, 由相关高校从高校在校生中遴选培养。

f43213efbba25e9e7f3e98deb60214fd.png

23.高水平艺术团招生的院校有哪些?

  经教育部批准,清华大学、北京大学、中国人民大学、北京交通大学、北京航空航天大学、北京理工大学、北京师范大学、中国政法大学、北京科技大学、北京化工大学、北京邮电大学、中国农业大学、北京林业大学、北京中医药大学、中国地质大学(北京)、中国地质大学(武汉)、对外经济贸易大学、南开大学、天津大学、大连理工大学、吉林大学、哈尔滨工业大学、复旦大学、同济大学、上海交通大学、上海财经大学、华东师范大学、东华大学、南京大学、东南大学、南京航空航天大学、南京理工大学、河海大学、南京农业大学、中国药科大学、浙江大学、厦门大学、山东大学、中国海洋大学、武汉大学、华中科技大学、华中农业大学、湖南大学、中南大学、中山大学、华南理工大学、重庆大学、四川大学、西南交通大学、电子科技大学、西安交通大学、西北工业大学、西北农林科技大学、长安大学等54所普通高校开展高水平艺术团招生。

  具有艺术特长的考生可查阅相关院校的招生简章或直接与招生院校联系。

f43213efbba25e9e7f3e98deb60214fd.png

24.什么是“定向就业招生”?

为了帮助边远地区、少数民族地区和工作环境比较艰苦的行业培养人才,委托培养单位与普通高校签订协议, 高校拿出部分计划进行“定向就业招生”。考生填报了定向志愿即视为愿意到定向单位就业。

  目前,我省招收的定向生有:为艰苦行业培养的定向生,如为人防工程、钢铁行业、核工业、航空工业培养的定向生等

f43213efbba25e9e7f3e98deb60214fd.png

25.什么是国家专项计划?

国家专项计划是指中央部门所属高校和省(区、市)所属重点高校面向原贫困地区定向招生专项计划。在我省实施区域详见附件一。按照教育部规定,专项计划实施区域的贫困县脱贫后2023年仍可继续享受专项计划政策。

微信截图_20230422194928.png

f43213efbba25e9e7f3e98deb60214fd.png

26.什么是高校专项计划?

高校专项计划是指教育部直属高校和其他试点高校单独招收农村学生专项计划。在我省实施区域见附件二。按照教育部规定,专项计划实施区域的贫困县脱贫后2023年仍可继续享受专项计划政策。

微信截图_20230422195008.png

f43213efbba25e9e7f3e98deb60214fd.png

27.什么是地方专项计划?

地方专项计划是指我省省属骨干高校单独招收农村学生专项计划。我省实施区域为县[含县级市和自2014年起经国务院批准的县(市)改区]。

f43213efbba25e9e7f3e98deb60214fd.png

28.什么是公费师范生?

公费师范生是师范院校师范生公费教育的简称。学生在校学习期间免交学费,免交住宿费,并补助生活费。公费师范生须成绩合格,毕业后到协议确定的地方中小学任教,有编有岗。

特别提醒考生,在享受优惠政策的同时也要注意义务和责任。公费师范生须按要求签订协议,承诺毕业后从事中小学教育,服务期6年。公费师范毕业生未履行协议的,要按规定退还巳享受的公费教育费用并缴纳违约金。具体事宜请考生在报考前向录取院校或者教育行政部门咨询。

f43213efbba25e9e7f3e98deb60214fd.png

29.什么是优师专项计划?

优师专项计划属于公费师范生范畴,是为中西部欠发达地区定向培养教师的专项计划,面向全省招生,分为国家优师专项(教育部直属师范大学承担)和地方优师专项(河北师范大学和河北民族师范学院承担), 面向我省乡村振兴重点帮扶县(原集中连片特困县、国家扶贫开发工作重点县)中小学就业。考生被优师专项计划录取后须签订协议详情请咨询有关院校。

优师专项计划和公费师范生的区别:优师专项计划面向我省乡村振兴重点帮扶县中小学就业。公费师范生则由设区市有关部门及相关县(市、区)人民政府按照“择优、就近、急需”的原则落实工作岗位。

f43213efbba25e9e7f3e98deb60214fd.png

30.什么是免费医学定向计划?

免费医学定向计划是高校与基层医疗卫生机构(主要为乡镇卫生院)达成合作协议,按培养要求培养医务人员的专项计划。学生在校学习期间免除学费、住宿费,并补助生活费。若考生入学后不能正常毕业,要按规定退还已享受的减免教育费用。具体见2023年有关规定和招生计划。

学生在获取录取通知书之前,须与培养高校和当地县级卫生行政部门签署定向就业协议,承诺毕业后到有关基层医疗卫生机构服务不少于6年。具体事宜请向录取院校咨询。

f43213efbba25e9e7f3e98deb60214fd.png

31.什么是对口升学考试?

对口升学考试是中等职业学校对口升学全省统一考试的简称,是指省内普通高校对口招收中等职业学校、中等专业学校及技工学校毕业学生的考试招生形式。对口升学考试包括文化考试和专业考试两部分,采取全省统一命题、统一考试、统一评定成绩的办法。

f43213efbba25e9e7f3e98deb60214fd.png

32.对口升学考试与普通高校招生考试能否兼报?

对口升学考试与普通高校招生考试两者不能兼报。第一,试卷不同。普通高校招生使用普通高考试卷,对口升学考试使用河北省对口招生试卷。第二,文化统一考试时间重合。考生不能同时参加普通高校招生考试与对口升学考试。第三,招生对象不同。普通高校招生的对象主要是高级中等教育学校毕业或具有同等学力者,对口升学考试的对象仅限于中等职业学校、中等专业学校及技工学校毕业学生。

f43213efbba25e9e7f3e98deb60214fd.png

33.什么是高职单招?怎样报考?

高职单招是高职院校单独考试招生的简称,是高职院校分类考试招生的重要组成部分,是技术技能人才选拔的重要渠道。按照国家有关规定,高职院校分类考试招生与普通高校考试招生相对分开,更加突出职业教育人才培养规律,实 文化素质+职业技能的评价方式。今年我省部分院校继续实施高职院校单独考试招生。高职单招生源范围为参加当年我省普通高校招生报名且符合所报考院校招生条件的考生。

2023年我省高职单招继续按专业划分为不同的考试类,实行院校联合考试招生。考试类是指以教育部《职业教育专业目录( 2021年)》划分的专业大类为基础,按照相近相通原则, 分 类别进行考试和录取的一种形式。实行按考试类考试方式,一是简化了考生报考手续。考生只需选择某一考试类报考,即认为向该类所有院校报考,考试成绩供该类所有院校使用,减轻了考生的考试负担,减少了考生的舟车劳顿。二是扩大了考生志愿填报选择范围,考生填报志愿时可以面向所有院校的本类专业,增加了考生的选择范围。三是考试和录取管理更加规范,由牵头院校统筹考试及录取组织等工作,管理更加统一,操作更加规范,考试及录取环境更加优化,更加有利于维护公平公正。

已通过 2023 年我省普通高校招生报名,且符合所报考院校招生条件的考生, 在 369时至 3917时,登 录河北省高职单招服务平台(网址:http://hebgzdz. sjziei .com)进行网上报考和选择考试类。报考时,考生依据相关要求和个人意愿选择考试类,并向各考试类牵头院校交纳考试费。

2023年高职单招继续按考试类将统考计划与对口计划分开编列。统考计划专业按相通相近原则分为10个考试类, 对口单招计划专业的考试类按对口高考的专业类划分。

具体各类牵头院校如下:

牵头院校                            考试类

                  统考考试类/对口专业类

河北科技工程职业技术大学/A考试一类(土木建筑、资源环境与安全、水利等所涉及专业)/对口建筑类

河北交通职业技术学院/B考试二类(交通运输、能源动力与材料等所涉及专业)

河北机电职业技术学院/C考试三类(装备制造等所涉及专业)/对口— 机械类

 保定职业技术学院/D考试四类(农林牧渔、轻工纺织生物与化工、食品药品与粮食等所涉及专业)/对口农林类

                                                          /对口畜牧兽医类 

秦皇岛职业技术学院/E 考试五类[公共管理与服务、旅游、公安与司法(公安专业除外)等所涉及专业]/对口旅游类

河北女子职业技术学院 /F 考试六类[教育与体育(体育专业除外)、新闻传播等所涉及专业]/对口学前教育类

沧州医学高等专科学校 /G 考试七类(医药卫生(临床医学类专业除外)等所涉及专业]/对口— 医学类

河北艺术职业学院 /考试八类(文化艺术等所涉及专业)/H 考试八类艺术音乐类

                                                                                                               /U 考试八类艺术舞蹈及表演类

                                                                                                               /V 考试八类艺术一美术设计类

                                                                                                            /W 考试八类文化服务类

河北对外经贸职业学院/I考试九类(财经商贸等所涉及专业)/对口财经类

石家庄信息工程职业学院/考试十类(电子信息等所涉及专业)/对口电子电工类

                                     /对口计算机类

考生报考时注意:普通高中生可报考统考计划中的任一考试类,但不得报考对口计划。中职生可报考统考计划,也可报考对口计划,二者选择其一。如报考对口计划,只能报考本人高考报名时所选专业类相对应的考试类;如报考统考计划,可任选考试类报考,须与该类考生一起参加考试、录取。

考生须按所选考试类的考试实施方案要求获取准考证,并按规定时间参加考试。考生不得跨类考试。

高职单招实行文化素质+ 职业技能的评价方式 ,总分为750分。文化素质考试科目为语文、数学,每科150分,满分300分,职业技能考试满分 450分。

具体考试内容按考生类别分为以下两类:

报考统考计划的考生。文化素质考试成绩使用普通高中学业水平合格性考试(以下简称学考)中语文和数学科目的成绩折算替代(具体折算办法由各牵头院校确定)。没有语文和数学科目学考成绩的考生,须参加文化素质考试。职业技能考试由专业基础考试和职业适应性测试两科组成。其中,专业基础考试满分100分,职业适应性测试满分350分。 

报考对口计划的考生。文化素质考试包括语文、数学两科。职业技能考试包括专业能力测试和技术技能测试,其中专业能力测试以教育部发布的中职专业教学标准中核心专业知识为基本依据,采用笔试考试,重点考察综合专业能力;技术技能测试以教育部发布的中职专业教学标准中核心技术技能为基本依据,可采取笔试、面试、实际操作考试等方式,鼓励实际操作考试,充分体现岗位技能、通用技术等内容。

高职单招实行平行志愿,设集中志愿和一次征集志愿。考生每次填报志愿可在同一考试类中选报5所院校,每所院校最多填报6个专业。所有考生均须进行志愿填报。考生于规定时间内通过单招平台按报考时选择的考试类填报志愿, 只能填报相对应的招生计划,不得跨考试类填报。

录取结束后,招生院校公布录取考生名单,向考生发放由校长签发的录取通知书。已被高职单招录取的考生不再参加当年普通高考、对口升学等考试及录取。

经河北省教育厅批准,2023年在河北省进行高职单招的省内院校有72所,名单如下:

f43213efbba25e9e7f3e98deb60214fd.png

34.保送生有哪些条件?

具备下列条件之一的我省考生可以申报保送生:

1高级中等教育阶段获得全国中学生奥林匹克竞赛决赛一等奖并被中国科学技术协会遴选为参加国际数学、物理、化学、生物学、信息学奥林匹克竞赛国家队集训的应届高中毕业生。

2)石家庄外国语中学推荐的部分应届高中毕业生。仅面向高校外国语言文学类专业保送,2024年起,除北京外国语大学、上海外国语大学、外交学院可继续招收一定数量的外国语中学推荐保送生安排在英语语种相关专业,其他高校招收的外国语中学推荐保送生均安排在除英语以外的小语种相关专业,鼓励高校培养小语种+复合型人才。严禁高校以保送生招生形式将外国语中学推荐保送的学生录取或调整到非外语类专业。推荐限额由教育部确定。

3符合公安部、教育部印发的《关于进一步加强和改进公安英烈和因公牺牲伤残公安民警子女教育优待工作的通知》(公政治[2018]27号)规定的公安烈士、公安英模和一级至四级因公伤残公安民警的子女,可保送至公安院校学习深造。

4符合应急管理部办公厅、教育部办公厅印发的《关于国家综合性消防救援队伍英烈和因公伤残等人员子女报考中国消防救援学院优待有关事项的通知》(应急厅[2021]16号)规定的烈士、英雄模范和一级至四级因公伤残消防救援人员子女,可保送录取至中国消防救援学院学习。

5符合国家体育总局办公厅《关于做好2023年高校保送录取优秀运动员有关事宜的通知》(体科字[2022]216号)中有关高校保送录取运动员条件的考生,如:

拥护中国共产党领导,拥护社会主义制度;无犯罪记录,无严重兴奋剂违规记录。

符合2023年高考报名条件,并取得生源所在地高考报名考生号。

运动成绩优异,满足以下任一条件:

奥运项目破世界纪录或亚洲纪录或全国纪录(不含青年纪录);被授予国际级运动健将称号;足球、篮球、排球项目被授予运动健将称号;除足球、篮球、排球外的其他奥运项目、围棋、象棋、国际象棋、武术套路、武术散打项目的运动健将,应参加国家体育总局规定的赛事和小项的最高组别比赛,且取得世界体育比赛前八名,或亚洲体育比赛前六名,或全国体育比赛前三名。

6根据教育部办公厅、国家体育总局办公厅、中国科协办

公厅《关于做好普通高校保送录取有关优秀华侨和港澳台学生(运动员)的通知》(教学厅[2020]2号)规定,对符合内地(祖国大陆)高考或全国联招报名条件,且符合以下两种情况之一的华侨和港澳台学生(运动员)予以保送录取:

参加国际数学奥林匹克竞赛CIMO”“国际物理奥林匹克竞赛 IPhO”“国际化学奥林匹克竞赛IChO”“国际生物学奥林匹克竞赛 IBO”“国际信息学奥林匹克竞赛 IOI5项比赛中任意一项比赛并获奖的学生。

获得中华人民共和国全国运动会、亚洲夏季运动会、亚洲冬季运动会、夏季奥林匹克运动会、冬季奥林匹克运动会前三名的运动员。

7符合《教育部办公厅关于做好有关高校保送录取世界技能大赛获奖选手工作的通知》(教学厅[2020]3号)规定凡在世界技能组织主办的世界技能大赛(World Skills Competition)”中获奖的中国国家代表队选手且符合以下条件者,具备保送至高校深造的资格:

符合有关省(区、市)高考报名条件的中职毕业生,可保送至高校相应的高职或本科专业。

符合有关省(区、市)专升本报名条件的高职毕业生,可保送至高校相应的本科专业。

中职或高职在校生在其应届毕业当年获得保送资格。

保送录取本科专业的高校限本科层次职业学校和应用型普通本科高校。

f43213efbba25e9e7f3e98deb60214fd.png


35.什么是高校招生“阳光工程”?

为了进一步规范招生管理,增加招生工作透明度,更好地维护广大考生的合法权益, 确保招生工作的公平、公正,教育部在高校招生工作中实施阳光工程,建立和完善以十公开为主要内容的信息公开制度,即:

1招生政策公开。

2高校招生资格公开。

3高校招生章程公开。

4高校招生计划公开。

5考生资格公开。

6录取程序公开。

7录取结果公开。

8咨询及申诉渠道公开。

9重大违规事件及处理结果公开。

10录取新生复查结果公开。

f43213efbba25e9e7f3e98deb60214fd.png

36.考生档案有哪些内容和作用? 

考生档案分电子档案和纸介质档案。

考生电子档案是高校录取新生的主要依据,其主要内容包括考生报名信息、体检信息、成绩信息、诚信记录(主要指考试违规以及在招生其他环节违规的简要事实及处理结果)、志愿信息、普通高中应届考生的综合素质评价和学业水平考试成绩信息及报考军队、公安、司法等院校的政治考核(察)、面试结论信息等。

考生的高考纸介质档案为河北省2023年普通高校录取新生综合信息登记表,由考生凭本人高校录取通知书、居民身份证和准考证到当地招办或中学领取。

考生入学时应携带高考纸介质档案和中学学籍档案或人事档案等,交高校学籍管理部门。

未经有关部门同意,任何单位和个人不得对外泄露考生档案信息。

f43213efbba25e9e7f3e98deb60214fd.png

37.香港、澳门的高校如何招生?

目前,香港、澳门的21所高校在河北省招生。15所香港院校有:香港中文大学、香港城市大学、香港大学、香港理工大学、香港科技大学、香港浸会大学、岭南大学、香港教育大学、香港都会大学、香港演艺学院、香港树仁大学、珠海学院、香港恒生大学、香港高等教育科技学院、东华学院。6所澳门院校有:澳门大学、澳门科技大学、澳门理工大学、澳门旅游学院、澳门镜湖护理学院、澳门城市大学。其中澳门城市大学和澳门科技大学除招收本科生外,还招收部分预科生。21所高校的招生对象是参加当年全国普通高等学校招生统一考试的考生。

香港、澳门的高校在河北省招生分3种模式:

一是香港中文大学和香港城市大学2所高校按照内地招生体制招生,即两所高校在河北招生计划中公布招生计划,使用高考成绩,考生志愿填报在本科提前批,由高校择优录取。

二是香港大学、香港理工大学、香港科技大学、香港浸会大学、岭南大学、香港教育大学、香港都会大学、香港演艺学院、香港树仁大学、珠海学院、香港恒生大学、香港高等教育科技学院、东华学院等13所院校采用自主招生形式,招生计划不分省。报考这13所香港高校的考生须按照高校的要求提前进行报名和填报志愿,参加高校组织的考试或面试,具体报名时间以各高校公布的时间为准, 考生可直接登录这13所高校的网站获取报名等有关信息。自主招生的13所香港高校根据考生综合情况择优录取,均在7月上旬结束录取工作。考生报考香港任何一所高校均不会与在河北省招生的内地普通高校录取冲突,若考生未在河北省本科提前批录取结束前被上述两种招生形式的香港高校录取,仍可继续参加内地高校的录取,但是已被香港任何一所高校录取的考生,将不能被其他香港高校或内地高校重复录取。

三是澳门大学、澳门科技大学、澳门理工大学、澳门旅游学院、澳门镜湖护理学院、澳门城市大学招生计划不分省,均使用高考成绩,考生可根据高校的分数要求,在规定时间内登录高校网站办理报名及填报志愿手续,由澳门高校择优录取。允许澳门高校录取的考生与内地其他院校间重复录取,报考澳门高校不会影响考生报考内地高校。若同时被澳门高校和内地高校录取,考生可自行决定是在澳门高校还是在内地高校就读。

2023年参加普通高考的考生若有意报考上述院校,可登录相应高校的网站获取信息。

f43213efbba25e9e7f3e98deb60214fd.png

38.高考各阶段考生密码有哪些重置办法?

考生应牢记在高考报名时所设置的密码,并且注意保密。在后期报名信息、体检信息、高考成绩查询以及志愿填报时都须使用此密码进行登录。若考生遗忘了此密码,可通过以下途径进行密码重置,重置后的默认密码为考生身份证号后8位。

1通过手机微信重置密码。考生使用手机关注河北省教育考试院微信公众号 hbsksy),进入密码找回页面,通过人脸识别自助进行密码重置。

2通过计算机重置密码。考生使用能够访问互联网并装有摄像头的计算机,通过人脸识别自助进行密码重置。此方法需要安装人脸认证组件,下载方式及使用说明请通过志愿填报或考生信息查询系统考生登录页面右下角找回密码链接进入查看。

3到就近报名点重置密码。考生携带本人身份证到任一报名点,通过读取身份证、人脸识别完成密码重置。

f43213efbba25e9e7f3e98deb60214fd.png

39.考生如何查看自己的高考报名及体检信息?

考生可凭考生号和密码登录省教育考试院普通高校招生考试信息管理与服务平台(网址:http: // gk. hebeea. edu. cn) 进入信息查询模块,查看自己的高考报名及体检信息。

f43213efbba25e9e7f3e98deb60214fd.png

            河北省2023年普通高校

        招生工作新视点

1.强化报名资格审查。对接教育部高校学信息和全省高学籍信息, 实时比对考生学籍信息;联合公安部门对考生户籍和居住证信息逐一核查,严防“ 高考移民”;与卫健、人社、退役军人等部门建立联审机制,强化优惠加分、优先录取等优惠资格审核,严防弄虚作假。

2.严厉打击违规行为。将防范手机等高科技作弊及群体性、有组织舞弊作为重点。考前联合公安、保密、无线电管理、通信管理等部门持续开展考试环境综合治理专项活动。考生入场全部采用人脸识别技术, 严防替考。强化安检措施, 严格执行国家教育考试入场安检规范, 严防手机等作弊器材入场。所有考场、 通道、卫生间等区域实现5G信号屏蔽全覆盖。考试期间强化监考巡考,实现全程监控、全程录像。考后严格执行考场视频回放制度,严查违规违纪行为。

3.严格三个专项计划报考资格。《教育部关于做好2023年普通高校招生工作的通知》(教学[2023]J1号)规定,从2023年起,往年被国家专项计划、高校专项计划、地方专项计划录取后放弃入学资格或退学的考生,不再具有专项计划报考资格。

4.加强涉考培训咨询机构治理。根据教育部部署,各地教育行政部门会同网信、科技、公安、文化和旅游、市场监管、体育等部门,加强对社会培训机构或个人开展涉考培训咨询的规范治理,严厉打击涉及虚假宣传、价格欺诈、组织或参与考试作弊、干扰破坏考试招生秩序等违规违法行为。

5.深化艺术类专业考试招生改革。按照教育部《关于进一步加强和改进普通高等学校艺术类专业考试招生工作的指导意见》精神,我省到2024 年,基本建立以统一高考为基础 、省级专业考试为主体,依据高考文化成绩、专业考试成绩,参考学生综合素质评价,分类考试、综合评价、多元录取的高校艺术类专业考试招生制度,形成促进公平、科学选才、监督有力的艺术人才选拔评价体系。

6.改革高校高水平艺术团招生办法。自2024 年起,高校高水平艺术团不再从高校招生环节选拔,由相关高校从在校生中遴选培养。

7.深化高校高水平运动队考试招生改革。自2024年起,进一步完善和规范高校高水平运动队考试招生工作,通过优化招生项目范围, 严格报考条件和资格审核,改进考试评价方式,提高文化成绩要求,完善招生录取机制,选拔培养德智体美劳全面发展且具有较高体育竞技水平的学生,为奥运会、世界大学生运动会等重大体育比赛和国家竞技体育后备人才培养体系提供人才支撑。


重要日程备忘




































f43213efbba25e9e7f3e98deb60214fd.png


1.哪些人员需要参加高考报名?

1)参加普通高校招生全国统一考试及河北省普通高中学业水平选择性考试的人员;

2)参加对口升学全省统一考试的人员;

3)参加保送生等提前单独招生的人员;

4)报考运动训练、武术与民族传统体育专业的人员;

5)参加高职单招、残障单招、消防单招、职教师资单招、“3 + 4”对口贯通等经教育部和我省批准的普通高校单独考试招生的人员;

6)其他参加普通高校招生录取或备案的人员。

报考高校少年班的学生,待相关高校确定合格名单后,再单独组织报名。

f43213efbba25e9e7f3e98deb60214fd.png

2.哪些人员不能参加高考报名?

1)具有普通高等学历教育资格的高校在校生,或已被普通高校录取并保留入学资格的学生;

2)高级中等教育学校非应届毕业的在校生;

3)在高级中等教育阶段非应届毕业年份以弄虚作假手段报名并违规参加普通高校招生考试(包括统考、对口和高校单独组织的招生考试)的应届毕业生;

4)因违反国家教育考试规定,被给予暂停参加普通高校招生考试处理且在停考期内的人员;

5)因触犯刑法已被有关部门采取强制措施或正在服刑者, 其中,未成年人按相关法律规定执行;

6)外省在我省借读以及在我省中等职业学校等学校就读但户籍不在我省的应届生(符合报考条件的随迁子女除外);

7)以大中专招生等名义将户口迁入我省学校集体户,但没有户籍所在学校学籍和连续就读记录的人员;

8)已在外省参加2023年普通高考报名或拟在外省参加普通高考报名的人员;

9)现役军人。

f43213efbba25e9e7f3e98deb60214fd.png

3.考生在我省参加高考报名须具备什么条件?

1)普通高校招生全国统一考试及河北省普通高中学业水平选择性考试(简称统考)报名条件:

具有河北省户籍的中国公民,或教育部规定的港、澳、台地区等非我省户籍人员;

遵守中华人民共和国宪法和法律;

高级中等教育学校毕业(含应届生)或具有同等学力;

身体状况符合相关要求。

2)中等职业学校对口升学全省统一考试(简称对口升学考试)报名条件:

具有河北省户籍的中国公民,或教育部规定的港、澳、台地区等非我省户籍人员;

遵守中华人民共和国宪法和法律;

各类中等职业学校(含中等师范、职业高中及其他中等专业学校)和技工学校毕业(按照教育部学籍管理规定,高中阶段转入职教类专业学习的学生,至毕业时必须具有一年半以上职教专业学籍);

身体状况符合相关要求。

f43213efbba25e9e7f3e98deb60214fd.png

4.外省进城务工人员随迁子女在我省参加高考报名应具备哪些条件?

在我省就业的外省进城务工人员随迁子女(以下简称随迁子女)参加我省高考报名,除符合我省高考报名条件外,同时还须具备以下条件:

1在父亲或母亲或法定监护人经常居住地(居住证所在市)接受高中段教育,至毕业时具有两年(含)以上连续就学记录(不含高中毕业后复读时间)和学籍;

2父亲或母亲或法定监护人具有居住证、就业创业证(或就业创业证电子凭证或就业失业登记证)。

提醒在读的高一、高二随迁子女的家长,须按照《河北省居住证实施办法(试行)》的有关规定,提早到当地公安部门申领居住证,以免影响考生高考报名。

f43213efbba25e9e7f3e98deb60214fd.png

5.考生报考“三个专项”计划,应具备哪些条件?

1国家专项计划申报条件:

在户籍所在县(市、区)报名,并符合2023年统一 高考报名条件;

本人具有实施区域当地连续3年以上户籍,其父亲或母亲或法定监护人具有当地户籍;

本人具有户籍所在县(市、区)高中连续3年学籍并实际就读。

申报国家专项计划须同时具备以上3项条件。

2高校专项计划申报条件:

在户籍所在县(市、区)报名,并符合2023年统一高考报名条件;

本人及父亲或母亲或法定监护人户籍地在实施区域的农

村,本人具有当地连续 3年以上户籍;

本人具有户籍所在县(市、区)高中连续3年学籍并实际就读。

申报高校专项计划须同时具备以上3项条件。有关高校可在此基础上提出其他报考要求,并在招生简章中明确,提醒考生报考时注意。

( 3 )地方专项计划申报条件:

在户籍所在县(市、区)报名,并符合2023年统一高考报名条件;

本人及父亲或母亲或法定监护人的户籍在实施区域的农村,且本人具有当地连续3年以上户籍;

本人具有户籍所在县(市、区)高中连续3年学籍并实际就读。

申报地方专项计划须同时具备以上3项条件。

f43213efbba25e9e7f3e98deb60214fd.png

6.考生报考免费医学定向计划应具备哪些条件?

1)在户籍地县(市、区)报名,并符合2023年统 一高 考报名条件;

2)本人及父亲或母亲或法定监护人户籍地须在农村,本人具有当地连续3年以上户籍。

申报免费医学定向计划须同时具备以上2项条件。

f43213efbba25e9e7f3e98deb60214fd.png

7.考生报名时需要提交哪些材料?

1)考生报名时应提交户口本、本人居民身份证、高中学业水平考试准考证或高级中等教育毕业证书(含同等学力证件等有效证件、证明)、思想政治品德考核鉴定等基本材料;

2)残疾考生(含参加残障单招、优秀残疾人运动员免试入学的考生)须提供本人残疾人证,并准确填写本人残疾人证号。申请享受2023年高考合理便利的残疾考生还须按报名点要求,如实填写残疾人报考河北省普通高等学校招生全国统一考试合理便利申请表”;

3)在职职工应有单位介绍信;

4)自主就业的退役士兵应有当地退役军人事务部门出具的介绍信;

5)在外省就读的我省户籍普通高中应届毕业生,回我省参

加高考报名, 应提供学籍所在地相关省级部门出具的高中学业水平考试成绩证明、综合评价信息材料;

6)随迁子女考生须提供高中段学籍证明、毕业学校出具的连续学习记录证明、本人居民身份证、户口本、父亲或母亲或法定监护人在县级及以上的公共就业服务机构领取的就业创业证(或就业创业证电子凭证或就业失业登记证)和在公安派出所领取的有效期内居住证;

7)在我省长期居住并具备报名资格的港、澳地区居民,须提供香港或澳门居民身份证和港澳居民来往内地通行证(或中华人民共和国港澳居民居住证)。台湾地区居民须提供在台湾居住的有效身份证明和台湾居民来往大陆通行证(或中华人民共和国台湾居民居住证);

8)在华取得永久居留权的外国人,须提供在有效期内的中华人民共和国外国人永久居留身份证。

f43213efbba25e9e7f3e98deb60214fd.png

8.今年高考报名时间和地点是怎样规定的?

2023年我省普通高校招生报名工作从2022年10月 27日 9时开始,至 11月 13日 17时结束。报名结束后不再安排补报。

考生原则上在户籍所在地招生考试机构指定的报名点报名,应届考生也可以在学籍所在地报名。社会考生(包括复读生)因特殊情况确需跨市报名的,须考生就读地市级招生考试机构与户籍地市级招生考试机构协商同意,但由此引发的后果由考生自己负责。

拟报考三个专项计划、免费医学定向计划和享受少数民族加分资格的考生,须在户籍地报名。

符合报考条件的随迁子女,须在其父亲或母亲或法定监护人居住证所在市级招生考试机构指定的报名点报名。

在我省长期居住并具备报名资格的港、澳地区居民和台湾地区居民,在居住地市级招生考试机构指定的报名点报名。

在华取得永久居留权的外国人,在居住地市级招生考试机构指定的报名点报名。

f43213efbba25e9e7f3e98deb60214fd.png

9.报名工作流程是怎样的?

2023年高考报名包括资格初审、网上申报、资格审核、现场确认、交费等环节。

1)报名资格初审。考生应按照报名点安排,在规定时间内,持规定的有效证件(证明)到指定报名点进行报名资格初 审。填写“河北省普通高校招生考生报名登记表(草表)”“河北省普通高校招生考生报名有关资格申请表(草表)”等表格。报名点对通过报名资格初审的考生进行身份证基础信息(包括姓名、性别、民族、身份证号、出生年月和身份证照片等信息)采集,同时进行普通高校学籍信息核对。

2)网上申报。完成报名资格初审的考生,在规定时间内登录河北省教育考试院“普通高校招生考试信息管理与服务平台”(网址:http:// gk. hebeea. edu. cn,或由省教育考试院网站http://www.hebeea.edu.cn 右侧导航栏的“普通高考信息服务”进入)进行网上申报。

网上申报主要内容有:

①密码设置。考生首次登录须修改初始密码(为身份证号后8 位)。登录密码用于报名系统登录、报名信息查看、成绩查询、志愿填报、录取结果查询等。密码设置要有一定强度,不要过于简单,考生要牢记登录密码并妥善保管。因考生个人密码保管不善造成的一切问题,责任由考生自负。遗忘密码的考生持本人身份证到报名点进行重置。

②基本信息填报。网上申报时,考生须按要求核对身份证信息,填报户籍信息、简历及就读信息、联系方式、录取通知书邮寄地址及父亲、母亲或法定监护人的相关信息等。其中,随迁子女考生还须如实填写父亲或母亲或法定监护人身份证号、居住证编号等信息;残疾考生(含参加残障单招、优秀残疾人运动员免试入学的考生)须如实填写本人残疾人证号及残疾类型,否则会影响普通高校招生全国统一考试合理便利的申请或单独招收残疾考生高校的报考,由此产生的后果由考生自负。

③考试信息填报。考试信息包含考试类型、报考类别、外语语种等,其中考试类型分为“统考”和“对口”,统考报考类别分为普通、艺术、体育。

④相关资格申请。包含优惠加分、优先录取、三个专项计划、订单定向免费医学生计划、残疾考生高考合理便利等资格的申请。考生满足相关条件方可申请并按要求提供相关审核材料,未在网上申请或未按要求提供审核材料的视为放弃资格。

3)报名资格审核和现场确认。资格初审和网上申报完成后,考生在规定时间内持身份证等有效证件(证明)及相关材料到指定报名点进行报名资格审核并完成图像采集、信息确认等工作。考生本人须认真核对 “2023年河北省普通高校招生考生报名登记确认表”并由本人签字确认。报名登记表不得由他人代签,签字确认后信息不可变更。如因考生个人原因造成填报信息错误的,后果由考生本人承担。未进行信息确认的报名信息无效。报名结束后不再安排补报名。

4)交纳报名费。考生完成信息确认后,按要求网上支付报名费,支付成功即报名完成。未交纳报名费的考生,报名信息无效。已交纳的报名费不再退费。

f43213efbba25e9e7f3e98deb60214fd.png

10.考生报名时如何选择考试类型和报考类别?

考试类型分为“统考”和“对口”。

1)统考。参加普通高校招生全国统一考试及河北省普通高中学业水平选择性考试的考生、参加经教育部批准的高校单独招生以及其他需要通过普通高校招生录取或备案的考生均选择“统考”,如报考保送生、国家体育总局组织文化考试的体育单招、消防单招、残障单招或职教师资班等。

统考报考类别分为普通、艺术、体育。

①普通:拟报考非艺术、体育专业的考生报考类别应选普通。报考普通类的考生不能兼报艺术、体育类。

②艺术:拟报考艺术专业的考生报考类别应选艺术。未按艺术报名的考生不能参加本年度艺术类专业考试及录取。

③体育:拟报考体育专业的考生报考类别应选体育。未按体育报名的考生, 不能参加我省普通体育类专业统一测试及录取。

(2)对口。参加中等职业学校对口升学全省统一考试的考生,考试类型选择“对口”。中等职业学校和技工学校的毕业考生可以报考,其他学校[包含普通高中、高中毕业同等学力、其他中等学历教育、高职(专科)学历教育、本科(含)以上学历教育]毕业的考生不能报考。

对口报考专业类分为: 旅游类、财经类、农林类、畜牧兽医类、机械类、电子电工类、建筑类、计算机类、医学类、学前教育类。考生只能选择与高级中等教育阶段所学专业相对应的对口专业类报考。报考机械类、农林类的考生还须选择测试项目。

f43213efbba25e9e7f3e98deb60214fd.png

11.报考艺术类考生如何选择艺术类别?

报考艺术类考生须选择艺术类别,艺术类别分为声乐类、器乐类、舞蹈类、美术类和不参加艺术统考的校考类(含校际联考)。

声乐类、器乐类、舞蹈类、美术类专业考试实行全省艺术统考, 凡报考使用艺术统考成绩作为专业成绩录取的院校艺术专业或需要相应类别艺术统考合格的院校艺术专业的考生,须选报其中一类,但不允许在四类统考之间跨类报考。报考声乐类、器乐类、舞蹈类的考生还须分别选择唱法、器种、舞种等细项。

不参加艺术统考的校考类(含校际联考)为招生院校组织的统考未涉及的艺术类校考。其中校际联考为院校联合组织的戏剧与影视学类、书法学、播音与主持艺术、服装表演类、航空服务艺术与管理等5类考试。

考生选报声乐类、器乐类、舞蹈类、美术类等艺术统考类别之一后,也可参加“不参加艺术统考的校考类(含校际联考)”专业的考试和录取。但考生选报“不参加艺术统考的校考类(含校际联考)”后,不能参加艺术统考的考试和录取,也不能参加对统考成绩有要求的艺术校考专业的录取。

f43213efbba25e9e7f3e98deb60214fd.png

12.考生可以选择哪些外语语种?

普通高考外语语种有英语、俄语、日语、德语、法语和西班牙语 6种, 考生只能选择其中一种。对口招生外语语种为英语。

f43213efbba25e9e7f3e98deb60214fd.png

13.享受优惠加分政策的考生如何申请加分?

拟享受优惠加分的考生,须在规定期限内提供相应的证件、材料向相关主管部门提出申请,相关主管部门对所有拟享受高考优惠加分的考生进行资格审核。未向主管部门申报或未通过资格审核和信息公示的考生, 录取时不予优惠加分。

优惠加分项目审核部门和审核登记所需材料如下:

1)少数民族自治县(含民族县)的少数民族考生。

考生在报名时提出申请,由考生报名地县级招生考试机构负责审核登记。审核所需材料包括:①户口本(含考生本人及父亲或母亲或法定监护人户籍);②身份证;③户籍所在县学籍学校满 3年学籍证明;④户籍所在县学籍学校 3年实际就读证明等。

少数民族自治县(含民族县)享受加分的少数民族考生须在户籍所在县参加高考报名,且考生本人具有户籍所在县高中阶段连续 3 年完整户籍 、学籍并实际就读, 其 父亲或母亲或法定监护人具有当地户籍。

我省的少数民族自治县(含民族县)包括:宽城县、丰宁县、围场县、青龙县、大厂县、孟村县、滦平县、隆化县、平泉市。

( 2 )归侨、归侨子女、华侨子女和台湾省籍(含台湾户籍)考生。

考生在报名时提出申请。其中,归侨、归侨子女、华侨子女

考生,要按照侨务部门规定的时间、地点和所需材料到户籍地县级以上侨办进行资格审核。台湾省籍(含台湾户籍)考生,由当地台湾事务管理部门进行资格审核。

归侨考生审核所需材料包括:①户口本和身份证;②县级以上侨务部门出具的考生归侨身份证明;③“三侨”考生身份审查表等。

归侨子女考生审核所需材料包括:①考生户口本和身份证;

②父(母)户口本;③父 (母)单位人事部门出具的归侨身份、与考生关系的证明;④“三侨”考生身份审查表等。

华侨子女考生审核所需材料包括:①考生户口本和身份证;②父母一方护照复印件、定居证复印件;③我驻外使领馆出具的华侨身份认证;④我驻外使领馆或国内单位人事部门出具的与考 生的亲属关系证明;⑤“三侨”考生身份审查表等。

台湾省籍(含台湾户籍)考生审核所需材料包括:①台湾省籍(不含台湾户籍)考生需提供户口本、身份证;②台湾户籍考生需提供有效身份证明和台湾居民来往大陆通行证(或中华人民共和国台湾居民居住证)等。

( 3 ) 自主就业退役士兵考生 。

考生在报名时提出申请,由档案所在地退役军人事务部门负责审核登记。主要审核查验退出现役证。

( 4 ) 烈士子女考生。

考生在报名时提出申请,由户籍地退役军人事务部门负责审核登记。审核所需材料包括:①烈士证明书;②烈士与考生关系证明等。

( 5 ) 服役期间荣立二等功(含)以上考生和在服役期间被战区(原大军区)以上单位授予荣誉称号的退役军人考生。

考生在报名时提出申请,由档案所在地退役军人事务部门负责审核登记。审核所需材料包括:①退出现役证;②奖(勋)章、立功受奖证书等。

( 6 ) 具有河北户籍农村独生子女考生 。

由卫生健康部门负责审核。考生在规定时间内向户口所在地卫生健康部门提出申请,审核所需材料包括:①参加高考的证明;②户口本;③身份证;④独生子女父母光荣证;⑤农村居民户口状况证明;⑥违法生育的处理情况证明;⑦如实填写河北省独生子女审定表(一式四份);⑧考生本人近期四张小二寸免冠照片;⑨父母离婚、丧偶的,要按照卫生健康部门的要求提供相关证明材料等。

审核通过的农村独生子女考生,按照报名点规定的时间,将经县(市、区)、乡(镇、街道)两级卫生健康部门审核盖章的河北省农村独生子女审定表交报名点,由报名点汇总后,报县级招生考试机构。

2016 年 1 月 1 日(含)以后出生的农村户口独生子女考生参加高考,不再享受加分。

f43213efbba25e9e7f3e98deb60214fd.png

14.符合优先录取条件的考生如何申请?

(1)符合优先录取条件的军人子女,不在报名时申请,以相关部门提供的名单为准。资格审核工作由省军区负责。

(2)符合优先录取条件的公安烈士、公安英模、因公牺牲和一级至四级因公伤残公安民警子女,不在报名时申请,以相关部门提供的名单为准。资格审核工作由公安部门负责。

(3)符合优先录取条件的残疾人民警察、退出部队现役考生在报名资格初审时携带伤残人民警察证、退役军人事务部门出具的身份证明和退出现役证等证件材料向报名点提出申请,并在网上报名时进行申请。

( 4 ) 符合优先录取条件的消防救援队伍人员及其子女,按照相关规定执行,以应急部门提供的名单为准。资格审核由应急管理部门负责。

( 5 ) 经共青团中央青年志愿者守信联合激励系统认定,获得5A级青年志愿者考生,以相关部门提供的名单为准。

f43213efbba25e9e7f3e98deb60214fd.png

15.报考普通体育类专业有哪些具体要求?

普通体育类专业包括体育教育、社会体育指导与管理等。符合2023年河北省普通高等学校招生报名条件、年龄不超过 22 周岁的考生均可按普通体育类报名。教练员、体育教师、优秀运动员(指省级以上优秀运动队的队员)可放宽到 28 周岁。一般要求男生身高 170 cm 以上,女生 160 cm 以上;任何一眼裸视不低于4.7。

凡按普通体育类参加高考报名的考生均须参加河北省统一组织的专业考试和文化考试。未按体育类报名的考生,不能参加本年度我省普通体育类专业测试及录取。

f43213efbba25e9e7f3e98deb60214fd.png

16.我省对于考生兼报是如何规定的?

“统考”和“对口”是两种考试类型,两者不能兼报。普通高中生只能选择“统考“,不能选择“对口”;各类中等职业学校(含中等师范、职业高中及其他中等专业学校)和技工学校毕业考生既可选择“对口”也可选择“统考“,但只能选择一种类型。

艺术、体育类考生可兼报普通类专业,但每次填报志愿时(集中填报志愿或征集志愿),同一批次(段)只能选择一个类别填报。

f43213efbba25e9e7f3e98deb60214fd.png

17.报名和参加各种专业(专门)考试及文化统一考试时是否需要携带本人居民身份证?

按照省招生委员会和省公安厅有关规定,所有报名参加高考的考生在报名、参加各种专业(专门)考试及文化统一考试时,必须交验本人居民身份证。因此,考生必须在高考报名前及时办理好居民身份证。身份证遗失或无法读取的考生应及时进行补办,如因办理不及时影响高考报名和考试,后果由考生自负。高一和高二年级的学生也应尽早办理居民身份证。

f43213efbba25e9e7f3e98deb60214fd.png


18.身份证、户口簿、学考及学籍的姓名、身份证号、民族、出生年月等内容不一致的如何办理报名手续?

如果身份证、户口簿、学考及学籍的姓名、身份证号、民族、出生年月等内容不一致, 会影响到考生的考试、录取、学籍注册及毕业, 必须按管理权限由相关主管部门核准更正一致后,方可参加高考报名。

f43213efbba25e9e7f3e98deb60214fd.png

19.在外省就读回我省参加高考的考生,普通高中学生学业水平考试和综合素质评价信息如何办理?

在校生须本人持转出省省级学业水平考试主管机构出具的学业水平考试成绩证明、考生居民身份证到我省现就读学校办理学业水平考试成绩认证手续;非在校生须本人持转出省省级学业水平考试主管机构出具的学业水平考试成绩证明、考生居民身份证到户口所在地招生考试机构办理学业水平考试成绩认证手续。

学籍转回我省普通高中的,按中学学籍管理规定和程序,由转入学校登录“河北省普通高中学生综合素质评价电子平台”,采集其综合素质评价信息 。

学籍未转回我省的,到户口所在地招生考试机构指定的报名点进行报名。报名点为普通高中的,由报名点在规定时间内登录“河北省普通高中学生综合素质评价电子平台”采集其综合素质评价信息;报名点不是普通高中的,由户口所在地招生考试机构委托一所设立报名点的普通高中,登录“河北省普通高中学生综合素质评价电子平台”采集其综合素质评价信息。

转回学生的高中综合素质评价信息的采集和学业水平考试成绩转入工作,一般截至高考当年 5月31日,其中参加河北省高职单招的考生须于3月1日前转入。

f43213efbba25e9e7f3e98deb60214fd.png

20.考生号的作用是什么?

所有参加普通高校招生(含对口升学)的考生,均须通过报名获得考生号。考生号是考生参加专业考试、文化课考试、录取和进行电子备案的重要信息之 对每位考生来说是唯,用于编排考场、建立电子档案和录取备案。

f43213efbba25e9e7f3e98deb60214fd.png

21.准考证的作用是什么?

准考证是考生参加高考的时间、考试地点、考场座位的通知单,是考生参加高考时进入考点、考场的重要证件,是新生入学报到的重要依据。

考生参加考试入场时,监考人员将与各市招生考试机构打印的准考证存根进行逐一核验。

f43213efbba25e9e7f3e98deb60214fd.png

22.考生个人信息表的使用范围有哪些?

报考艺术、体育、对口、保送生、强基计划、高水平艺术团、高水平运动队、体育单招及院校单独招生等需要专业(专门)考试的考生,如高校要求考生提供个人信息表,考生可登录省教育考试院“普通高校招生考试信息管理与服务平台”(由省教育考试院网站 http://www. hebeea.edu.cn 右侧导航栏的“普通高考信息服务”进入),进入“信息查询”模块, 自行查询打印“2023年河北省普通高等学校招生考生个人信息表”,报名点不再盖章。

f43213efbba25e9e7f3e98deb60214fd.png

23.考生为什么要签订《河北省普通高校招生考试考生诚信承诺书》?

为了加强对考生的招生考试诚信教育,使每一名考生都阅读和了解《考场规则》《国家教育考试违规处理办法》《中华人民共和国教育法》《中华人民共和国刑法》及《最高人民法院 最高人民检察院关于办理组织考试作弊等刑事案件适用法律若干问题的解释》中涉考条款等法规、规章,自觉遵守考试纪律,杜绝招生考试中徇私舞弊和腐败行为的发生,教育部要求考生应签订诚信承诺书。考生在招生考试中违反诚信承诺书的内容,违规行为将会被记录到考生的诚信档案中。

承诺书.png

f43213efbba25e9e7f3e98deb60214fd.png

24.如何确定考试科目和交纳考试费?

已参加高考报名的考生,在 2023年 5月上旬,须按照规定时间登录省教育考试院“普通高校招生考试信息管理与服务平台”(网址: http://gk.hebeea.edu.cn,或通过省教育考试院网站http://www. hebeea. edu.cn 右侧导航栏的“普通高考信息服务”进入),选择统一高考文化考试(含全国统考、普通高中学业水平选择性考试)考试科目,同时交纳考试费。对口文化考试缴费同期进行。未交纳考试费的考生不得参加相应科目考试。

考生应对选考科目认真检查,一经确认不能再修改。对口专业考试、体育专业考试、艺术专业考试、高职单招、“3 + 4”对口转段等其他考试费用的收取,按有关文件执行。

f43213efbba25e9e7f3e98deb60214fd.png

25.报考选择性考试科目有什么要求?

参加普通高等学校招生全国统一考试的考生均应参加选择性考试,其中普通高中在校学生应在相应科目高中学业水平合格性考试合格的基础上报考选择性考试科目。

f43213efbba25e9e7f3e98deb60214fd.png

          河北省2023年普通高校

        招生工作新视点

1.强化报名资格审查。对接教育部高校学信息和全省高学籍信息, 实时比对考生学籍信息;联合公安部门对考生户籍和居住证信息逐一核查,严防“ 高考移民”;与卫健、人社、退役军人等部门建立联审机制,强化优惠加分、优先录取等优惠资格审核,严防弄虚作假。

2.严厉打击违规行为。将防范手机等高科技作弊及群体性、有组织舞弊作为重点。考前联合公安、保密、无线电管理、通信管理等部门持续开展考试环境综合治理专项活动。考生入场全部采用人脸识别技术, 严防替考。强化安检措施, 严格执行国家教育考试入场安检规范, 严防手机等作弊器材入场。所有考场、 通道、卫生间等区域实现5G信号屏蔽全覆盖。考试期间强化监考巡考,实现全程监控、全程录像。考后严格执行考场视频回放制度,严查违规违纪行为。

3.严格三个专项计划报考资格。《教育部关于做好2023年普通高校招生工作的通知》(教学[2023]J1号)规定,从2023年起,往年被国家专项计划、高校专项计划、地方专项计划录取后放弃入学资格或退学的考生,不再具有专项计划报考资格。

4.加强涉考培训咨询机构治理。根据教育部部署,各地教育行政部门会同网信、科技、公安、文化和旅游、市场监管、体育等部门,加强对社会培训机构或个人开展涉考培训咨询的规范治理,严厉打击涉及虚假宣传、价格欺诈、组织或参与考试作弊、干扰破坏考试招生秩序等违规违法行为。

5.深化艺术类专业考试招生改革。按照教育部《关于进一步加强和改进普通高等学校艺术类专业考试招生工作的指导意见》精神,我省到2024 年,基本建立以统一高考为基础 、省级专业考试为主体,依据高考文化成绩、专业考试成绩,参考学生综合素质评价,分类考试、综合评价、多元录取的高校艺术类专业考试招生制度,形成促进公平、科学选才、监督有力的艺术人才选拔评价体系。

6.改革高校高水平艺术团招生办法。自2024 年起,高校高水平艺术团不再从高校招生环节选拔,由相关高校从在校生中遴选培养。

7.深化高校高水平运动队考试招生改革。自2024年起,进一步完善和规范高校高水平运动队考试招生工作,通过优化招生项目范围, 严格报考条件和资格审核,改进考试评价方式,提高文化成绩要求,完善招生录取机制,选拔培养德智体美劳全面发展且具有较高体育竞技水平的学生,为奥运会、世界大学生运动会等重大体育比赛和国家竞技体育后备人才培养体系提供人才支撑。


重要日程备忘




































f43213efbba25e9e7f3e98deb60214fd.png


1.怎样进行思想政治品德考核?

思想政治品德考核主要是考核考生本人的现实表现。考生所在学校或单位应对考生的政治态度、思想品德作出全面鉴定,并对其真实性负责。无就读学校或工作单位的考生原则上由所属的乡镇、街道办事处鉴定。

报考军队院校、公安院校、司法院校、北京电子科技学院等有特殊要求院校(专业)的考生,还应进行相应的政治考核或考察。

f43213efbba25e9e7f3e98deb60214fd.png

2.哪些情况属于思想政治品德考核不合格?

有下列情形之一,且未能提供对错误的认识及改正错误的现实表现等证明材料的, 应认定为思想政治品德考核不合格。

1)有反对宪法所确定的基本原则的言行或参加邪教组织, 情节严重的;

2)触犯刑法、治安管理处罚法,受到刑事处罚或治安管理处罚且情节严重、性质恶劣,尚在处罚期内的。

f43213efbba25e9e7f3e98deb60214fd.png

3.报考军队院校的考生怎样进行政治考核?

政治考核由各县级人民武装部会同本县招生考试机构、考生户口所在地公安部门和所在中学具体组织实施。考生户籍所在地与其就读的普通中学或工作单位等不在同一区域的,政治考核工作由考生高考报名地的县级人民武装部负责,考生户籍所在地的县级人民武装部配合。

政治考核结论分为合格和不合格两种。具体事宜,以当年文件规定为准。

f43213efbba25e9e7f3e98deb60214fd.png

4.报考公安院校的考生怎样进行政治考察?

凡志愿报考公安普通高等院校的考生,由考生本人登录省教育考试院网站查询打印“2023年河北省普通高等学校招生考生个人信息表”,凭打印的个人信息表、身份证、户口本,到其户籍所在地公安派出所提出政治考察申请,并填写公安院校公安专业招生政治考察表,各县级公安机关组织考察实施人员具体进行网上核查、档案审查、走访调查等工作,在考察表上按要求填写考察意见,统一报市级公安机关,市级公安机关研究审核后,填写审核意见报送至省公安厅政治部。考察合格的考生,须从省教育考试院网站或省公安厅官方网站下载打印并签写《本人患病经历和有关情况说明》。

考察的项目和标准,参照公安机关录用人民警察的有关规定执行。考察实施机关为县级公安机关,审核机关为市级公安机关,由省公安厅政治部对考察情况进行分析,综合作出考察结论。

具体事宜,以当年文件规定为准。

f43213efbba25e9e7f3e98deb60214fd.png

5.报考司法类高校提前录取专业的考生怎样进行政治考察?

  有的司法类高校提前录取专业须单独组织政治考察,有的司法类高校涉警专业须组织政治考察、面试、体检及体能测试等, 具体安排详见高校招生章程或向有关高校咨询。

f43213efbba25e9e7f3e98deb60214fd.png

6.报考军队飞行学员的考生怎样进行政治考核?

报考军队飞行学员的考生(须全面检测合格),由军队招飞机构组织政治考核。具体时间以军队招飞机构相关安排为准。

f43213efbba25e9e7f3e98deb60214fd.png

7.报考北京电子科技学院的考生怎样进行政审?

  报考北京电子科技学院的考生,填报志愿后由保密机要部门组织政审。具体要求以学校当年公布的招生章程为准。

f43213efbba25e9e7f3e98deb60214fd.png

8.综合素质评价如何使用?

  普通高中学生综合素质评价作为高校招生录取的参考依据,投档时,我省将考生的综合素质评价情况提供给招生高校。具体使用办法由招生高校在招生章程中予以明确。

f43213efbba25e9e7f3e98deb60214fd.png

           河北省2023年普通高校

        招生工作新视点

1.强化报名资格审查。对接教育部高校学信息和全省高学籍信息, 实时比对考生学籍信息;联合公安部门对考生户籍和居住证信息逐一核查,严防“ 高考移民”;与卫健、人社、退役军人等部门建立联审机制,强化优惠加分、优先录取等优惠资格审核,严防弄虚作假。

2.严厉打击违规行为。将防范手机等高科技作弊及群体性、有组织舞弊作为重点。考前联合公安、保密、无线电管理、通信管理等部门持续开展考试环境综合治理专项活动。考生入场全部采用人脸识别技术, 严防替考。强化安检措施, 严格执行国家教育考试入场安检规范, 严防手机等作弊器材入场。所有考场、 通道、卫生间等区域实现5G信号屏蔽全覆盖。考试期间强化监考巡考,实现全程监控、全程录像。考后严格执行考场视频回放制度,严查违规违纪行为。

3.严格三个专项计划报考资格。《教育部关于做好2023年普通高校招生工作的通知》(教学[2023]J1号)规定,从2023年起,往年被国家专项计划、高校专项计划、地方专项计划录取后放弃入学资格或退学的考生,不再具有专项计划报考资格。

4.加强涉考培训咨询机构治理。根据教育部部署,各地教育行政部门会同网信、科技、公安、文化和旅游、市场监管、体育等部门,加强对社会培训机构或个人开展涉考培训咨询的规范治理,严厉打击涉及虚假宣传、价格欺诈、组织或参与考试作弊、干扰破坏考试招生秩序等违规违法行为。

5.深化艺术类专业考试招生改革。按照教育部《关于进一步加强和改进普通高等学校艺术类专业考试招生工作的指导意见》精神,我省到2024 年,基本建立以统一高考为基础 、省级专业考试为主体,依据高考文化成绩、专业考试成绩,参考学生综合素质评价,分类考试、综合评价、多元录取的高校艺术类专业考试招生制度,形成促进公平、科学选才、监督有力的艺术人才选拔评价体系。

6.改革高校高水平艺术团招生办法。自2024 年起,高校高水平艺术团不再从高校招生环节选拔,由相关高校从在校生中遴选培养。

7.深化高校高水平运动队考试招生改革。自2024年起,进一步完善和规范高校高水平运动队考试招生工作,通过优化招生项目范围, 严格报考条件和资格审核,改进考试评价方式,提高文化成绩要求,完善招生录取机制,选拔培养德智体美劳全面发展且具有较高体育竞技水平的学生,为奥运会、世界大学生运动会等重大体育比赛和国家竞技体育后备人才培养体系提供人才支撑。


重要日程备忘




































f43213efbba25e9e7f3e98deb60214fd.png


1.考生为什么要体检?

普通高等学校招生体检工作,是高等学校录取时对考生进行德智体美劳全面考核的一项重要内容。在录取阶段省级招办要给高等学校提供完整、清晰的考生体检电子档案,以便于高等学校录取时审核。因此,所有考生(包括保送生、各类单招等)均须在招生考试机构规定的时间内,到指定体检站参加体检。非招生考试机构指定的体检站为考生作出的体检结果无效, 未按招生考试机构要求参加体检,所产生的后果由考生本人负责。

f43213efbba25e9e7f3e98deb60214fd.png

2.考生体检应注意什么?

1)了解体检政策。《教育部、卫生部、中国残疾人联合会关于印发<普通高等学校招生体检工作指导意见>的通知》(教学[2003] 3 号)、《教育部办公厅、卫生部办公厅关于普通高等学校招生学生入学身体检查取消乙肝项目检测有关问题的通知》(教学厅[2010] 2 号)、《河北省招生委员会、河北省卫生计生委关于做好河北省普通高等学校招生体检工作的通知》(冀招委普[2014] 11号)和《关于转发<教育部办公厅、卫生部办公厅关于普通高等学校招生学生入学身体检查取消乙肝项目检测有关问题的通知〉的通知》(冀招委普 [2010] 9 号)等文件是体检工作主要依据,考生要认真学习。

( 2 ) 做好充分准备 。第一, 要保证较充足的睡眠。睡眠不足会导致免疫力下降,加上过度疲劳和考前的紧张,很容易患上感冒,感 冒可能导致转氨酶一过性增高, 从 而 影响体检结果。所以体检前考生要合理利用时间,保证充分休息、充足睡眠。第二,要注意饮食,不吃辛辣、油腻和不易消化的食物,不饮酒 ,不做剧烈运动等。第三,建议体检前尽量不服药物、不吃保健品。避免因药物引起转氨酶一过性增高影响了体检结果。若因疾病遵医嘱不能停药时还可以继续服用,体检时应向医生讲明。第四,体检前还要做好个人卫生,尽量穿宽松的衣服,便于体检;勿带贵重物品,以免丢失。抽血检查一般在上午进行,考生应空腹抽血,禁早餐。

3)克服紧张心理。有些考生在体检时往往因心情过于紧张而导致血压升高、心率加快、口吃、视力下降、对色觉检查图谱不能正确描述等,使体检结果不能如实反映身体状况。如体检量血压时, 有的考生缺乏常识,在精神没有平稳时急于检查,影响了血压的稳定, 出现血压升高的情况。又如, 检查眼科有两方面的内容, 即视力和色觉。在做这两方面检查时应注意:①在查视力时不要用遮盖物压迫眼球,以免影响正常结果;②在色觉检查时,医生要求受检者在指定时间内辨认色盲本内的数字或图形,因此,在受检时不要一翻开色盲本就立即回答,而应看准后再回答,但不要犹豫不决,超过规定时间才回答会被误定为色盲或色弱,从而影响某些专业的录取。

4)坚持实事求是。考生体检时,要与体检医生积极配合, 如实回答医生提出的问题,如 实 填 写 既 往 病史。不得由他人替检,不得弄虚作假。视力不好的考生, 体 检时不要戴隐形眼镜,佩戴义眼或佩戴助听器的考生体检时向体检医生讲明。高校入学体检复查时,被查出隐瞒病史或弄虚作假的将取消入学资格,请考生不要存有侥幸心理。

5)重视体检结果。经过全面的系统检查以后,考生会得到一个河北省普通高等学校招生考生体检结果通知单。通知单包含考生体检信息以及根据考生体检信息、《普通高等学校招生体检工作指导意见》得出的志愿填报指导参考,其对考生准确了解自身身体状况和填报志愿有较好的参考作用。考生的体检结果是招生院校录取的重要依据,因此考生应特别重视体检结果通知单中的相关信息。考生在拿到通知单后要认真核对,并签字确认。如果对体检信息有异议,可在规定时间内申请复检。

特别提醒考生,填报志愿时一定要认真对照《普通高等学校招生体检工作指导意见》及高校招生章程中的有关要求慎重选择专业,避免因为身体原因被高校退档而失去录取机会。

f43213efbba25e9e7f3e98deb60214fd.png

3.高考体检有哪些项目?

高考体检共分眼科、耳鼻喉科、口腔科、外科、内科、化验及X 线摄影几部分。眼科主要检查视力、色觉和眼病;耳鼻喉科主要检查听力、嗅觉、耳鼻咽喉疾病;口腔科检查唇腮、牙齿等;外科检查身高、体重、皮肤、颈部、四肢、面部、关节等;内科主要检查血压、心脏和腹部器官;化验主要检验转氨酶(A.L.T) (—般在早晨进行,要求考生空腹)等;X 线摄影主要检查胸、肺等器官。

f43213efbba25e9e7f3e98deb60214fd.png

4.高考体检是怎样进行的?

考生体检在全封闭的体检站内进行,医务人员由体检领导小组负责聘请和培训,一律佩戴标志上岗。考生由中学教师带队,凭打印的带有考生照片的体检表或体检证进入体检站参加体检。无关人员不得进入体检站。体检结束后,打印体检信息核对表交由考生确认签字。

报考军队院校、公安院校、司法类院校提前录取专业及民航飞行技术专业的考生,还须参加有关部门另行组织的体检。

f43213efbba25e9e7f3e98deb60214fd.png

5.对体检结果有异议的考生怎样申请复查?

对体检结果有异议的考生,应当场提出复检申请,或在得到本人体检结果5天内向当地招生考试机构申请复检,由各地复检医院进行复检,体检结果以复检结果为准。如考生对复检结果仍有异议, 应于接到复检结果15日内提出终检申请, 由终检医院对有异议的复检结果做出最终裁定。

f43213efbba25e9e7f3e98deb60214fd.png

6.《普通高等学校招生体检工作指导意见》的具体内容是什么?

f43213efbba25e9e7f3e98deb60214fd.png

7.报考军队院校军检是如何进行的?

军检包括面试、体格检查和心理检测,由省军区统一组织。省军区根据考生报考志愿和报考基本条件,区分院校、首选科目、男女生, 按照各 院校招生计划数的一定比例, 高分到低分的顺序,确定参加军检对象。

全省在部分市设立军检站,由所在地军分区(警备区)负责组织实施。

面试主要考察了解考生的报考动机、形象气质、逻辑思维和语言表达等方面的基本素质, 通常采取目测、口令调整和语言交流等方法进行。

体检按照《军队院校招收学员体格检查标准》(军后卫[2016]J305号)执行,由军队后方医院或军队二级甲等以上医疗机构承检。体检结论分为指挥专业合格、装甲专业合格、空降专业合格、特种作战专业合格、防化专业合格、医疗专业合格、油 料专业合格、水面舰艇专业合格、潜艇专业合格、潜水专业合格、雷达专业合格、测绘专业合格、其他专业合格、不合格等14 种。

具体事宜以当年文件为准。

f43213efbba25e9e7f3e98deb60214fd.png

8.军队院校招收学员体格检查标准具体内容是什么?


外2.png

外3.png

外4.png

外5.png

外6.png

外7.png

外8.png

外9.png

外10.png

外11.png

外12.png

f43213efbba25e9e7f3e98deb60214fd.png

9.报考公安院校面试体检和体能测评是如何安排的?

面试体检和体能测评工作由省公安厅具体组织实施,一般安排在高考志愿填报前进行。达到报考资格条件且考察合格的考生凭准考证、身份证和“公安院校公安专业本专科招生患病经历申报表”等参加面试体检(无须空腹)和体能测评。如有变化,以当年相关文件为准。

f43213efbba25e9e7f3e98deb60214fd.png

10.公安院校招生对面试体检和体能测评有哪些要求?

面试体检标准参照公安机关录用人民警察的有关规定执行,详见 《公务员录用体检通用标准(试行)》《公务员录用体检特殊标准(试行)》。同时,还应符合下列条件:

f43213efbba25e9e7f3e98deb60214fd.png

11.司法类高校提前录取专业的面试、体能测试是如何安排的?

有的司法类高校提前录取专业仍须单独组织面试、体能测试,详见高校招生章程或向有关高校咨询。

f43213efbba25e9e7f3e98deb60214fd.png

12.司法类高校提前录取专业对身体条件有哪些要求?

司法类高校提前录取专业对考生的身体条件除执行《普通高等学校招生体检工作指导意见》外,还参照教育部高校学生司、司法部法规教育司联合下发的《中国政法大学、西南政法大学、中南财经政法大学、华东政法学院、西北政法学院和中央司法警官学院提前批录取专业招生办法》(教学[2003]16号),具体要求:

五官端正、体形匀称,无各种残疾,心理健康; 裸眼视力任何一眼不低于4.7 , 无色盲、色弱;

男性身高不低于1.70米,体重不低于50千克;女性身高不低于1.60 米,体重不低45 千克;

体能测试成绩合格。

f43213efbba25e9e7f3e98deb60214fd.png




13.报考军队飞行学员体检是如何安排的?

军队招收飞行学员对身体的要求更加严格, 除参加普通体检外, 还须参加招飞机构组织的初选、复选、定选三个阶段的选拔。相关安排及要求按军队招飞机构规定执行。

初选。主要包括宣传动员、个人报名、学校推荐、身体初步检测。

复选。在招飞选拔中心设置的检测站进行,主要包括体格检查、心理选拔、文化摸底测试和政治考核。

定选。在空军招飞局检测基地进行,主要包括体格检查、心理选拔和审批录取。

招飞选拔实行单项淘汰,有一项不合格即终止检测。

f43213efbba25e9e7f3e98deb60214fd.png

14.报考民航飞行学员面试体检是如何安排的?

民航招收飞行学员对身体有特殊要求,除参加普通体检外,还须参加民航招飞单位组织的初检面试、招飞体检鉴定(含飞行职业心理学检测)。具体详询相关民航招飞单位。

f43213efbba25e9e7f3e98deb60214fd.png

15.普通高校为什么对新生进行身体复查?

为确保新生质量,进一步了解掌握学生的身体状况,便有目的、有计划地组织教学, 开展体育活动, 教育 、卫生行政部门要求,各普通高校在新生报到后应组织学生进行体检复查。

f43213efbba25e9e7f3e98deb60214fd.png

16.新旧视力对照表

16.png



17.血压原用计量单位与法定计量单位换算表


f43213efbba25e9e7f3e98deb60214fd.png

            河北省2023年普通高校

        招生工作新视点

1.强化报名资格审查。对接教育部高校学信息和全省高学籍信息, 实时比对考生学籍信息;联合公安部门对考生户籍和居住证信息逐一核查,严防“ 高考移民”;与卫健、人社、退役军人等部门建立联审机制,强化优惠加分、优先录取等优惠资格审核,严防弄虚作假。

2.严厉打击违规行为。将防范手机等高科技作弊及群体性、有组织舞弊作为重点。考前联合公安、保密、无线电管理、通信管理等部门持续开展考试环境综合治理专项活动。考生入场全部采用人脸识别技术, 严防替考。强化安检措施, 严格执行国家教育考试入场安检规范, 严防手机等作弊器材入场。所有考场、 通道、卫生间等区域实现5G信号屏蔽全覆盖。考试期间强化监考巡考,实现全程监控、全程录像。考后严格执行考场视频回放制度,严查违规违纪行为。

3.严格三个专项计划报考资格。《教育部关于做好2023年普通高校招生工作的通知》(教学[2023]J1号)规定,从2023年起,往年被国家专项计划、高校专项计划、地方专项计划录取后放弃入学资格或退学的考生,不再具有专项计划报考资格。

4.加强涉考培训咨询机构治理。根据教育部部署,各地教育行政部门会同网信、科技、公安、文化和旅游、市场监管、体育等部门,加强对社会培训机构或个人开展涉考培训咨询的规范治理,严厉打击涉及虚假宣传、价格欺诈、组织或参与考试作弊、干扰破坏考试招生秩序等违规违法行为。

5.深化艺术类专业考试招生改革。按照教育部《关于进一步加强和改进普通高等学校艺术类专业考试招生工作的指导意见》精神,我省到2024 年,基本建立以统一高考为基础 、省级专业考试为主体,依据高考文化成绩、专业考试成绩,参考学生综合素质评价,分类考试、综合评价、多元录取的高校艺术类专业考试招生制度,形成促进公平、科学选才、监督有力的艺术人才选拔评价体系。

6.改革高校高水平艺术团招生办法。自2024 年起,高校高水平艺术团不再从高校招生环节选拔,由相关高校从在校生中遴选培养。

7.深化高校高水平运动队考试招生改革。自2024年起,进一步完善和规范高校高水平运动队考试招生工作,通过优化招生项目范围, 严格报考条件和资格审核,改进考试评价方式,提高文化成绩要求,完善招生录取机制,选拔培养德智体美劳全面发展且具有较高体育竞技水平的学生,为奥运会、世界大学生运动会等重大体育比赛和国家竞技体育后备人才培养体系提供人才支撑。


重要日程备忘




































f43213efbba25e9e7f3e98deb60214fd.png


1.普通高校招生文化考试时间如何安排?

普通高校招生全国统一考试和普通高中学业水平选择性考试于6月7日至9日进行 ,详见下表 :

高考.png

f43213efbba25e9e7f3e98deb60214fd.png

2.普通高校招生文化课考试科目是如何设置的?

河北省 2023 年普通高校招生考试包括全国统一考试和普通高中学业水平选择性考试,满分750分。

统一高考科目为语文、数学、外语3门, 使用原始成绩计入考生高考文化总成绩,每门满分150分。其中,外语分英语、俄语、日语、法语、德语和西班牙语等6个语种,考生任选其中一个语种参加考试。所有语种均进行听力考试。

选择性考试科目为思想政治、历史、地理、物理、化学、生物 6 门,考生须从历史、物理 2 门首选科目中选择 1 门, 再从思想政治、地理、化学、生物 4 门再选科目中选择 2 门参加考试。其中, 历史、物理使用原始成绩计入考生高考文化总成绩每门满分100分;思想政治、地理、化学、生物按等级赋分后计入考生高考文化总成绩,每门满分100分。普通高中在校学生应在相应科目高中学业水平合格性考试合格的基础上报考选择性考试科目。

统一高考科目试题由教育部命制,选择性考试科目试题由我省自主命制。

f43213efbba25e9e7f3e98deb60214fd.png

3.思想政治、地理、化学、生物4门再选科目考试成绩

是如何进行等级赋分的?

等级赋分是按统一规则,由原始成绩进行等级划定后,再由等级转换而来的分数。转换后赋分成绩满分为100分,赋分起点为30分。先将每门再选科目考生的原始成绩从高到低划分到 A、B、C、D、E 共 5 个等级,各等级人数所占比例分别约为 15 % 、35 % 、35 %、13 % 和 2%。再将A至E等级内的考生原始成绩,依照等比例转换原则,分别对应转换到100~86、85~71、70~56、55~41 和 40 ~30五个分数区间,得到考生的赋分成绩。

高考成绩公布时,考生查询到的再选科目成绩是等级赋分后的成绩,不需要考生自行转换。

f43213efbba25e9e7f3e98deb60214fd.png

4.中等职业学校对口升学考试科目及考试时间如何安排?

中等职业学校对口升学考试分文化考试和专业考试两部分, 满分 750 分。

对口文化考试科目为语文、数学、英语(不进行听力考试), 每科试卷满分均为120分,与全国统一高考同期进行,详见下表:

微信截图_20230421224730.png

  对口专业考试(含专业理论考试和专业技能考试)按专业类别采取全省统一命题、统一考试、统一评定成绩的办法,由有关高校承办,安排在2月下旬至3月中旬进行,满分 390 分。旅游类、财经类、农林类、畜牧兽医类、机械类、电子电工类、建筑类、计算机类、医学类等 9 类, 专业理论满分 240 分, 专业技能满分 150 分。学前教育类只考教育理论, 采取笔试形式, 满分390 分。

f43213efbba25e9e7f3e98deb60214fd.png

5.考点、考场是如何安排的?

考点一般设在县级以上(含县级)人民政府所在地,按照国家教育考试标准化考点规范要求建设的中学等办学机构。考场内考生单人、单桌、单行, 每场不超过 30 人。为加强考场管理,由省以考区为单位按一定规则用计算机随机编排考场。

f43213efbba25e9e7f3e98deb60214fd.png

6.普通高考外语听力考试如何进行?

普通高考英语听力考试采用由考点统一使用有线广播系统或考场播放器进行放音的方式,其他语种采用考场播放器放音的方式。外语科目考试, 考生须于考前30分钟进入考场,考前15分钟禁止迟到考生进入考场。考前10 分钟,开始试听,试听结束时将有“试音到此结束"的提示音。考试正式开始后先进行外语听力考试。听力考试期间,考生须保持安静。听力部分结束后,会播出“听力部分到此结束”的提示音。

f43213efbba25e9e7f3e98deb60214fd.png

7.残疾考生如何申请高考合理便利?

  申请高考合理便利的残疾考生,报名时须如实填写本人残疾人证号,提供本人第二代中华人民共和国残疾人证(以下简称残疾证)、身份证,如实 填写“残疾人报考河北省普通高等学校招生全国统一考试合理便利申请表”并交当地招生考试机构。招生考试机构组织专家组进行现场综合评估:

  (一)专家组成员由本级招生考试机构、残联、卫健委等部门相关人员及班主任组成。根据残疾考生申请的类型, 专家组成员须包含相应医疗学科的副高级职称以上专家。

  (二)专家组对考生身份及残疾情况进行现场确认,结合残疾考生的残疾程度、日常学习情况、提出的合理便利申请以及考试组织条件等因素进行综合评估,并将评估意见填写在“残疾人报考河北省普通高等学校招生全国统一考试合理便利申请表”中,由专家组全体成员签字确认。

  市招生考试机构根据专家组意见提出审核意见,经省复核同意后,下发 《河北省普通高等学校招生全国统一考试残疾考生申请合理便利结果告知书》(以下简称《告知书》)。

  当地招生考试机构应及时将《告知书》送达考生或法定监护人确认、签收,如考生或法定监护人拒收则视为自动放弃,则由送达人写明情况并签字。

  残疾考生对《告知书》内容如有异议,可从收到《告知书》之日起5个工作日内,向省教育行政部门提出书面复核申请。合理便利审核结果仅作为普通高校招生统一高考和选择性考试提供相应合理便利的依据。如考生需在艺术、体育专业统考以及对口专业考试、高职单招考试中申请合理便利,须在考试前10个工作日向考试承办院校(美术统考考生须在考试前 5 日向市级招生考试机构)提出申请,按要求提供《告知书》和残疾证等有关证明材料,院校(或市级招生考试机构)在保证正常组考的前提下,为考生提供力所能及的合理便利,具体便利措施以院校(或市级招生考试机构)答复为准。

f43213efbba25e9e7f3e98deb60214fd.png

8.艺术类专业统考是如何安排的?

河北省对美术类、音乐类(分声乐类和器乐类)、舞蹈类专业实行统考。美术类专业统考由河北师范大学具体组织实施,考点设在各市;音乐类专业统考由河北经贸大学具体组织实施;舞蹈类专业统考由保定学院具体组织实施。美术类统考大纲参照《河北省普通高校招生美术类专业联考大纲》执行,音乐类、舞蹈类统考大纲分别按 2018 年公布的《河北省普通高校招生音乐类专业统考大纲及说明》《河北省普通高校招生舞蹈类专业统考大纲及说明》执行。考生须按高考报名时选择的艺术类别参加相应类别的艺术统考,不能跨类报考及录取。

(一)美术类专业统考

(1)考试科目和分值(满分为300分)。

素描(100分):考试时间180分钟。考试范围:人物头像、石膏像、静物等写生或默写。

速写(100分):考试时间30分钟。考试范围:人物(单体或组合)动态或静态写生、慢写或默写等。

色彩(100分):考试时间150分钟。考试范围:人物头像、静物写生或人物头像、静物、风景默写。

( 2 ) 考生报考以下对美术技能有专业考核要求且符合教育部艺术类专业设置规定的专业,均应参加美术类专业统考。

本科专业:戏剧影视美术设计、动画、美术学、绘画、雕塑、摄影、中国画、实验艺术、跨媒体艺术、文物保护与修复、漫画、纤维艺术、艺术设计学、视觉传达设计、环境设计、产品设计、服装与服饰设计、公共艺术、工艺美术、数字媒体艺术、陶瓷艺术设计、新媒体艺术、包装设计、环境艺术设计、美术、公共艺术设计、游戏创意设计、展示艺术设计、数字影像设计、时尚品设计、舞台艺术设计、科技艺术、美术教育、珠宝首饰设计与工艺等专业。

专科专业:艺术设计、视觉传达设计、数字媒体艺术设计、产品艺术设计、服装与服饰设计、环境艺术设计、书画艺术、公共艺术设计、游戏艺术设计、展示艺术设计、美容美体艺术、工艺美术品设计、广告艺术设计、室内艺术设计、家具艺术设计、动漫设计、人物形象设计、摄影与摄像艺术、雕刻艺术设计、皮具艺术设计、包装艺术设计、陶瓷设计与工艺、首饰设计与工艺、玉器设计与工艺、刺绣设计与工艺、雕塑设计、服装陈列与展示设计、舞台艺术设计与制作、民族美术、民族服装与饰品、民族传统技艺、影视动画、影视多媒体技术等专业。

其中摄影、动画、戏剧影视美术设计等本科专业,动漫设计、游戏艺术设计、人物形象设计、美容美体艺术、摄影与摄像艺术、舞台艺术设计与制作、影视多媒体技术和影视动画等专科专业,一般应按美术类招生。如高校招生有特殊需要,上述专业不按美术类招生的,须在招生章程中公布。

(3)考试安排。

缴费办法。

2022 年11月23日至11月26日14时,美术类专业统考考生登录河北师范大学招生信息网(网址:http://zsjyc.hebtu.edu.cn/zsw/) “专业测试”栏目查询缴费办法并缴纳美术类专业考试费。逾期未缴费考生视为自愿放弃专业考试,不能参加本年度美术类专业统考。

下载准考证。

2022年12月28日14时至2023年1月2日,美术类专业统考考生登录河北师范大学招生信息网(网址:http://zsjyc.hebtu.edu.cn/zsw / ) “专业测试”栏目下载本人准考证。

考试时间、地点。

考试时间: 2023年1月2日, 素描8 :00—11 : 00, 速写11:15—11:45, 色彩 14:00—16:30。

考试地点:高考报名所在地的市,具体地点见准考证。

考生持准考证和身份证原件,按准考证规定的时间、地点及要求参加专业考试。

河北师范大学咨询电话: 0311-80786666。

(二)音乐类专业统考(分声乐统考和器乐统考)

(1)考试科目和分值(满分为 200 分)。专项(150分,分声乐、器乐,考生根据自己报考的艺术类别和艺术统考细项参加相应的专项考试)、乐理 (15分)、音乐素质(练耳15分,视唱20分)。

( 2 ) 考生报考以下对声乐或器乐专项技能有专业考核要求且符合教育部艺术类专业设置规定的专业,均应参加相应的音乐类专业统考。

本科专业:音乐表演、音乐学、作曲与作曲技术理论、流行音乐、音乐治疗、音乐教育、录音艺术等专业。

专科专业:音乐表演、现代流行音乐、录音技术与艺术、音乐制作、钢琴伴奏、钢琴调律、作曲技术等专业。

其中录音艺术等本科专业,录音技术与艺术等专科专业,一般应按音乐类招生。如高校招生有特殊需要,上述专业不按音乐类招生的,须在招生章程中公布。

(3)考试安排。

缴费办法。

2023年1月28日至2月1日,音乐类专业统考考生凭考生号和身份证号登录河北经贸大学考试报名缴费平台(网址:https://hbyytk.hueb.edu.cn:8083) 或通过河北经贸大学财务信息网(网址:https://cw.hueb.edu.cn)登录考试报名缴费平台,核对显示的“考生号、姓名”无误后,按“二选一“要求填报专项测试信息(根据系统提示填报两首测试曲目及相关信息,待测试日候考时由计算机随机选取其中一首用于测试)并在网上缴纳音乐类专业考试费。逾期未缴费考生视为自愿放弃专业考试,不能参加本年度音乐类专业统考。

下载准考证。

2023 年 2 月 7 日至 2 月 10 日,音乐类专业统考考生登录河北经贸大学考试报名缴费平台(网址:https: //hbyytk.hueb.edu.cn:8083)下载本人准考证。

考试时间、地点。

乐理、练耳测试时间、地点:

声乐考生测试时间: 2023 年 2 月 19 日 13 : 30—14  : 00 测试乐理, 14 : 00—14: 20 测试练耳。器乐考生测试时间:2023 年 2 月19 日 16 : 30—17 : 00 测试乐理,17 : 00一17 : 20 测试练耳。测试地点:河北经贸大学(石家庄市学府路47号),具体地点见准考证。

专项、视唱测试时间、地点:

器乐考生自 2023 年 2 月 10 日起开始测试,具体测试时间见准考证。

声乐考生自 2023 年 2 月 20 日起开始测试,具体测试时间见准考证。

测试地点:河北经贸大学(石家庄市学府路 47 号)音乐楼(校医院对面)。

考生持准考证和身份证原件,按准考证规定的时间、地点及要求参加音乐类专业统考。对拒不按“二选一” 规则参加测试的考生(如测试时所报两首测试曲目经专家鉴定为同一曲目;或拒不按候考时计算机随机选取的曲目参加测试的),其专项测试成绩按 0 分处理。

河北经贸大学咨询电话: 0311-87655611。

(三)舞蹈类专业统考

(1)考试科目和分值(满分为200分)。专项(舞蹈基本功能力测试 65 分, 舞蹈技巧测试 30 分,个人作品表演 55 分)、即兴舞蹈 (30分)、节奏(20分)。

(2)考生报考以下对舞蹈专项技能有专业考核要求且符合教育部艺术类专业设置规定的专业,均应参加舞蹈类专业统考。

本科专业:舞蹈表演、舞蹈学、舞蹈编导、舞蹈教育、舞蹈表演与编导等专业。

专科专业:舞蹈表演、舞蹈编导等专业。

(3)考试安排。

缴费办法。

2022年 12 月 5 日至 12 月 9 日, 舞蹈类专业统考考生凭考生号和身份证号后 6 位登录“保定学院校园统一支付平台”(网址 :http://bdxyjfpt.bdu.edu.cn/xysf/ )缴费。登录成功后,须认真核实个人信息 , 然后点击“其他缴费”,按系统提示缴纳舞蹈类专业考试费(不能使用支付宝和信用卡支付)。逾期未缴费考生视为自愿放弃专业考试,不能参加本年度舞蹈类专业统考。

下载准考证。

2023 年 1 月 9 日至 1 月 13 日,舞蹈类专业统考考生登录保定学院招生信息网(网址:http://zsxx.bdu.edu.en) 下载本人准考证,打印准考证前须认真阅读考试须知和注意事项,须填报舞蹈专项测试中“个人作品表演"的作品名称等信息。

考试时间、地点。

考试时间:2023 年 1 月 31 日开始测试,具体测试时间见准考证。

考试地点:保定学院音乐楼(地址:保定市莲池区七一东路3027号)。

考生持准考证和身份证原件,按准考证规定的时间、地点及要求参加舞蹈类专业统考。

保定学院咨询电话: 0312-5972065。

f43213efbba25e9e7f3e98deb60214fd.png

9.考生如何查询艺术类专业统考成绩?

2023 年 1 月 20 日前确定美术类专业统考合格标准(按教育部规定,高等学校按美术类本科专业要求招生的,考生美术类专业统考的最低合格要求为 3 门科目总成绩不低于180分,且其中2门科目各不低于 60 分)并公布省统考成绩。美术类专业考生可登录河北师范大学招生信息网(网址: http://zsjyc.hebtu.edu.cn/zsw/) "专业测试”栏目查询成绩。

2023 年 3 月 5 日前确定音乐类专业统考合格标准并公布省统考成绩。音乐类专业考生可登录河北经贸大学考试报名缴费平台

(网址:https://hbyytk.hueb.edu.cn:8083) 查询成绩。

2023 年 2 月 24 日前确定舞蹈类专业统考合格标准并公布省统考成绩。舞蹈类专业考生可登录保定学院招生信息网(网址:http://zsxx.bdu.edu. cn) 查询成绩。

艺术类专业统考单科成绩四舍五入保留两位小数,总成绩为各单科成绩之和。

省统考成绩公布后,考生如对本人当次成绩有异议,须在承办院校规定时间内,按要求提出复核申请, 逾期不再受理。复核事项为:考生试卷或考试信息是否与本人信息一致,是否漏评、漏统(登),各小题得分合成后是否与提供给考生的成绩一致。成绩复核不提供考生查卷服务。

2023年我省仍不发放艺术类专业测试报考证和艺术类专业统考合格证。如需要报考证,考生可登录河北省教育考试院“普通高校招生考试信息管理与服务平台”(由河北省教育考试院网站http://www.hebeea.edu.cn右侧导航栏的“普通高考信息服务”进入)“信息查询”模块,自行查询打印“2023年河北省普通高等学校招生考生个人信息表”。

 

f43213efbba25e9e7f3e98deb60214fd.png



10.艺术校际联考是如何安排的?

2023年继续组织戏剧与影视学类、书法学、播音与主持艺术、服装表演类、航空服务艺术与管理等五类专业的校际联考工作。承办院校具体负责该类考试的命题、组考、成绩发布及复核等工作。2023年各类校际联考承办院校分别为:河北大学承办戏剧与影视学类、书法学测试(咨询电话:0312-5079698);河北地质大学承办播音与主持艺术测试(咨询电话:0311-87207400);河北科技大学承办服装表演类测试(咨询电话:0311-81668135);河北民族师范学院承办航空服务艺术与管理测试(咨询电话:0314-2370333)。各类别校际联考考试安排、成绩发布等事项详见各承办院校发布的校际联考报考公告。

河北省普通高校招生艺术类专业校际联考各类别涉及专业如下:(1) 戏剧与影视学类:戏剧学、电影学、戏剧影视文学、广播电视编导、戏剧影视导演、影视摄影与制作、影视技术、戏剧教育、音乐剧、影视编导、广播影视节目制作等专业;

(2)书法学:书法学、书画艺术等专业;

(3)播音与主持艺术:播音与主持艺术、播音与主持等专业;

(4)服装表演类:服装与服饰设计(服装设计与表演)、时尚表演与传播等专业;

(5)航空服务艺术与管理:航空服务艺术与管理等专业。

 

f43213efbba25e9e7f3e98deb60214fd.png



11.院校艺术类专业校考是如何安排的?

院校艺术类专业校考的时间、地点、测试内容、合格标准及成绩通知方式等由相应招生院校公布, 请考生登录相关院校网站查阅或直接向相关院校咨询。

f43213efbba25e9e7f3e98deb60214fd.png

12.我省对艺术类专业校考的相关规定有哪些?

(1)我省省统考涉及的专业, 高校一般应直接使用统考成绩作为考生的专业考试成绩。确有必要补充考核的艺术类本科专业,高校应面向省级统考合格生源(其中美术类本科专业须我省美术类专业统考本科合格)组织校考。省统考未涉及的艺术类专业,高校可组织校考。

(2)报考院校进行校考的艺术类专业,录取时,对于我省统考涉及的,须在相应类别统考合格(美术类本科专业须我省美术类专业统考本科合格)基础上采用院校校考成绩;对于我省统考涉及不到的,直接采用院校校考成绩。

(3)报考所有艺术类专科专业、独立学院和民办院校艺术类本科专业及省内院校的艺术类本科专业, 属统考类别涉及的,录取时必须使用河北省专业统考成绩(教育部另有规定的除外)。省外院校的艺术类本科专业, 属河北省统考类别涉及的,也有部分院校录取时使用河北省专业统考成绩。

(4)凡我省校际联考涉及的专业,省内院校必须使用校际联考成绩作为录取依据(教育部另有规定的除外),省外高校是否使用校际联考成绩,请 考生查阅相关高校艺术类招生简章。如省外高校和教育部有特殊规定的省内高校直接使用校考成绩作为专业成绩录取,无须考生校际联考成绩合格。

f43213efbba25e9e7f3e98deb60214fd.png

 


13.普通体育类专业考试有哪些要求?

(1)参加体育专业考试的考生应充分评估自己身体状况,根据自己身体条件情况参加专业考试。不建议有心脏疾病等隐患的考生参加考试。考生赴考前,应到正规医院进行心电图及脑电图等相关项目检查,并根据检查结果及医生建议决定是否赴考,以防考试时发生意外,危及身体健康。为防止考生在考试过程中发生意外伤害,根据教育部和国家体育总局有关文件精神,建议考生在参加考试前购买人身意外伤害保险(含往返交通)。对在考试期间因自身原因发生的意外伤害事故,责任由考生自负。

(2)按照教育部有关文件精神,参加河北省普通体育类专业考试的考生, 须签订“同意接受兴奋剂检查和不使用兴奋剂的承诺书”, 拒不签订承诺书或拒绝接受兴奋剂检查的考生,视为主动放弃考试资格。

(3)考生须按考前公告要求按时完成网上缴费和网上报到手续,逾期未缴费或未报到的考生视为放弃专业测试资格,不安排补缴费或补报到。不接受未办理高考报名手续或没有居民身份证的考生参加考试。

(4)考试内容:100米跑、立定三级跳远、原地推铅球、800米跑等四项。

(5)考试成绩:每项100分,满分400分。录取时,专业成绩四舍五入保留两位小数。

(6)复核申请办法:考生每完成一个项目,现场公布该项目成绩,并由考生本人签字确认。如对成绩或判罚存有异议,考生须当场提出书面复核申请,过期不再受理。

(7)相关规定:一是测试过程中如因特殊情况(极端天气等)中断测试, 考点将根据具体情况调整测试时间并公告。调整后的时间一般为正常测试全部结束后的第二天,具体时间见考点招生网,调整测试时间的考生不再缴费。二是凡因犯规、缺考等原因造成测试项目无成绩者不得再次测试该项目。三是测试开始后因自身原因无法继续测试的考生不允许再次测试该项目。四是凡已取得成绩(含单项)的考生,不允许再测试。五是测试时间一经确定,原则上不予调整。因与其他测试时间冲突或身体等客观原因无法按时测试的考生,需本人提前向考点申请,并出具有关证明,经考点同意后,通过调整测试时间方式解决。

  普通体育专业考试的具体安排可通过省教育考试院或承办高校的官网、官微查看,或向有关承办高校进行咨询

f43213efbba25e9e7f3e98deb60214fd.png

14.普通体育专业考试是怎样安排测试顺序的?

普通体育专业考试采取循环递进方式安排各市考生测试顺序。具体办法为:每2市为一组,按往年测试顺序将14个市分为7个组,逐年按组循环递进。2023年测试顺序依次为:

微信截图_20230421230546.png

f43213efbba25e9e7f3e98deb60214fd.png

15.河北体育学院少数民族传统体育项目测试是怎样安排的?

河北体育学院少数民族传统体育项目测试采取单独测试的办法进行,具体要求见河北体育学院招生信息网《河北体育学院2023年社会体育指导与管理专业(少数民族传统体育项目)招生简章》。

f43213efbba25e9e7f3e98deb60214fd.png

16.对口专业考试是如何安排的?

对口专业考试分专业理论和专业技能考试, 安排在2月下旬至3月中旬进行。各专业类具体考试安排如下:

(一)建筑类、财经类

1.缴费时间及办法

考生须于 2023 年 1 月 1 日至 1 月 8 日通过微信缴费,具体缴费方式如下:在微信中向下滑动,在“小程序界面中搜索“农行微缴费”,左上角位置图标选择“邢台市,在“更多”窗口中选择“民生服务”中的“考试报名费“ ,然后选择“ 河北科技工程职业技术大学考试费,输入缴费号和姓名(或者名称),即可缴费。标准为建筑类每生200元,财经类每生130元,个人缴费号为 14 位考生号,缴费咨询电话:0319-2272042。请考生务必注意:逾期未缴费考生视为自动放弃专业考试;缴费后没有参加专业考试的,不退费。

如考生所在学校为考生统一办理缴费手续,请于2023年1月3日前将包含 有学校名称、缴费总额对应的所有考生信息(专业考试类别、考生号、姓名、缴费全额)的EXCEL表格发送至709402515@ qq.com。邮件的主题为 “ 2023 年对口专业考试XX学校缴费信息,并请务必在邮件中注明缴费学校名称、缴费总人数、缴费总全额、联系电话(固话和手机)、联系人 等,集体缴费号于2023 年 1 月 5 日在 河北科技工程职业技术大学招生信息网(网址 :http://zs.xpc.edu.cn/ ) 公布,如有问题请拨打电话(0319-2271766)或通过 QQ( 709402515)联系,缴费方法及时间与个人缴费一致。

2.考试时间安排

(1)专业理论考试时间

建筑类:2023 年 3 月 11 日上午。

财经类:2023 年 3 月 12 日上午。 

(2)专业技能考试时间

建筑类:2023 年 3 月 11 日下午、3 月 12 日至 3 月 13 日全天(如有特殊情况技能考试时间顺延)。

财经类:2023 年 3 月 12 日下午。

(3) 准考证打印时间

所有已缴费考生于 2023 年 3 月 6 日至 3 月 11 日登录河北科技工程职业技术大学招生信息网(网址:http://zs.xpc.edu.cn/) ,点击“准考证打印”栏目,打印准考证、考生须知 。

3.考试地点

(1)建筑类

所有考生在河北科技工程职业技术大学参加考试。

(2)财经类

所有已缴费考生于 2023 年 3 月 1 日登录河北科技工程职业技术大学招生信息网(网址:http://zs.xpc.edu.cn/) 查看。

4.联系电话

建筑类、财经类:0319-2273053、2273675、2271766。

5.成绩查询

考生于 2023 年 3 月 29 日至 3 月 31 日登 录河北科技工程职业技术大学招生信息网(网址:http://zs.xpc.edu.cn/)查询考试成绩。如考生对成绩有异议, 请于2023 年 4 月 1 日前向河北科技工程职业技术大学招生办公室提出成绩复核申请。

(二)医学类

1.缴费时间及办法

考生须于2023年1月1日至1月8日通过微信缴费,具体缴费方式请使用微 信扫描下方二维码:

微信截图_20230421231043.png

在客户编号处输入14位考生号,确认个人信息后点击“确定”,然后勾选账单完成缴费。在页面下方可点击“缴费情况”查询缴费情况。缴费标 准为每生150 元。缴费咨询电话:18931316891。请考生务必注意:逾期未缴费考生视为自动放弃专业考试;缴费后没有参加专业考试的,不退费。

中职学校如需批量查询考生缴费情况,请登录河北北方学院招生网(网址: https://zs.hebeinu.edu.cn/) ,点击“对口医学考试缴费查询”按钮,将考生号整体贴入后点击“查询”按钮,即可显示未缴费考生考生号。缴费数据从1月1日至1月8日期间每日18点更新一次,请留意页面数据更新时间提示。

2.考试时间安排

(1)专业理论考试时间

2023年3月12日上午。

(2)专业技能考试时间

2023年3月12日下午、3月13日至3月14日全天(如有特殊情况技能考试时间顺延)。

(3)准考证打印时间

所有已缴费考生于2023年3月7日至3月11日登录河北北方学院招生网 (网址:https://zs.hebeinu.edu .cn/) ,点击“对口医学类准考证打印”按钮打印准考证、考生须知。

3.考试地点

考试地点在张家口市,具体考点信息请于2023年3月1日登录河北北方学院 招生网(网址:https://zs.hebeinu.edu.cn/) 查询。

4.联系电话

0313-4029183

5.成绩查询

考生于2023年3月27日至3月29日登录河北北方学院招生网(网址:https: //zs.hebeinu.edu.cn/),点击“对口医学成绩查询”按钮查询考试成绩。 如考生对成绩有异议,请于2023年3月30日前向河北北方学院招生办公室提出成绩复核申请。

(三)机械类、电子电工类、旅游类

1.缴费时间及办法

考生须于2023年1月1日至1月5日凭缴费编号(个人14位考生号)和身份证号通过中国建设银行缴费平台,网上缴纳专业考试费:标准为机械类、电子电工类、旅游类每生280元。请考生务必注意:逾期未缴费考生视为自动放弃专业考试;缴费后没有参加专业考试的,不退费。特别提醒:本年度缴费不受理集体缴费。

缴费方式如下:

笫一步,通过以下方法打开中国建设银行缴费平台。

方法一:通过中国建设银行悦生活登录缴费平台(网址:http://life.ccb.com/cn/paymentv3/bill_item/2012072820341820275793.html),选择“河北省”—“石家庄市”—“河北师范大学(招生考试)”。

方法二:扫描二维码打开缴费平台。

微信截图_20230421231441.png

方法三:关注微信公众号“中国建设银行(微信号CCB _elutong),点击“悦生活”—“生活缴费”—“教育服务”—“考试报名费,所在城市选择“石家庄市,缴费单位选择“河北师范大学(招生考试)点击“下一步”,打开缴费平台。

方法四:关注微信公众号“河北师大招生(微信号 hbsd­zs) ,点击右下角专业测试”—“考试缴费入口”,打开缴费平台。

笫二步,输入和确认个人信息,并支付费用。

(1)在对应位置输入缴费编号(个人14位考生号)、身份证号,点击“下一步”。

(2)点击“未缴费”,核对订单显示的信息:对口专业测试,应缴金额(机械类、电子电工类、旅游类每生280元)。

(3)点击订单最后的“详情(或V),核对“姓名、对口专业类无误后, 再点击“去支付(或缴费)” 即可通过“网上银行支付、账号支付、扫码支付”缴费,“网上银行支付、账号支付”可以使用考生本人或他人的建行卡账号;“扫码支付可以使用建行龙支付或者微信。点击“支付”后,如果缴费或支付不成功, 请20分钟后再次登录缴费平台。

(4)支付成功后的考生,可以点击“已缴费”查询缴费结果。

2.考试时间安排

(1)机械类:2023年2月18日至2月23日。

(2)电子电工类:2023年2月18日至2月23日。

(3)旅游类:2023年2月18日至2月23日。

(4)准考证打印:机械类、电子电工类、旅游类考生于 2023 年 2 月 11 日以后登录河北师范大学招生信息网(网址:http:// zsjyc.hebtu.edu.cn/zsw/)的“专业测试”栏目下载打印准考证

(机械类、电子电工类、旅游类具体考试时间详见准考证)。

3.考试地点

河北师范大学(石家庄市南二环东路20号)。

4.联系电话

(1)机械类、电子电工类:0311-80787908

(2)旅游类:0311-80786104

(3)河北师范大学招生办公室:0311-80786666

5.考生参加专业技能考试自备工具

(1)机械类专业:着工装、工帽、工作鞋,并自备工量刃具(见附表)。

(2)电子电工类专业:万用表、电烙铁及常用焊接组装工具、电工工具各一套;穿工作服和绝缘鞋。

(3)旅游类专业:着工装(不带学校标志)和平底鞋。 

微信截图_20230422194026.png

微信截图_20230422194041.png微信截图_20230422194104.png

6.成绩查询

机械类、电子电工类和旅游类考生请于2023年3月22日以后登录河北师范大学招生信息网(网址:http://zsjyc.hebtu.edu.cn /zsw/)的“专业测试”栏目查询专业考试成绩,如考生对成绩有异议,请于3月23日前向河北师范大学招生办公室提出成绩复核申请。成绩复核联系电话:0311-80786666 (兼传真)。

(四)畜牧兽医类、农林类、计算机类

1.缴费时间及办法

考生须于2023年1月3日至1月8日通过微信扫描“河北科技师范学院 对口专业考试智能缴费系统”二维码办理缴费手续,标准为计算机类、农林类每生140元,畜牧兽医类每生150元,缴费咨询电话:0335-8076333。请考生务必注意:逾期未缴费考生视为自动放弃专业考试;缴费后没有参加专业考试的,不退费。学校不受理集体缴费,职中如需要已缴费学生名单, 可在本校缴费结束后至2023年1月9日前与河北科技师范学院QQ(2823106274) 联系。

缴费流程如下:

(1) 用微信“ 扫一扫”功能,扫描“河北科技师范学院对口专业考试智能缴费系统”二维码,进入缴费登录界面,输入考生号, 点击“确定”。

微信截图_20230422194244.png

(2)进入个人缴费信息界面,核对本人信息无误后,点击“去缴费”。

(3)进入缴费明细界面,核对本人缴费明细信息,如缴费信息无误,勾选 “我已阅读并同意《缴费系统服务说明》”,点击“立即缴费”。

(4)输入个人微信支付密码,进行缴费即可。

2.考试时间安排

(1)专业理论考试时间

畜牧兽医类:2023年3月4日上午。

农林类: 2023年3月4日下午。

计算机类:2023年3月18日上午。

(2)专业技能考试时间

畜牧兽医类:2023年3月4日下午开始。

农林类:2023年3月5日上午开始。

计算机类:2023年3月18日下午开始。

具体考试时间安排请于2023年2月6日后登录河北科技师范学院本科招生网“通知公告”栏查询。

(3)准考证打印时间

考生于每类考试开始前5日登录河北科技师范学院本科招生网“通知公告”栏下载打印准考证(具体考试时间和地点详见准考证)。

3.考试地点安排

畜牧兽医类、农林类、计算机类具体考试地点安排请于2023年2月6日后登 录河北科技师范学院本科招生网“通知公告”栏查询。

4.成绩查询

考生于2023年4月4日后登录河北科技师范学院本科招生网“录取查询/对口成绩查询“模块查询考试成绩。如考生对成绩有异议,请于4月6日 17:00前向河北科技师范学院招生办公室提出成绩复核申请。

5.如遇特殊原因考试时间等发生变化,将通过河北科技师范学院本科招生网站进行通知,请随时关注。

6.联系电话0335-8076333;河北科技师范学院本科招生网网址http://zhaosheng.hevttc.edu.cn/。

(五)学前教育类

1.缴费时间及办法

考生须于1月3日至1月9日凭考生姓名和身份证号通过“银联校园安心付缴费平台“网上缴费,标准为每生200元。请考生务必注意:逾期未缴费考生视为自动放弃专业考试;缴费后没有参加专业考试的,不退费。缴费方式如下:

(1)个人缴费方式

手机安心付客户端 缴费流程。

第一步:下载校园安心付手机APP ,成功下载后点击进入登录界面,选择 “登录/注册”按钮,点击后会弹出一个填写“手机号码”的界面。

笫二步:按照系统指示完成账号注册和登录密码设置,账号注册登录成功后即可进入校园安心付主界面。

笫三步:成功登录绑定考生信息后,点击主页面右上的缴费”选项,进入缴费项目界面,可清楚看到个人信息和缴费明细,确认后点击“去支付”。

笫四步:进入核对订单界面,请再次核实确认订单信息,无误后选择支付方式:银联在线支付,完成缴费。

笫五步:缴费成功后,可通过已缴费栏目进行查询,或进入个人中心,查看本机的交易记录。

如遇问题,请联系校园安心付客服热线:400-028-1024。

[服务时间:9:00—20:00 (平时);9:00—18:00 (周六、周日)]。

考生可扫描二维码下载校园安心付手机客户端。

考生也可关注“石家庄幼儿师范高等专科学校”公众号进行缴费,方法如下:

微信截图_20230422194431.png

笫一步:关注“石家庄幼儿师范高等专科学校”公众号。

笫二步:点击导航栏中“校园服务”栏目找到“缴费操作指引”,点开并认真阅读。

笫三步:按“缴费操作指引”的说明,选择网页缴费或微信小程序缴费。

(2)集体缴费方式及账号

如果考生所在学校为考生统一办理缴费手续,2023年1月3日至1月5日将 要缴费的所有考生信息(专业考试类别、身份证号、考生号、姓名、缴费金额、中职学校名称、联系电话)的EXCEL表格发送邮件至sjzyzcwgg@126.com (如果考生是个人缴费,则不需要发送电子邮件)。邮件的主题为“2023年学前教育类对口专业考试**学校缴费信息”,并请务必在邮件中注明缴费学校名称、缴费总人数、缴费总金额、联系人、联系电话(固话和手机)。

单位名称:石家庄幼儿师范高等专科学校

开户行名称:建设银行石家庄华能支行

开户行账号:13050161903208666666 

开户行行号:105121061148

银行转账汇款时,请在备注栏写明缴费单位名称、缴费人数。

2.考试时间安排

(1) 2023年3月11日上午。

(2)考生于2023年3月6日以后登录石家庄幼儿师范高等专科学校网站(网址:http://www.sjzysgz.com/)查看“ 2023年河北省学前教育类对口专业考试准考证打印”,打印准考证(具体考试时间、地点详见准考证)。

3.乘车路线

(1)石家庄火车站北站乘 301 路幼师学校站下车即到。

(2)石家庄火车站乘 320 路至横山站转乘 301 路幼师学校站下车即到。

(3)地铁3号线转1号线至西王站转乘游 337 路幼师学校站下车即到。

4.联系电话

0311-83932501。

5.成绩查询

考生于2023年3月22日后登录石家庄幼儿师范高等专科学校网站(网址:http://www.sjzysgz.com/)查询考试成绩,如考生对成绩有异议,请于 2023年3月23日上午11时前向石家庄幼儿师范高等专科学校提出成绩复核申请。

考生注意事项:

(1)考生凭准考证、身份证及承办高校要求的疫情防控所需材料,按规定时间和地点参加考试。参加专业理论考试时,请自带黑色签宇笔、2B铅笔、橡皮、小刀、尺子等文具。其中,财经类考生参加专业技能考试还需自备红色、黑色签字笔,空白名章,印台;建筑类考生还需自带三角板。2023 年医学类考点设置有调整,请考生及时登录河北北方学院网站查看。

(2)考生要遵守考场规则。不得携带规定以外的物品参加考试,按要求接受安全检查。考生参加专业技能考试时,不允许穿校服或有所在学校标识的服装,不得佩戴校徽。自觉服从监考员等考试工作人员管理,不得扰乱考场及其他考试工作地点的秩序。如不遵守考场规则,有违规行为的,严格按照《中华人民共和国教育法》以及《国家教育考试违规处理办法》(教育部令第33号)执行,涉嫌违法的,将移送司法机关,依照《中华人民共和国刑法》等追究法律责任。

(3)经审核通过的残疾考生, 如需在对口专业考试期间享受合理便利, 须在考试前10个工作日向承办高校提出申请,同时提供《河北省普通高等学校招生全国统一考试残疾考生申请合理便利结果告知书》和残疾人证等有关证明材料。高校在保证正常组考的前提下,为考生提供力所能及的合理便利,具体便利措施以高校答复为准 。

(4)考生按公布的时间及方式查询专业考试成绩,对考试成绩有异议的, 在承办高校规定时间内提出成绩复核申请。

(5) 疫情防控须知。考生要按照国家和我省疫情防控有关要求,严格遵守考点院校疫情防控措施,做好个人防护。承办高校将根据疫情防控形势在本校网站公 布疫情防控有关要求,考生考前要及时登录查看并严格 遵守, 以免影响考试。

(6)有关专业考试未尽事宜,考生可直接与承办高校联系。

f43213efbba25e9e7f3e98deb60214fd.png


17.考生如何参加外语口试?

拟报考外语专业和有外语口试要求专业的考生,应参加全省统一组织的外语口试。其中俄语、日语、法语、德语、西班牙语安排在6月14日进行,俄语口试由河北师范大学负责,日语口试由河北大学负责,法语、德语、西班牙语口试由河北外国语学院负责。英语口试安排在6月14日至15日进行,由各市招生考试机构负责。考生要按规定的时间、地点,携带准考证和居民身份证参加口试,逾期不再补试。口试所需费用由考生自理。

口试前,给考生一定的准备时间阅读口试试题。一般由两名教师对考生进行口试或采用人机对话方式。首先是通过朗读外语单词、词组、句子以及短文,考查考生语音、语调以及朗读技巧。其次是通过用外语回答问题, 来考核考生的思维能力、听力、口头表达和模仿能力。

口试成绩满分为 150 分,不计入考生高考文化总成绩,仅在录取时供对外语口试成绩有要求的专业(类)参考使用。

f43213efbba25e9e7f3e98deb60214fd.png

  18.《考场规则》的主要内容有哪些?考生应遵守如下考场规则

(一)自觉服从监考员等考试工作人员管理,不得以任何理由妨碍监考员等考试工作人员履行职责,不得扰乱考场及其他考试工作地点的秩序, 不得危害他人的身体健康和生命安全,不得违反考试期间的考试防疫要求。

(二)凭准考证、身份证等材料,按规定时间和地点参加考试。应主动接受监考人员和考试工作人员按规定进行的身份验证、身体健康监测和对随身物品等进行的必要检查。因装有心脏起搏器等不宜进行金属探测器检查的考生,应在检查前声明,并出示县级以上医院出具的证明,如不提前声明,后果自负。

(三)除 2B 铅笔、黑色字迹的钢笔或签字笔、直尺、圆规、三角板、无封套橡皮、透明文具袋、无字垫板等规定的考试用品外,其他任何物品(如手机等通讯工具、手表等计时设备)不得带入考场。

(四)考生入场后,对号入座,将准考证和身份证放在桌面左上角以便核验,要认真核对考试科目、考场号、座位号是否正确。领到答题卡和试卷后,应在指定位置和规定时间内准确、清楚地填写姓名、考生号、考场号、座位号等。凡漏填、错填或书写字迹不清的答卷,影响评卷结果的,责任由考生自负。

遇试卷、答题卡分发错误及试题字迹不清、重印、漏印或缺页等问题,应举手询问,在开考前报告监考人员;开考后,再行报告、更换的,延误的考试时间不予延长;涉及试题内容的疑问,不得向监考人员询问。听力考试期间,不得向监考人员询问并保持安静。

(五)开考信号发出后方可开始答题。

(六)开考前30分钟(第一科为考前40分钟)考生开始入场,开考15分钟后迟到考生不得进入考点(有听力考试的外语科14:45以后迟到考生不准进入考场)参加当次科目考试。考试结束后方可交卷出场。考试时间内非经监考人员允许不得离场。

(七)在与题号相对应的答题区域内答题,写在草稿纸上或非题号对应的答题区域的答案一律无效。不得用规定以外的笔和纸答题,不得在答卷上做任何标记。

(八)在考场内须保持安静,不得吸烟,不得喧哗,不得交头接耳、左顾右盼、打手势、做暗号,不得夹带、旁窥、抄袭或有意让他人抄袭,不得传抄答案或交换试卷、答卷、草稿纸,不得传递文具、物品等,不得将试卷、答卷或草稿纸带出考场。如身体出现异常情况,应立即报告考试工作人员。

(九)考试结束信号发出后,立即停笔并停止答题。在监考人员依序收齐答卷、试卷、草稿纸后,根据监考人员指令依次退出考场和考试区域。再选科目考试当天,需要继续参加本半天下一场考试的,须到规定的考场和座位参加考试;不需要参加考试的,要及时离开考点。

(十)如不遵守考场规则,不服从考试工作人员管理,有违规行为的,按 照《中华人民共和国教育法》《国家教育考试违规处理办法》处理,并将记入国家教育考试诚信档案;涉嫌犯罪的,移送司法机关追究法律责任。

f43213efbba25e9e7f3e98deb60214fd.png

19.考生如何配合考试工作人员进行安全检查?

为确保高考安全平稳进行,我省使用金属探测器对进入考场的考生进行安全检查。严防考生携带易燃易爆物品、管制刀具、枪支、手机等无线收发装置及其他考试规定以外的物品进入考场。

考生参加考试时,尽量不要穿戴带有金属的饰品和衣物,入场时须主动将违禁物品(如手机等通讯工具、手表等计时设备)放到指定位置。接受检查时应双臂张开,配合监考员进行检查。身体正面至背面,从上至下,包 括头部、躯干、四肢(含脚部)等,以及考生的透明笔袋,均要进行检查。佩戴眼镜的考生要取下眼镜接受检查。检查脚部时,要抬起脚,离地 20 厘米左右。如在检查过程中出现报警声,考生应按监考员要求进行解释、出示相关金属物品。检查完毕确定无违禁物品后,考生方能入场。因身体原因,不能进行检查的考生(如装有心脏起搏器等),应说明情况,并出具县级以上医院的证明。

入场后因特殊原因离开考场后返回时,仍要对考生再次检查。对以各种形式阻挠或拒绝接受检查的考生,一律不准进入考场;对各类扰乱考场秩序、危害考试安全的人员,除依照教育部《国家教育考试违规处理办法》予以处理外,情节严重的,公安机关还要依照《中华人民共和国治安管理处罚法》予以处罚;构成犯罪的,依法追究刑事责任。

f43213efbba25e9e7f3e98deb60214fd.png

20.考生考前需做好哪些心理准备?

每位考生都希望考出自己的实力、考出好的成绩,相信这也是考生家长的期望。要实现这一目标,除扎实的功底和健康的身体外,充分做好心理准备必不可少。

(1)适量练习,保持活力。好多同学都有这样的感觉,几天不做数学题后再考试,审题迟疑缓慢,入手不顺,运算不畅且易出错。所以必须坚持每天做适量的练习,特别是重点和热点题型,保持思维的灵活和思路畅通。做题时要限时完成,否则容易养成做题拖拉的毛病,临场时缺少思维激情, 控制不好时间,发挥不出应有水平。

(2)合理饮食起居,调节身心。每年高考临近,都有一些考生精神特别紧张,甚至病倒。我们提醒大家,要注意防止两个极端的做法:一是彻底放松,破坏了长期形成的生物钟,会适得其反;二是挑灯夜战,加班加点, 导致考前极度疲劳,临场提不起精神。我们建议考生,休息调整是必要的,但是必须微调,特别要把兴奋状态逐步调整到高考考试时间,这当然离不开家长与考生的共同努力。高考前还要注意饮食的科学性和规律性,不能 大吃大喝。考生考前的饮食宜清淡又要保证全面的营养,每天要摄入足够的淀粉食物,保证用脑的需要。总之,生活要有节奏,张弛有度,保持心态平稳。

(3)充满自信,沉着应战。高考不仅是知识的比赛和智力的竞争,也是思维品质的考查和心理素质的较量。考试前保持必胜的信念是十分重要的,走进考场要信心百倍,即使遇到难题也不要慌张,因为大家拿到的考题是一样的。另外,进入考场后适度的紧张是正常的也是必要的,因为它有利于激情的产生,千万不要因此而引起不必要的慌张。只要大家精心准备、充满自信、沉着应战,就一定能取得满意成绩,迎来美好前程!

f43213efbba25e9e7f3e98deb60214fd.png

21.考生考前应注意哪些主要事项?

(1)考生应于 6 月 6 日 16 : 00 至 17 : 00 到考点熟悉环境对考点的各类设施做到心中有数,包括考场、候考区、考点办公室、医务室、饮水处、自行车放置处、厕所、应急疏散通道及场所等所在位置都要清楚。特别是要熟悉并牢记 6 月 9 日再选科目的考试时间、考场号、座位号,避免记错时间,进错考场,坐错座位。

(2)距离考点较远的考生要提前联系安排好食宿。

(3)准考证、身份证等进入考场必需的有效证件、材料,必须妥善保存。

(4)准备好考试用具,如0.5毫米的黑色字迹签字笔、直尺、圆规、三角板、2B铅笔、橡皮等。

(5)确认未携带违禁物品。

f43213efbba25e9e7f3e98deb60214fd.png

22.考生遇有特殊情况应如何处理?

(1) 因交通原因不能按时到场。考生在考前应到考点实地看一看,由此计算时间和路线。出发时要留有余地,以防迟到。万一发生交通堵塞或交通事故,考生可以出示准考证,求得公安交警的帮助。

(2)丢失或忘带准考证、身份证。每科考试,监考老师都会按规定查验准考证、身份证。考生万一丢失或忘带,在入场时向考点报告,由监考老师会同考点主考核对准考证存根、进行人脸识别等,确定无误后可以先参加考试。准考证到当地招办办理补发,身份证到公安派出所办理,在下一场考试前,要交验准考证、身份证。有带队老师的,考生可请带队老师帮助。

(3)考场上突然身体不适,如呕吐、腹泻等。出现这种现象多半是考生精神过于紧张、饮食不洁或休息不好所致。一旦发生这种情况,应及时向监考老师示意,监考老师会及时与考点办公室联系,由考点医务人员进行治疗。若考生中途需离开考场,须工作人员陪同,治疗后可继续应考,但耽误时间不补。

(4)试卷漏印或字迹不清。考生如遇试卷漏印或字迹不清应及时向监考老师声明,申请更换。

遇到突发性事件,首先要保持镇静,切忌过度紧张,手忙脚乱。要进行有效的自我心理调适,通过积极的心理暗示或做深呼吸等方法消除紧张情绪,尽快将精力集中到所答的题目上来。

f43213efbba25e9e7f3e98deb60214fd.png

23.答题过程中考生需要注意哪些事项?

开考信号发出后,方可答题。答题时要将答案填涂在答题卡规定的地方。

在考场内须保持安静,不准吸烟,不准喧哗,考试中不准交头接耳、左顾右盼、打手势、做暗号,不准夹带、偷看、抄袭或有意让他人抄袭,不准传抄答案或交换试卷、答题卡。如遇到试卷字迹不清、污染等问题可举手询问。开考 15 分钟后迟到考生不准再进入考点,其中有听力考试的外语科开考前 15 分钟后禁止迟到考生进入考场。考试结束前不得离开考场。在整个答题过程中,尽自己的能力答好题,不要分散自己的注意力。

应严格遵守考场纪律,不要存有一丝一毫的侥幸心理,以免因小失大,追 悔莫及。如有其他考生违规干扰自己答题时, 应及时向监考老师反映, 以免出现雷同试卷时被处罚。

考试结束时间到,考生应立即停笔,并将自己的试卷按答题卡、试卷、草稿纸的顺序整理好后、两臂下垂将手置于桌下待收。试卷收齐后,考生应依次走出考场在考点内等候,待试卷全部清点无误后,由考点主考通知门卫开门方可外出。

f43213efbba25e9e7f3e98deb60214fd.png

24.针对网上评卷,考生答题时要注意哪些问题?

(1)考生要仔细阅读答题卡上的注意事项,并 按 注意事项上的规定认真执行。

(2)认真填写考生信息(姓名、考生号等),字迹要清晰工整。

(3)在指定位置粘贴条码,条码一律横贴,即条码上的考生号和姓名为从左向右的方向;条码必须贴在指定的红色条码框中。要保持条码的整洁和完整,不要在条码上面或条码周围乱写乱画。

(4)不要填写缺考标记,缺考考生的缺考标记由监考老师填写。

(5)注意书写用笔,选择题必须使用正规厂家生产的 2B 铅笔填涂(使用劣质铅笔填涂可能会影响计算机扫描识别),涂写要均匀、饱满、轻重适度,修改时务必要用橡皮擦干净;非选择题最好使用0. 5毫米的黑色字迹签字笔书写,以确保字迹清晰。

(6)作图题直接用0.5毫米的黑色字迹签字笔绘制,也可先用铅笔绘制,确认后再用0.5毫米黑色字迹签字笔描清楚。

(7)严禁在答题卡上的图像定位点(黑方块)周围作任何涂写和标记。

(8)答非选择题时,字迹要工整、清楚,不要写得太细长,字距适当,答题行距不宜过密。严格按照答题要求在相应题号指定的答题区域内第一行开始答题,并在规定区域内书写,切不可超出黑色边框,超出黑色矩形边框的答案无效。

(9) 答题时如需对非选择题答案进行修改,可用修改符号“双下划线”将原书写内容划去,然后紧挨着在其上方或下方写出新的答案,修改内容书写时与正文一样不能超过该题答题区域的黑色矩形边框,否则修改的答案无效。修改选择题答案可使用橡皮擦,但应注意不要使答题卡造成破损。禁止使用涂改胶带纸改错或用透明胶带纸粘扯欲修改的内容。

(10)规范作答选考题。按要求选好题目后作答,用2B铅笔将所选题目对应的题号右侧方框涂黑。评卷时按所涂题号进行评分,多涂、多答均按所涂的首题进行评分,不涂则按本选考题的首题进行评分。

(11)保持卡面清洁,不要折叠答题卡。

f43213efbba25e9e7f3e98deb60214fd.png

25.网上评卷有哪些流程?

计算机网上评卷是对传统考试评卷方式的一场革命。评卷时,通过高速扫描仪快速扫描考生的答题卡,选择题部分由计算机对比标准答案自动给分;非选择题部分按照事先划定的范围切割成一个个的图片,存入计算机中,然后由评卷老师在计算机上评阅。

非选择题的评卷工作是分科、分题进行的,按各科试卷的试题多少,分若干组,采取流水作业的办法评阅全卷。在正式评卷前,对参加评卷的教师进行培训,学习研究试题答案及评分标 准、评分办法、评卷纪律、注意事项等。在此基础上,由组长组织评卷员对考生的答卷进行试评, 通过试评, 在 评 分 标准规定的范围内, 制定各题的评分细则, 形成文字材料,报 学 科领导小组审查同意后执行, 以确保全组正确掌握评分标准。

非选择题实行“三评加终评”的评卷流程。评卷开始后,网上评卷系统将事先分割好的、隐去考生所有个人信息的考生答题图片随机分发到两位评阅相应题目的评卷员的计算机上,评卷员根据评分标准在计算机上对考生的答题给出分数。当评阅同一考生同一题目的两位评卷员所给的分数之差小千事先规定的差值阀限时,计算机自动取两个人的平均分作为该考生该题目的最终得分。如果出现两位评卷员所给的分数之差大于事先规定的差值阙限的情况,计算机自动将该考生该题目的图片发给该题目的题长,由题长进行三评。三评结束后,计算机自动对三位评卷员所给的分数进行两两对比,如果某两位评卷员所给的分数之差小于事先规定的差值阀限,计算机自动求取平均分;如果都大于规定的差值阀限,计算机会将该考生该题目图片发给学科评卷领导小组裁定,由学科组长组织评卷专家给出最终分数。

  选择题和非选择题评卷完毕后,由计算机汇总生成考生的成绩。

f43213efbba25e9e7f3e98deb60214fd.png

26.网上评卷有何优点?

一是减少评卷误差。评卷中的误差是考生和家长最为关心的问题,也是传统评卷中始终难以解决的问题。由于网上评卷系统对每道题都设置一个差值阈限,这样使误差得到了有效的控制。写分、加分、登分的误差减少到最小。在评卷过程中,评卷员直接敲击键盘打分,减少了笔误。另外,考 生成绩得分由计算机自动记录、加分、汇总,不会因字迹潦草而在加分、登分时错认数字,使登分、加分失误导致成绩差错的现象得到了有效的控制。

二是方便过程监控。传统的高考评卷凭借复查和抽查答卷来对评卷过程监控,但这不能实现连续的、全面的动态监控,所以一直没能真正有效地解决监控问题。网上评卷系统可使组长对每个评卷员进行实时动态监控, 对评卷质量进行追踪监控,通过对各个评卷员工作情况的比较,发现个别评卷员存在的问题,及时作出调整。

三是优化评卷流程。实行网上评卷取消了答卷流通的人工管理。以前在开始评卷之前,各评卷院校必须将答卷取走,评卷完毕后再送到省登分点,现在直接通过专线传输,解决了答卷交接中的安全问题。评卷结束后,考生的成绩自动合成,免去了许多烦琐的数据计算和统计。

四是确保评卷公正。评卷员凭本人用户名和密码进入评卷系统进行独立评阅,有效地防止了评卷员间的相互影响,评卷员在评卷过程中看到的答卷永远都是新的,既不知道这份答卷是初评还是复评,也不知道别人的评卷结果,这有利于评卷员进行独立判断、评分。而在传统评卷中,老师们直接在答卷上批改,一道题往往由多个老师来评,复评老师在评分时难免会受到初评老师的影响。网上评卷系统避免了这种现象的发生。

f43213efbba25e9e7f3e98deb60214fd.png

27.考生如何查询高考成绩?

预计6月下旬,考生可登录省教育考试院网站(网址:http:// www.hebeea. edu.cn)、省教育考试院公众号(hbsksy)、省教育考试院APP客户端或河北招生考试信息服务网(网址:http://www.hebeeb.com)查询考试成绩。其中,选择性考试再选科目成绩为等级赋分后的成绩。

f43213efbba25e9e7f3e98deb60214fd.png

           河北省2023年普通高校

        招生工作新视点

1.强化报名资格审查。对接教育部高校学信息和全省高学籍信息, 实时比对考生学籍信息;联合公安部门对考生户籍和居住证信息逐一核查,严防“ 高考移民”;与卫健、人社、退役军人等部门建立联审机制,强化优惠加分、优先录取等优惠资格审核,严防弄虚作假。

2.严厉打击违规行为。将防范手机等高科技作弊及群体性、有组织舞弊作为重点。考前联合公安、保密、无线电管理、通信管理等部门持续开展考试环境综合治理专项活动。考生入场全部采用人脸识别技术, 严防替考。强化安检措施, 严格执行国家教育考试入场安检规范, 严防手机等作弊器材入场。所有考场、 通道、卫生间等区域实现5G信号屏蔽全覆盖。考试期间强化监考巡考,实现全程监控、全程录像。考后严格执行考场视频回放制度,严查违规违纪行为。

3.严格三个专项计划报考资格。《教育部关于做好2023年普通高校招生工作的通知》(教学[2023]J1号)规定,从2023年起,往年被国家专项计划、高校专项计划、地方专项计划录取后放弃入学资格或退学的考生,不再具有专项计划报考资格。

4.加强涉考培训咨询机构治理。根据教育部部署,各地教育行政部门会同网信、科技、公安、文化和旅游、市场监管、体育等部门,加强对社会培训机构或个人开展涉考培训咨询的规范治理,严厉打击涉及虚假宣传、价格欺诈、组织或参与考试作弊、干扰破坏考试招生秩序等违规违法行为。

5.深化艺术类专业考试招生改革。按照教育部《关于进一步加强和改进普通高等学校艺术类专业考试招生工作的指导意见》精神,我省到2024 年,基本建立以统一高考为基础 、省级专业考试为主体,依据高考文化成绩、专业考试成绩,参考学生综合素质评价,分类考试、综合评价、多元录取的高校艺术类专业考试招生制度,形成促进公平、科学选才、监督有力的艺术人才选拔评价体系。

6.改革高校高水平艺术团招生办法。自2024 年起,高校高水平艺术团不再从高校招生环节选拔,由相关高校从在校生中遴选培养。

7.深化高校高水平运动队考试招生改革。自2024年起,进一步完善和规范高校高水平运动队考试招生工作,通过优化招生项目范围, 严格报考条件和资格审核,改进考试评价方式,提高文化成绩要求,完善招生录取机制,选拔培养德智体美劳全面发展且具有较高体育竞技水平的学生,为奥运会、世界大学生运动会等重大体育比赛和国家竞技体育后备人才培养体系提供人才支撑。


重要日程备忘




































f43213efbba25e9e7f3e98deb60214fd.png


1.考生填报志愿的作用是什么?

考生填报志愿的作用有两个:一是表达自己的升学愿望;二是作为普通高校录取的依据,无志愿考生不能参加录取。

f43213efbba25e9e7f3e98deb60214fd.png

2.填报志愿的时间是如何安排的?

志愿填报分批分段进行。

本科提前批、本科批及对口本科批的集中填报志愿时间安排在6月下旬; 在本科批填报最后一次征集志愿时,同时进行专科提前批、专科批及对口专科批的集中填报志愿。具体填报时间印制在考生准考证背面,并通过省教育考试院官方网站、微信公众号(hbsksy)等渠道向社会公布。

军队院校填报志愿安排在6月下旬,在本科提前批、本科批及对口本科批的集中填报志愿开始前进行。具体要求详见当年军队院校招生相关文件。

f43213efbba25e9e7f3e98deb60214fd.png

3.考生填报志愿方式和流程是怎样的?

考生填报志愿采取远程网上填报方式进行。考生在规定时间,使用计算机(系统不支持手机、平板等移动终端设备)网络登录省教育考试院网站(网址:http://www.hebeea.edu.cn,或直接访问http://gk.hebeea.edu.cn), 通过志愿填报系统进行志愿填报。须使用IE11、Chrome、Edge及以上版本浏览器,其他类型及版本不予支持,由于浏览器类型及版本使用不当可能会造成无法填报志愿或填报错误等后果,责任由考生自负。

具体填报流程如下图所示:

微信截图_20230421233549.png

(1)考生须使用用户名和密码登录系统,用户名为考生本人的考生号或身份证号,密码为考生高考报名时所设置的密码。

(2)首次登录时,考生须认真阅读《2023年河北省普通高校招生志愿填报承诺书》,并确认承诺。

(3)考生在志愿填报规定时间内,点击“填报”按钮,进入“2023 年河北省普通高校招生志愿填报表”页面。

(4)点击所要填报的批次(段),进入该批次(段)志愿填报页面。依次选择计划性质、科类(填报高校专项计划、高水平运动队、高水平艺术团的考生还须选择获得的资格类型),然后输入院校代号、专业代号(顺序志愿还须选择是否“服从专业调剂”),逐条完成填报后点击“保存”按 钮。保存成功后返回到“2023 年河北省普通高校招生志愿填报表”页面, 所报志愿会在“所填志愿”栏中显示出来。

在填报平行志愿(不含对口)时,需要提醒的是:

志愿填报系统提供了多种调整志愿顺序的方法:清空、插入、上移、下移、删除、调整、拖动。顺序调整完成后,须点击“保存”按钮,否则调整志愿顺序无效。

志愿填报页面左侧的“志愿导航”栏可以拖动,方便考生快速定位及志愿保存。

(5)考生在填报过程中如需修改志愿信息,可在“批次入口”选择“栏中点击相应批次(段)名称,进入志愿修改页面,在规定时间内进行志愿修改。在当次志愿填报截止时间前,考生可多次登录系统进行志愿修改和保存,最终以其网上最后一次修改并保存成功的志愿为准。

(6)考生志愿核查无误后,点击“安全退出”按钮安全退出系统。

(7)每次志愿填报结束后,系统将关闭 3 小时(以实际操作时间为准)进行系统维护。系统维护完成重新开放后,考生可登录系统查看自己所填报的志愿信息。


4.考生在网上填报志愿时应注意哪些事项?

(1)志愿填报应由考生本人完成,不要让他人代为填报,否则,造成的一切后果由考生承担。

(2)考生应在规定时间内填报志愿,未在规定时间内填报志愿的,视为自动放弃。提醒考生应尽早填报,避免因网络拥堵、断电等原因影响志愿填报。

(3)考生填报志愿时须“一个批次(段)一保存"。每批次(段)填报志愿后,须点击“保存”按钮进行志愿保存,未保存的志愿无效。志愿修改后,如未点击“保存”按钮,则修改无效,以上一次保存的志愿为准。

(4)考生须按照志愿顺序号依次填报平行志愿。填报志愿时,所填志愿序号必须连续,中间不能留空,否则系统无法保存。例如未填序号 1 (第一行)的院校及专业,而跳过去直接填了序号2 (第二行)的院校及专业,志愿填报系统不予支持。

(5)考生在上网填报之前应先在纸上填好2023 年河北省普通高校招生考生志愿填报草表,尽量不要在网上一边思考一边填报。这样既可以提高网上填报志愿的效率和准确度,同时又避免造成网络拥堵。如果考生登录系统后 20 分钟内无任何操作,再次进行操作时,系统会进行“考生未登录”提示,或被强制退出系统且未进行保存过的志愿不会保存,考生须重新登录系统进行填报。

(6)考生对填报志愿中系统给出填报有误提示,应从批次(段)、报考科类、计划性质、院校代号、专业代号、选科要求、性别要求,器种、唱法或舞种要求和本人情况等方面逐一查找出错原因。如果志愿信息存在错误无法进行确认,请在修改信息后再进行确认,否则可能导致志愿信息无效或不能体现本人最终意愿。

(7)多名考生使用同一台计算机填报志愿时, 每位考生填报志愿后一定要点击“安全退出”按钮,并关闭所有显示本人信息的页面,否则,后一位考生所填报的志愿信息可能会覆盖前面考生的志愿信息。同时还要注意,不要同时打开两个或两个以上填报页面,以免发生错误。

(8)考生要高度重视密码的设置与保管,密码设置不要过于简单,更不要将密码告诉他人,以防被他人篡改志愿,影响录取。如果忘记登录密码可进行密码重置,具体办法请参考第一章“高考各阶段考生密码有哪些重置办法?

f43213efbba25e9e7f3e98deb60214fd.png

5.高校专业(类)对考生选考科目有哪些要求?

一是对首选科目要求。高校专业(类)对考生的首选科目做出要求,考生选择的首选科目符合相关要求,方可填报相关志愿。比如,历史 科目组合下的专业(类),只有首选科目为历史的考生才能填报;物理科目组合下的专业(类),只有首选科目为物理的考生才能填报。

二是对再选科目要求。高校专业(类)对考生再选科目要求可分为四种:

(1)不提再选科目要求的,符合首选科目要求的考生均可报考;

(2)提出1门再选科目要求的,考生必须选考该科目方可报考,如某校某专业对化学科目提出要求,符合首选科目要求且选考化学的考生才能报考;

(3)提出2门再选科目要求且均须选考的,考生须同时选考这 2 门科目方可报考,如某校某专业对化学和生物科目提出要求,符合首选科目要求且同时选考化学和生物的考生才能报考;

(4)提出2门再选科目要求且只需选考其中1门的,考生选考该 2 门科目中的任意 1 门即可报考,如某校某专业要求考生选考化学或生物科目,符合首选科目要求且只要选考科目中有化学或生物的考生即可报考。

f43213efbba25e9e7f3e98deb60214fd.png

6.如何查询招生院校对选考科目的要求?

考生可以登录拟报考高校官方网站进行查询,或查阅《2023年河北省普通高等学校招生计划》。

f43213efbba25e9e7f3e98deb60214fd.png

7.2023年各录取批次及志愿是如何设置的?

(1)普通类

普通类录取批次分为本科提前批、本科批、专科提前批、专科批等四个批次。

本科提前批。分为 A、B、C 三段,分段依次录取。

本科提前批 A 段。包括有政治考察、面试、体检等特殊要求的国家专项计划、军队、公安、司法等本科专业。实行以学校为单位的顺序志愿模式。设 1 次集中填报志愿和 1 次征集志愿,每次可填报 1 所学校, 每所学校设 6 个专业志愿和 1 个专业服从调剂选项。

本科提前批 B 段。包括除本科提前批 A 段以外的其他国家专项计划、公费师范生和免费医学定向生等本科专业。实行以“专业(类)+学校”为单位的平行志愿模式,1个“专业(类) + 学校”为1个志愿。设1次集中填报志愿和1次征集志愿,每次最多可填报96个志愿。

本科提前批C段。包括高水平运动队、高水平艺术团、高校专项计划、定向就业招生等特殊类型招生本科专业。实行以学校为单位的顺序志愿模式。只设1次集中填报志愿,不进行志愿征集,可填报1所学校,设6个专业志愿和1个专业服从调剂选项。

本科批。包括列入本科提前批的普通类本科专业、地方专项计划、本科预科班等。实行以“专业(类)+学校”为单位的平行志愿模式,1个 “专业(类)+学校”为 1个志愿。设1次集中填报志愿和2次征集志愿, 每次最多可填报96个志愿。

专科提前批。包括有面试、体检等特殊要求的公安、司法等专科专业。实行以学校为单位的顺序志愿模式。设1次集中填报志愿和1次征集志愿, 每次可填报1所学校,每所学校设6个专业志愿和1个专业服从调剂选项。

专科批。包括未列入专科提前批的普通类专科专业。实行以“专业(类) +学校”为单位的平行志愿模式,1个“专业(类)+学校”为1个志愿。设1次集中填报志愿和2次征集志愿,每次最多可填报96个志愿。

(2)艺术类

艺术类招生分本科提前批和专科提前批两个批次。

本科提前批。分为 A、B、C 三段,分段依次录取。

本科提前批 A 段。包括教育部批准的独立设置艺术高校、参照执行高校的有关艺术类本科专业,原211工程高校的艺术类校考本科专业,有特殊要求无法实行平行志愿投档的艺术类统考和校际联考等本科专业。实行以学校为单位的顺序志愿模式。设 1 次集中填报志愿和 1 次征集志愿, 每 次 可填报 1 所学校, 每所学校设 6 个专业志愿和 1 个专业服从调剂选项。

本科提前批B段。包括使用河北省艺术统考或校际联考成绩作为专业成绩进行录取,并执行我省平行志愿统一投档原则的艺术类本科专业。实行以“专业(类)+学校”为单位的平行志愿模式,1个“专业(类)+学校” 为1个志愿。设1次集中填报志愿和1次征集志愿,每次最多可填报70个志愿。

本科提前批 C 段。包括除本科提前批A段、B段以外的其他艺术类本科专业。实行以学校为单位的顺序志愿模式。设1次集中填报志愿和1次征集志愿,每次可填报1所学校,每所学校设6个专业志愿和1个专业服从调剂选项。

专科提前批。包括艺术类专科专业。分平行志愿和顺序志愿两种模式。其中,使用河北省艺术统考或校际联考成绩作为专业成绩进行录取的, 实行以“专业(类)+学校”为单位的平行志愿模式,1 个“专业(类) +学校”为1个志愿,设1次集中填报志愿和1次征集志愿, 每次最多可填报 70 个志愿;使用艺术校考成绩作为专业成绩进行录取的,实行以学校为单位的顺序志愿模式,设1 次集中填报志愿和1次征集志愿,每次可填报1所学校,每所学校设6个专业志愿和1个专业服从调剂选项。

(3)体育类

体育类招生分本科提前批和专科提前批两个批次。

本科提前批。分为A、B两段,分段依次录取。

本科提前批A段。包括河北体育学院少数民族传统体育项目。实行以学校为单位的顺序志愿模式。设1次集中填报志愿和1次征集志愿。

本科提前批B段。包括使用河北省普通体育类专业考试成绩作为专业成绩进行录取的体育类本科专业。实行以“专业(类)+学校”为单位的平行志愿模式,1个“专业(类)+学校”为 1个志愿。设1次集中填报志愿和1次征集志愿,每次最多可填报70个志愿。

专科提前批。包括使用河北省普通体育类专业考试成绩作为专业成绩进行录取的体育类专科专业。实行以“专业(类)+学校”为单位的平行志愿模式,1个“专业(类)+学校”为1个志愿。设1次集中填报志愿和1次征集志愿,每次最多可填报70个志愿。

(4)对口类

对口类分对口本科批和对口专科批。均设1次集中填报志愿和1次征集志愿, 均实行平行志愿投档。每次最多可填报5所学校,每所学校设6个专业志愿和1个专业服从调剂选项。对口本科批随本科提前批B段填报志愿、录取, 对口专科批随专科提前批填报志愿、录取。

f43213efbba25e9e7f3e98deb60214fd.png

8.什么是征集计划?

征集计划是某批次(段)集中填报志愿录取结束后的院校缺额和新增计划, 由三部分组成:一是部分学校专业(类)因生源不足未录取满额,产生缺额;二是部分学校专业(类)因考生填报志愿不当或因身体条件不符合录取要求等原因被退档,形成缺额;三是平行志愿投档后,又有部分学校专业(类)向我省新增计划。

未被录取的考生可在规定时间内,按照公布的征集计划填报征集志愿。

f43213efbba25e9e7f3e98deb60214fd.png

9.填报平行志愿有哪些建议?

认真查阅《2023 年河北省普通高等学校招生计划》,包括院校名称、办学性质、专业(专业类)、层次、选考科目要求、学制、学费、计划数、专业备注等。

认真阅读院校的招生章程,详细了解院校招生政策、录取规则、学科优势 、专业设置、地理位置等方面信息,要特别注意各专业对考生的要求,如身体条件、外语语种、单科成绩、综合素质评价、选考科目要求等,对照拟报考学校专业(类)的要求,检查自己是否符合条件,避免出现因相关科目成绩偏低、身体条件不符合要求等原因被退档。

确定填报意向。综合考虑自己的选考科目、成绩位次、兴趣 爱好、专业特长、身体条件等各方面的因素,明确自己立志攻读的专业范围和院校范围,初步选择足够数量的专业(类)+院校”志愿。

分析研究相关数据。考生可根据成绩位次,结合往年院校专业的计划、录取等方面情况综合分析, 重点要研究近两年院校专业录取情况,收集汇总相关专业录取最高分、最低分和平均分对应的位次,与自己的成绩位次相比较,对初选的专业(类)+院校志愿进行筛选。

选择足够多的志愿。新高考模式下,平行志愿批次普通类每次最多可填报 96 个志愿,艺术类、体育类每次最多可填报70志愿。考生要珍惜机会, 尽可能多地选择并填报志愿。应尽量填满志愿,多一个志愿,就多一次录取机会。

合理安排志愿顺序。根据自己的成绩合理定位,志愿之间要有合理层次梯度,不宜全部填报高分段,也不必全部填报低分段志愿。要把心仪的志愿填在靠前的位置。

f43213efbba25e9e7f3e98deb60214fd.png

10.对平行志愿的认识误区有哪些?

一般来说对平行志愿的认识误区主要有三个:

误区一 :可以一档多投或多次投档。虽然同一批(段)考生一次可以填报多个学校专业(类)志愿,但在实际投档过程中,投档机会最多只有一次,也就是说,即使有多个学校专业(类)志愿符合投档要求,也只能投档到这些志愿中排在最前面的那一个,而且一旦投档,其余志愿随即失效,不再投档。投档后因某种原因被退档,即使后 面还有符合投档条件的志愿,也不会再次投档。

误区二:平行志愿没有先后顺序。对考生个人来说,平行志愿是有先后顺序的。平行志愿的检索顺序就是考生所填报的 1、2、3、4、5…… 志愿顺序,所以考生一定要精心安排志愿顺序,把最心仪的志愿填在靠前的位置。

误区三:实行平行志愿后没有了风险。平行志愿虽然可以降低风险,但不能消除所有风险:一是被退档的风险。一些学校专业(类)对考生的身体条件、外语语种、单科成绩等有要求。考生电子档案投到某个学校的专业(类),如果不符合该专业(类)的上述要求就会被退档。二是电子档案投不出去的风险。如果考生定位不准确,志愿填报过高,几个平行志愿之间没有拉开适当的梯度,就可能导致所填报的志愿都达不到投档要求,考生电子档案无法投出。

f43213efbba25e9e7f3e98deb60214fd.png

11.考生填报志愿的主要依据和参考资料有哪些?

一是招生计划。2023年我省普通类、体育类按历史科目组合和物理科目组合分别编制招生计划,艺术类不分历史科目组合和物理科目组合,统一编制招生计划。内容主要包括学校名称、学校代号、专业名称、层次、选考科目要求、计划数、学制、学费和备注等。考生填报志愿时,各项信息都要以招生计划公布的内容为准。

二是招生章程。主要内容包括高校全称、校址、层次(本科、专科)、办学类型、办学主体、专业教学培养使用的外语语种,身体健康状况要求, 进档考生录取规则(如对考生加分成绩的使用、投档成绩相同考生的处理、进档考生的专业安排办法及专业调剂录取办法等),学费标准,家庭经济困难学生资助政策及有关办理程序,颁发学历证书的学校名称、证书种类等信息,联系电话,网址,以及其 他须知等。考生可登录教育部“阳光高考信息平台(网址:https://gaokao.chsi.com.cn)、各高校的官方网站查看院校招生章程。如发现有关内容与招生计划不一致,要及时向相关院校咨询核实。

特别提醒:考生初步确定报考高校专业(类)后,一定要仔细阅读其招生章程,如高校招生专业(类)对外语语种、身体条件等有特殊要求的,会 在招生章程中予以明确。受篇幅和字段限制,志愿填报系统、《2023年河北省普通高等学校招生计划》以及《2023 年河北省普通高等学校招生章程》中的信息是不完备的,有些高校报送招生计划时仅摘录了招生章程中的部分内容,考生填报志愿前务必登录教育部“阳光高考信息平台(网址:https://gaokao.chsi.com.cn)或高校的官方网站查阅招生章程。

三是院校网站公布的信息。考生通过高校网站可了解高校基本情况、专业设置、地理位置等方面信息。

四是《普通高等学校招生体检工作指导意见》。其中规定了对患有某些疾病或有生理缺陷者不宜报考的专业。考生应根据体检结果,对照体检工作 指导意见和有关高校的招生章程,避开自已不宜报考的专业。

五是各类考生成绩统计表。高考成绩公布后,省教育考试院将在官网、官微公布当年“河北省普通高校招生各类考生成绩统计表,考生可在填报志愿时参考。

六是其他参考资料。包括《全国普通高校在河北招生录取分数分布统计》《2023年河北省普通高校招生报考指南》《2023 年河北省普通高等学校招生艺术类报考指南》。其中,《全国普通高校在河北招生录取分数分布统计》主要内容为近三年全国普通高校在河北省招生录取情况,包括分学校、分专业的计划数、录取数、最高分、最低分、平均分、差值及各专业录取分数分布统计;《 2023年河北省普通高校招生报考指南》和《2023 年河北省普通高等学校招生艺术类报考指南》介绍了我省高考录取的相关 政策规定、操作程序和注意事项,并收集整理了近年来高校录取的相关数据和资料,这些资料都可以为考生填报志愿提供参考。

f43213efbba25e9e7f3e98deb60214fd.png

12.什么是志愿填报辅助系统?

志愿填报辅助系统是为了更好地服务考生,在填报正式志愿前让考生提前进行计划查询并对心仪院校专业进行收藏及意向志愿预选。志愿填报辅助系统仅对集中填报志愿提供相关辅助功能,征集志愿时不提供。

志愿填报辅助系统免费提供给考生使用。

f43213efbba25e9e7f3e98deb60214fd.png

13.志愿填报辅助系统操作流程及注意事项有哪些?

考生在规定时间内,通过计算机网络访问省教育考试院网站(网址:http://www.hebeea.edu.cn, 或直接访问http://gk.hebeea. edu.cn),登录系统。须使用IE11、Chro me、Edge及以上版本浏览器,其他类型及版本不予支持。系统不支持使用手机、平板等移动终端。

(1)系统登录。考生须使用用户名和密码登录系统 ,用户名为考生本人的考生号或身份证号,密码为考生高考报名时所设置的密码。志愿填报辅助系统开放期间,同一考生在同一时间段只允许在一个终端登录,请广大考生妥善保管好密码及个人信息,谨防他人盗用。考生登录系统20分钟内无任何操作,再次进行操作时会被强制退出系统,考生需要重新登录。

(2)阅读提示。考生登录成功后,会显示温馨提示页面,请考生认真阅读。阅知后系统会自动显示批次列表页面。

(3)计划查询。系统会根据考生报考类别及选考科目情况将考生能够填报的批次、计划性质、科类全部列出,考生可以通过点击“进入”按钮,进入到相应批次、计划性质、科类的计划查询页面。

(4)收藏招生计划。在招生计划列表中,考生可以对感兴趣的专业进行收藏。收藏的招生计划将会加入到收藏列表。

(5)添加意向志愿。在收藏列表中,考生可以将心仪专业添加为意向志愿。在意向志愿列表页面,考生可对意向志愿进行顺序调整或删除。保存成功的意向志愿可在志愿填报系统中相应批次、计划性质、科类下进行一键加载。需提醒的是:意向志愿调整完毕后,务必点击“保存”按钮进行保存,否则调整结果无效。

(6)系统退出。退出系统时应点击页面右上方的“退出系统,以确保系统正常退出。

f43213efbba25e9e7f3e98deb60214fd.png

14.什么是河北省高考志愿填报智能参考系统?

河北省高考志愿填报智能参考系统是河北省教育考试院组织开发的、新高考后帮助考生进行志愿填报的智能参考系统,在专业(类)+学校”模式下帮助考生合理选择高考志愿,目的是为考生提供服务。

该系统包括院校信息、专业简介、百问百答、专业选择测试等服务模块,方便考生及家长了解院校、专业信息、高考相关政策、明确报考方向,提高考生志愿填报效率。该系统只需考生输入成绩、选考科目等信息后,就可筛选出该考生能报考的专业(类),考生还可以按照自己对省份、城市、学校、专业等个人偏好挑选出想报考的高校和专业(类),并可将志愿填报结果导出备查。

系统开放后,考生凭省教育考试院发放的使用码,登录河北招生考试信息服务网(网址:http://www.hebeeb.com)使用。

f43213efbba25e9e7f3e98deb60214fd.png

15.志愿填报系统、志愿填报辅助系统和志愿填报智能参考系统有什么区别?

志愿填报系统是考生填报高考志愿的唯一途径,无论是集中填报志愿还是征集志愿,考生都必须在规定时间内登录省教育考试院网站,通过该系统进行志愿填报,未按时填报或在其他系统填报的,都无法参加高考录取。

志愿填报辅助系统和志愿填报智能参考系统仅在集中填报志愿时提供辅助服务,不能代替志愿填报系统。使用这两个系统遴选的参考志愿必须通过志愿填报系统进行填报,方可生效。

f43213efbba25e9e7f3e98deb60214fd.png

16.填报志愿时考生如何对待优惠加分政策?

对具备加分资格的考生,我省按照加分后的分数投档。其中,我省地方性加分项目只适用于我省高校省内招生。

对于同时符合多种加分条件的考生,投档时,报考省外院校的使用国家加分最高一项,报考河北省院校的使用国家加分与省内加分中最高一项。根据教育部有关规定,所有高考加分项目及分值均不得用千高校不安排分省招生计划的艺术类专业、高水平艺术团、高水平运动队、高校专项计划等招生项目。

录取时是否考虑加分因素,按院校的有关规定执行。由于部分院校不承认或部分承认优惠加分政策,因此,享有优惠加分政策投档的考生,有可能因实际文化成绩较低而被院校退档。请考生在填报志愿前,务必认真阅读有关院校招生章程或向院校咨询。

f43213efbba25e9e7f3e98deb60214fd.png

17.艺术类专业考生填报志愿应注意哪些问题?

(1)填报使用我省艺术统考或校际联考成绩作为专业成绩录取的院校专业志愿,需要相应的专业统考或校际联考成绩、文化成绩达到我省划定的相应批次(段)、类别录取控制分数线(教育部批准的部分独立设置本科艺术院校和参照执行的院校艺术类本科专业,高考文化成绩录取控制分数线由高校确定)。

(2)填报各批次(段)我省统考涉及的, 但使用院校艺术类专业校考成绩录取的院校专业志愿,需要同时具备下列三个条件方可报考:一是院校专业校考合格;二是我省相应类别的专业统考成绩合格(美术类本科专业须我省美术类专业统考本科合格);三是文化成绩达到相应批次(段)的录取控制分数线(教育部批 准的部分独立设置本科艺术院校和参照执行的院校艺术类本科专业,高考文化成绩录取控制分数线由高校确定)。

(3)填报各批次(段)我省统考未涉及的,使用院校艺术类专业校考成绩录取的院校专业志愿,需要同时具备下列两个条件方可报考:一是院校专业校考合格(部分无须专业考试,直接按高考文化课成绩录取的艺术统考未涉及的校考专业除外);二是文化成绩达到相应批次(段)的录取控制分数线(教育部批准的部分独立设置本科艺术院校和参照执行的院校艺术类本科专业,高考文化成绩录取控制分数线由高校确定) 。

(4)河北省普通高等学校艺术类专业招生计划不区分历史科目组合、物理科目组合,统一编制。艺术类计划也有少数专业只招首选科目为历史或只招首选科目为物理的考生,我省根据院校的要求在招生计划中予以公布,不符合相应选考科目要求的考生不能填报和录取。为了便于考生查阅,艺术类专业招生计划在《2023年河北省普通高等学校招生计划(历史科目组合·对口)》《2023 年河北省普通高等学校招生计划(物理科目组合)》中均有编列,但它们是同一计划,并非历史科目组合和物理科目组合各自单独的招生计划数,个别专业单独注明只招首选科目为历史或只招首选科目为物理的考生的除外。

(5)使用校考成绩录取的专业若集中填报志愿未录取满额,对在我省未列分省计划的院校,我省不统一组织征集志愿填报,由缺额院校自行组织;对在我省列有分省计划的院校,我省统一组织征集志愿填报。

f43213efbba25e9e7f3e98deb60214fd.png

18.怎样填报艺术类平行志愿?

(1)正确理解平行志愿投档原则和录取规则。平行志愿投档原则是各省根据本省实际情况将考生档案投档到学校专业的具体办法,我省艺术类平行志愿按综合成绩投档。录取规则是指考生档案投到学校后,学校对进档考生依据其向社会公布的招生章程(或招生简章)中相关规定进行录取的具体办法。投档原则解决的是我省如何向学校投档的问题,而录取规则解决的是学校对进档考生如何确定录取或退档的问题,在主体和时间顺序上不 一样。

(2)根据综合成绩的位次情况,合理确定拟填报学校专业(类)。我省艺术类各类别平行志愿投档综合成绩构成不同,美术类、音乐类(含声乐类和器乐类)、舞蹈类、书法学、服装表演类综合成绩=高考文化总成绩(含政策性加分)×0.3+(专业成绩+专业满分)×750×0.7,结果四舍五入保留3位小数。其中,美术类专业满分300分,音乐类(含声乐类和器乐类)、舞蹈类、书法学、服装表演类专业满分均为200分;戏剧与影视学类、播音与主持艺术综合成绩=高考文化总成绩(含政策性加分)×0.7+(专业成绩十专业满分)×750×0.3 , 结果四舍五入保留3位小数。戏剧与影视学类、播音与主持艺术专业满分均为 200 分; 航空服务艺术与管理综合成绩即高考文化总成绩(含政策性加分)。

考生应根据自己高考文化总成绩和艺术统考(校际联考)专业成绩准确计算出综合成绩。在文化成绩公布后,我省将公布考生的综合成绩分布统计表,考生在知线知分知位次的情况下,综合考虑院校办学实力、专业实力、往年录取情况,结合本人意愿进行填报。

(3)清楚掌握拟报考学校专业(类)特殊要求和相关信息。部分专业(类)对文化成绩、专业成绩、选考科目、身体条件、乐器种类、声乐唱法、舞蹈种类等有要求。考生务必仔细查阅拟报考院校的招生章程(或招生简章), 结合招生计划的有关提示,准确填报,避免填报限报专业后被学校退档。

(4)注意规避平行志愿投档模式的风险。考生应认真研究分析招生院校专业的基本情况、招生计划和相关要求,结合分数、位次及本人意愿确定自己的填报志愿方案。普通类平行志愿风险 在艺术类专业平行志愿投档录取过程中也同样存在,因此选择的学校专业(类)一定要准确恰当定位, 志愿之间要适当拉开梯度。

f43213efbba25e9e7f3e98deb60214fd.png

 


19.体育类考生填报志愿有哪些注意事项?

(1)使用河北省普通体育类专业测试成绩作为专业成绩录取的体育类专业全部实 行平行志愿投档。在思想政治品德考核合格和身体条件符合要求、专业和文化成绩均达到录取控制分数线的前提下,按照“分数优先,遵循志愿”的原则结合高校专业(类)要求和调档比例,依据考生综合成绩从高到低的顺序,逐个依次检索考生院校专业志愿进行投档。综合成绩计算公式为:综合成绩=高考文化总成绩(含政策性加分)×0.3+(专业成绩专业满分)×750×0.7,结果四舍五入保留3位小数。体育类专业满分400分。

(2)报考普通体育类院校专业(类)志愿的考生必须参加河北省普通体育类专业测试,且专业成绩和文化成绩达到我省划定的相应批次(段)录取控制分数线。特别提醒:考生应注意平行志愿投档原则和录取规则的区别 (可参阅“艺术类平行志愿填报注意事项有关内容)。此外,要清楚了解拟报考院校对文化成绩、专业成绩、身体条件等要求,避免错填限报专业。

f43213efbba25e9e7f3e98deb60214fd.png

20.艺术类、体育类考生是否可以兼报普通类专业?

艺术类和体育类考生,可以兼报普通类专业,但每次填报志愿时(集中填报志愿或征集志愿),同一批次(段)只能选择同一个类别填报,不能既 填报艺术或体育类志愿,同时又填报普通类志愿。

f43213efbba25e9e7f3e98deb60214fd.png

21.填报军队院校志愿应注意哪些事项?

军队院校招生计划安排在本科提前批A段,实行顺序志愿投档,设1次集中填报志愿和1次征集志愿,考生每次可填报1所学校,每所学校设6个专业志愿和1个专业服从调剂选项。

具体要求详见当年军队院校招生相关文件。

f43213efbba25e9e7f3e98deb60214fd.png

22.填报招飞计划应注意哪些事项?

报考空军、海军、民航招飞计划的考生,须参加相关招飞单位组织的考核、面试体检且合格。招生计划安排在本科提前批A段,实行顺序志愿投档。

具体详见当年招飞单位相关要求。

f43213efbba25e9e7f3e98deb60214fd.png

23.填报公安院校志愿应注意哪些事项?

报考公安院校的考生须为具有河北省户籍的普通高中毕业生,须参加由省公安厅统一组织的考察、面试体检和体能评测且合格。公安院校招生计划安排在本科提前批A段和专科提前批,实行顺序志愿投档,设1次集中填报志愿和1次征集志愿,每批(段)每次只能填报1所院校,每所学校设6个专业志愿和1个专业服从调剂选项。

具体要求详见当年公安院校招生相关文件。

f43213efbba25e9e7f3e98deb60214fd.png

24.如何填报定向就业招生志愿?

定向就业招生计划单列,解放军陆军工程大学定向人防计划安排在本科提前批A段,实行顺序志愿投档,只能填报1所院校志愿,设6个专业志愿和1个专业服从调剂选项。其他院校定向计划安排在本科提前批C段,只能填报1所院校志愿,设 6个专业志愿和1个专业服从调剂选项。详见当年招生计划。

f43213efbba25e9e7f3e98deb60214fd.png

25.高水平运动队招生如何填报志愿?

报考高水平运动队的考生,须经教育部“阳光高考信息平台公示无误后方可填报。高水平运动队在本科提前批C段录取,只设1次集中填报志愿,不 进行志愿征集。可填报1所学校设6个专业志愿和1个专业服从调剂选项。其中,院校拟录取的一级及以上运动员由招生院校将名单报我省办理录取手续,已经录取的不再填报高考志愿。

f43213efbba25e9e7f3e98deb60214fd.png

26.高水平艺术团招生如何填报志愿?

报考高水平艺术团的考生, 经教育部“阳光高考信息平台公示无误后方可填报。高水平艺术团在本科提前批C段录取,只设1次集中填报志愿, 不进行志愿征集。可填报1所学校,设6个专业志愿和1个专业服从调剂选项。

f43213efbba25e9e7f3e98deb60214fd.png

27.国家专项计划招生如何填报志愿?

国家专项计划志愿安排在本科提前批A段和B段。通过国家专项计划资格审核的考生方可填报。

本科提前批A段的国家专项计划实行顺序志愿,设集中填报志愿和征集志愿,每次只能填报1所学校,设6个专业志愿和1个专业服从调剂选项。本科提前批 B 段的国家专项计划实行平行志愿,设集中填报志愿和征集志愿,每次最多可填报96个志愿,与免费医学定向生、公费师范生(含优师专项)同批一起填报,同批一起录取。

f43213efbba25e9e7f3e98deb60214fd.png

28.高校专项计划招生如何填报志愿?

高校专项计划列在本科提前批C段,实行顺序志愿, 不进行志愿征集,可填报1所学校,设6个专业志愿和1个专业服从调剂选项。经有关高校考核合格且在教育部“阳光高考信息平台公示的考生方可填报。

f43213efbba25e9e7f3e98deb60214fd.png

29.地方专项计划招生如何填报志愿?

地方专项计划安排在本科批,设集中填报志愿和两次征集志愿,每次最多可填报96个志愿,与其他普通类专业同批一起填报,同批一起录取。通过地方专项计划资格审核的考生方可填报。

f43213efbba25e9e7f3e98deb60214fd.png

30.如何填报公费师范生(含优师专项)志愿?

教育部直属部分师范院校和我省部分师范院校在我省安排公费师范生招生,批次安排在本科提前批B段,设集中填报志愿和征集志愿,每次最多可填报96个志愿,与免费医学定向生、国家专项计划同批一起填报,同批一起录取。

普通类的优师专项列在普通类本科提前批 B 段,编列到公费师范生类,单列院校代号,院校名称后单独标注“优师专项与公费师范生、免费医学定向生、国家专项计划一起实行平行志愿。除以上普通类公费师范生(含优师专项)招生之外,艺术、体育、高校专项计划等招生类型的公费师范生(含优师专项)安排在相应类别和批次。考生被公费师范生(含优师专项)录取后须签订协议,详情请咨询有关院校。

f43213efbba25e9e7f3e98deb60214fd.png

31.如何填报预科班志愿?

预科班招生包括少数民族本科预科班和边防军人子女预科班。含在普通本科批内,设集中填报志愿和征集志愿,每次最多可填报96个志愿,与其他普通类学校同批一起填报,同批一起录取。

   其中少数民族本科预科班只招收少数民族考生,边防军人子女预科班只招收通过边防军人子女资格审查的考生。

f43213efbba25e9e7f3e98deb60214fd.png

32.志愿填报截止后还可以补报志愿吗?

志愿填报系统将按照规定的志愿填报时间准时开放和关闭,考生须按照有关操作流程,在规定时间内完成志愿填报。志愿填报截止后, 接受任何形式的补报志愿。

f43213efbba25e9e7f3e98deb60214fd.png

33.未填报集中志愿的考生,是否可以填报该批(段)的征集志愿?

   可以。未录取的考生,都可按照征集志愿填报时间安排,登录省教育考试 院网站查询征集计划,并在规定时间内填报征集志愿。

f43213efbba25e9e7f3e98deb60214fd.png

34.如何对待“冷门”和“热门”专业?

所谓“冷门”,人们通常理解为报考人数相对较少,竞争度相对较低的专业;所谓“热门,是指报考的考生较多,竞争十分激烈的专业。一般来说,“冷”与“热”是与当时的就业情况相联系的,就业形势好的专业成为考生追逐的热点,然而相当部分“热门”专业的就业并不是恒定的,报考时是“热门,就业时不一定是“热门”。因为社会对这些专业的需求可能逐渐饱和了,甚至过剩了,反倒成了“冷门”。相反有的专业报考时并不是很“热,可到就业时却受录用单位的欢迎。有志向的考生不要被“热门”“冷门”之类的字眼束缚而放弃自己的爱好和兴趣。

f43213efbba25e9e7f3e98deb60214fd.png

35.填报志愿时如何兼顾个性因素?

(1)志向。每个人的志向、理想是激发其奋发努力的动力之一,也是事业成功不可缺少的条件之一。所以在填报志愿时,首先要明志,只有明确自己的志向,才能合理地选择自己的人生。

(2)特长。特长又称为“性向特长”,是指完成某一特定的活动所必须具备的潜在能力。所谓潜在,就不是现实的能力,而是指能够轻易掌握这种能力的可能性。在与自己性向特长相吻合的事业或专业领域内,往往可以比较快地获得成功,因此从这一点出发,又把性向特长定义为“对从事特定职业或职业群工作成功率的预见力”。选择了符合自己特长的专业, 无 疑在未来的学习、工作中可以扬长避短,充分发挥自己的聪明才智。俗话说,最了解自己的还是自己。每个考生都应认真做一次自我分析,看看到底喜欢哪一个学科。是动手能力强,还是善于动脑;表象思维和逻辑思维哪一个更有优势;组织管理能力、艺术修养、口头与书面表达能力,在同学中处什么位置等等。这些都是选择志愿的参考因素。

(3)气质和性格。众所周知, 由先天性遗传因素和后天教育环境的不同, 使青少年形成了不同的气质和性格。如有的同学活泼开朗,有的则沉着稳健;有的是急脾气,有的是慢性子;有的反应灵活敏捷,有的则小心谨慎、三思而后行。气质和性格对人的实践活动有一定的影响,在未来的事业中, 每个考生总是要从事一定的实践活动。因此在选择志愿时应考虑考生本人的气质和性格。比如,要求反应迅速灵活的专业,对于多血质和胆汁质的考生较为合适;要求耐久性和细心谨慎的专业,对黏液质和抑郁质的考生较为合适;要求性格活跃、善于交际的某些专业(如新闻、外交、旅游、艺术等),对于外向型性格的考生较为合适;要求沉静和有坚忍不拔毅力的专业(如数学、物理等抽象理论学科),则适合于内向性格的考生。

 

f43213efbba25e9e7f3e98deb60214fd.png

36.河北招生考试信息服务网网上咨询活动何时开始?

我省将于6月中旬至高考录取结束,在河北招生考试信息服务网(网址: http: //www.hebeeb.com) 设立网上咨询大厅,开展网上咨询活动,相关工作人员将在线与考生互动,解答有关志愿填报、录取等方面的疑问。

f43213efbba25e9e7f3e98deb60214fd.png

           河北省2023年普通高校

        招生工作新视点

1.强化报名资格审查。对接教育部高校学信息和全省高学籍信息, 实时比对考生学籍信息;联合公安部门对考生户籍和居住证信息逐一核查,严防“ 高考移民”;与卫健、人社、退役军人等部门建立联审机制,强化优惠加分、优先录取等优惠资格审核,严防弄虚作假。

2.严厉打击违规行为。将防范手机等高科技作弊及群体性、有组织舞弊作为重点。考前联合公安、保密、无线电管理、通信管理等部门持续开展考试环境综合治理专项活动。考生入场全部采用人脸识别技术, 严防替考。强化安检措施, 严格执行国家教育考试入场安检规范, 严防手机等作弊器材入场。所有考场、 通道、卫生间等区域实现5G信号屏蔽全覆盖。考试期间强化监考巡考,实现全程监控、全程录像。考后严格执行考场视频回放制度,严查违规违纪行为。

3.严格三个专项计划报考资格。《教育部关于做好2023年普通高校招生工作的通知》(教学[2023]J1号)规定,从2023年起,往年被国家专项计划、高校专项计划、地方专项计划录取后放弃入学资格或退学的考生,不再具有专项计划报考资格。

4.加强涉考培训咨询机构治理。根据教育部部署,各地教育行政部门会同网信、科技、公安、文化和旅游、市场监管、体育等部门,加强对社会培训机构或个人开展涉考培训咨询的规范治理,严厉打击涉及虚假宣传、价格欺诈、组织或参与考试作弊、干扰破坏考试招生秩序等违规违法行为。

5.深化艺术类专业考试招生改革。按照教育部《关于进一步加强和改进普通高等学校艺术类专业考试招生工作的指导意见》精神,我省到2024 年,基本建立以统一高考为基础 、省级专业考试为主体,依据高考文化成绩、专业考试成绩,参考学生综合素质评价,分类考试、综合评价、多元录取的高校艺术类专业考试招生制度,形成促进公平、科学选才、监督有力的艺术人才选拔评价体系。

6.改革高校高水平艺术团招生办法。自2024 年起,高校高水平艺术团不再从高校招生环节选拔,由相关高校从在校生中遴选培养。

7.深化高校高水平运动队考试招生改革。自2024年起,进一步完善和规范高校高水平运动队考试招生工作,通过优化招生项目范围, 严格报考条件和资格审核,改进考试评价方式,提高文化成绩要求,完善招生录取机制,选拔培养德智体美劳全面发展且具有较高体育竞技水平的学生,为奥运会、世界大学生运动会等重大体育比赛和国家竞技体育后备人才培养体系提供人才支撑。


重要日程备忘




































f43213efbba25e9e7f3e98deb60214fd.png


1.什么是“学校负责、招办监督”的录取原则?

高校和省级招办应按照“学校负责、招办监督”的原则实施新生录取工作。高校应按照向社会公布的招生章程中的录取规则进行录取。对思想政治品德考核合格、身体健康状况符合相关专业培养要求、投档成绩达到同批录取控制分数线并符合学校调档要求的考生,是否录取以及所录取的专业由高校自行确定,高校负责对已投档但未被录取考生的退档原因作出解释, 高校不得超计划录取。省级招办负责监督在本地区招生高校执行国家招生政策、招生计划情况,纠正违反国家招生政策、规定和违背录取规则等行为。

f43213efbba25e9e7f3e98deb60214fd.png

2.各批各类录取控制分数线是如何划定的?

省招生委员会根据高校在我省安排的招生计划数、生源情况、考生成绩等因素,分类别综合考虑,确定各批次录取控制分数线。

普通类根据考生高考文化总成绩(含政策性加分),按照历史科目组合、物理科目组合招生计划数的一定比例,结合生源等情况,分别划定本科、专科录取控制分数线以及特殊类型招生录取控制分数线。特殊类型招生录取控制分数线主要用于强基计划、高校专项计划、高水平艺术团及其他有相应文化成绩要求的院校(专业)招生。

艺术类不区分历史科目组合、物理科目组合,根据艺术各类别考生高考文化总成绩(含政策性加分)、专业考试成绩和招生计划数,结合生源等情况,划定各类别本科、专科录取控制分数线。

体育类根据考生高考文化总成绩(含政策性加分)、专业考试成绩,按照历史科目组合、物理科目组合招生计划数的一定比例,结合生源等情况,分别划定本科、专科录取控制分数线。

对口类根据考生对口总成绩(含政策性加分),按照各专业类招生计划数的一定比例,结合生源等情况,分别划定本科、专科录取控制分数线。

f43213efbba25e9e7f3e98deb60214fd.png

3.投档原则是什么?

普通高校招生普通类、体育类按照历史科目组合和物理科目组合,分别投档录取;艺术类一般不区分历史科目组合和物理科目组合,一起投档录取,有特殊要求的结合高校有关要求投档。

投档模式分为顺序志愿和平行志愿两种。顺序志愿的调档比例原则上在 120%以内,平行志愿的调档比例原则上在105% 以内,高水平运动队、高水平艺术团、高校专项计划、部分艺术类等特殊类型招生的高校专业不限定投档比例。

(一)普通类

顺序志愿投档原则为“志愿优先、遵循分数”,将控制线上未录取的有志愿考生,结合高校要求,依据高校调档比例,按高考文化总成绩(含政策性加分)从高分到低分排序依次投档,由高校择优录取。

平行志愿投档原则为“分数优先、遵循志愿、一次投档、不再补档“,将控制线上未录取的有志愿考生,结合高校各专业

(类)要求,依据高校调档比例,按高考文化总成绩(含政策性加分)从高分到低分排序,遵循考生的志愿顺序依次投档,由高校择优录取。

投档时,当遇到多名考生高考文化总成绩(含政策性加分)相同时,依次按语文数学两门成绩之和、语文数学两门中的单科最高成绩、外语单科成绩、首选科目单科成绩、再选科目单科最高成绩、再选科目单科次高成绩由高到低排序投档;如仍相同,比较考生志愿顺序,顺序在前者优先投档,志愿顺序相同则全部投档,是否录取由高校决定。

军队院校、综合评价招生等有特殊投档要求的,按照有关文件执行。

(二)艺术类、体育类

顺序志愿按照“志愿优先”的原则进行投档,投档时根据考生志愿,将报考该校且符合条件的考生档案全部提供给有关高校审阅, 由 高校根据向社会公布的招生章程(或招生简章)中的录取规则进行录取,遗留问题由高校负责解释。

平行志愿按照“分数优先、遵循志愿、一次投档、不再补档”的原则, 将控制线上未录取的有志愿考生,结合高校各专业(类)要求,依据高校各专业(类)调档比例,按综合成绩从高分到低分排序,遵循考生的志愿顺序依次投档,由高校择优录取。投档时,当遇到多名考生综合成绩相同时, 依次按高考文化总成绩(含政策性加分)、语文数学两门成绩之和、语文数学两门中的单科最高成绩、外语单科成绩、首选科目单科成绩、再选科目单科最高成绩、再选科目单科次高成绩由高到低排序投档;如仍相同, 比 较考生志愿顺序,顺序在前者优先投档,志愿顺序相同则全部投档,是否录取由高校决定。

综合成绩计算公式为:

①美术类、音乐类(含声乐类和器乐类)、舞蹈类、书法学、服装表演类: 综合成绩=高考文化总成绩(含政策性加分)×0.3 + (专业成绩 专业满分)×750× 0.7 ,结果四舍五入保留3位小数。其中,美术类专业满分 300 分,音乐类(含声乐类和器乐类)、舞蹈类、书法学、服装表演类专业满分均为 200 分。

②戏剧与影视学类、播音与主持艺术:综合成绩=高考文化总成绩(含政策性加分)×0.7+(专业成绩+专业满分)×750×0.3,结果四舍五入保留 3位小数。戏剧与影视学类、播音与主持艺术专业满分均为200分。

③航空服务艺术与管理综合成绩即高考文化总成绩(含政策性加分)。

④体育类:综合成绩=高考文化总成绩(含政策性加分)×0.3 + (专业成绩-专业满分)×750 ×0.7,结果四舍五入保留3位小数。体育类专业满分400分。 

(三)高水平运动队、高水平艺术团、高校专项计划等特殊类型

高水平运动队、高水平艺术团、高校专项计划等特殊类型招生,将填报志愿且符合条件的考生档案全部提供给有关高校审阅,由高校根据向社会公布的招生章程中的录取规则进行录取,遗留问题由高校负责解释。

(四)对口类

对口类专业按对口总分(含政策性加分)投档,实行平行志愿。投档原则为“分数优先、遵循志愿、一次投档、不再补档” 将同一专业类控制线上未录取的有志愿考生,结合高校要求和调档比例确定投档考生数,按对口总分(含政策性加分)从高分到低分排序,遵循考生的志愿顺序依次投档。

遇到多名考生对口总分(含政策性加分)相同且多于应投考生数时,依次比较语文、数学、英语单科成绩,若单科成绩均相同,则全部投档。

f43213efbba25e9e7f3e98deb60214fd.png

4.高校招生录取规则是什么?

高校招生录取规则由高校依据国家有关规定制定,并通过招生章程向社会公布。主要内容有选考科目要求、调档比例、专业安排办法、身体要求、外语语种、成绩要求等。在统考成绩达到 同批录取控制分数线的考生中,高校确定调阅考生档案的比例。我省在国家允许的投档比例范围内,按高校提出的调档比例和相关工作规程向其投放考生电子档案。招生院校在我省投放的考生电子档案范围内,按事先公布的录取规则择优录取。

“学校”为志愿单位投档的高校,专业安排办法一般有以下三种方式:

(1)分数优先。所谓分数优先就是投档到该校的所有考生,从高分的学生开始依次按专业进行录取,也就是说,分数高的学生优先按专业顺序进行录取。

(2)志愿优先。所谓志愿优先就是在报考该院校的考生中,首先考虑的是考生的专业志愿,也就是说先考虑第一专业志愿, 再考虑第二专业志愿,依次类推。比如,一个考生未能达到第一专业志愿最低录取分数,将转入下面的专业志愿排序,依次类 推。如该专业前面已录满,不管该生分数多高,也不能进入该  专业。

(3)分数级差。所谓分数级差就是在考生的前后两个专业之间设定一个分数级差,比如分数级差是3分,那么某一专业第二专业志愿的考生只要比报考该专业的第一志愿考生高出3分以上,报考该专业第三志愿的考生只要比报考该专业的第二志愿考生高出3分以上,比报考该专业的第一志愿考生高出6分以上,就可以优先录取到本专业。

另外,还要了解高校对考生的加分政策、优惠条件、投档分数等其他要求。具体专业安排办法请认真阅读高校招生章程或向有关院校咨询。

f43213efbba25e9e7f3e98deb60214fd.png

5.各批次(段)在录取时是否相互影响?

录取是按批次(段)依次进行的,如果考生被某一批次(段)的院校录取,就不能参加以后批次(段)的录取,也不得转录;如果考生在某一批次(段)未被录取,将自动转入下一批次(段)重新开始参加录取,其上一批次(段)所填志愿不会影响下一批次(段)录取。

f43213efbba25e9e7f3e98deb60214fd.png

6.网上录取基本流程是什么?

(1)院校进行网络用户注册。

(2)院校核对招生计划。

(3)院校根据模拟投档(本科批)情况调整投档比例和招生计划。

(4)我省按院校调档要求提供考生电子档案。

(5)院校网上审阅考生电子档案。

(6)院校拟定预录取名单和预退档名单。

(7)录取审核。

(8)统一打印、寄发录取新生名册等材料。

(9)院校按新生名册寄发考生录取通知书。

f43213efbba25e9e7f3e98deb60214fd.png

7.平行志愿投档是如何进行的?

实行平行志愿的批次,投档时,按照“分数优先、遵循志愿、一次投档、不再补档”的原则,将控制线上未录取的有志愿考生按投档成绩从高到低排序,结合高校各专业(类)要求,依据高校调档比例确定投档考生数进行投档。具体地讲,就是先从排在第一位的考生开始,计算机根据该生填报的 1、2、3、4、5……多个平行志愿专业(类)或院校,从第 1 个志愿开始依次检索,如果第 1 个志愿有空额就投档,如已满额,则检索第 2 个志愿,依此类推。一旦投档,该生后面的志愿即失效。然后再检索排在第二位的考生……直到所有专业(类)或院校计划满额或者没有符合投档条件的考生为止。如果某一考生填报的所有志愿都不符合投档条件,则不能被投档。

特别强调的是:在同一批(段)虽然考生一次可以填报多个志愿,但在实际投档过程中,投档机会最多只有一次,也就是说,无论有几个志愿符合 投档要求,只能投档到排在前面的那一个,一旦投档,其余志愿随即失效,不再投档。投档后因某种原  因被退档,即使后面还有符合投档条件的志愿,也不再投档。

每批(段)平行志愿投档后,如专业(类)或院校退档后出现缺额,根据公平原则,不能再续发考生档案。缺额计划通过征集志愿完成。

f43213efbba25e9e7f3e98deb60214fd.png

8.征集志愿是如何投档的?

征集志愿投档原则与集中填报志愿的投档原则相同。相关批次最后一次征集志愿录取时,如果线上生源不足,有降分投档的可能。

f43213efbba25e9e7f3e98deb60214fd.png

9.什么是投档线?

在录取时,按照院校专业(类)的招生计划和调档要求,根据考生志愿填报情况,向招生院校专业(类)批量投档。批量投档的分数最低的考生成绩即为该院校专业(类)的投档线(亦称提档线、调档线、进档线)。

f43213efbba25e9e7f3e98deb60214fd.png

10.国家确定的优惠加分项目有哪些?分值是多少?

符合下列条件的考生可在考生文化统考成绩总分基础上增加分值投档, 由高校审查决定是否录取。

(1)少数民族自治县(含民族县)的少数民族考生增加10 分。

(2)归侨、归侨子女、华侨子女和台湾省籍(含台湾户籍)考生增加10 分。

(3)自主就业退役士兵增加 10 分。

(4)烈士子女增加 20 分。

(5)在服役期间荣立二等功以上或被战区以上单位授予荣誉称号的退役军人增加 20 分。

f43213efbba25e9e7f3e98deb60214fd.png

11.河北省确定的优惠加分项目有哪些?分值是多少?

具有河北省户籍的农村独生子女考生增加10分,加分范围只适用于我省高校在省内招生,可在考生文化统考成绩总分基础上增加分值投档,由高校审查决定是否录取。2016年1月1日(含)以后出生的农村户口独生子女考生参加高考,不再享受加分。

f43213efbba25e9e7f3e98deb60214fd.png

12.投档录取时如何使用优惠加分?

对于同时符合多种加分条件的考生,投档时,报考省外院校的使用国家加分最高一项,报考河北省院校的使用国家加分与省内加分最高一项。具备加分资格的考生,我省按照加分后的分数投档,录取时是否考虑加分因素, 由招生院校按有关规定执行。所有高考加分项目及分值均不得用于高校不安排分省招生计划的艺术类专业、高水平艺术团、高水平运动队、高校专项计划等招生项目。

f43213efbba25e9e7f3e98deb60214fd.png

13.优先录取的规定有哪些?

优先录取是录取环节的照顾政策, 包括两种照顾方式。一是参加全国统考录取并达到有关高校投档要求的优先录取,指对符合这类照顾条件的考生, 按有关高校投档要求进行投档后,高校对进档考生中符合照顾资格的考生应予以优先录取。符合这些条件的考生虽然没有享受加分,但根据志愿和分数等情况被投档到高校之后,应优先录取。二是同等条件下优先录取,指考生被投档到高校之后, 如果一些考生条件相当(如相关考试成绩、专业志愿、身体条件、获奖情况等条件相同),但因高校招生计划不足不能全部录取时,应优先录取符合照顾条件的考生。两种优先录取类型的照顾体现在投档后的高校录取环节,在投档环节没有照顾。

①平时荣获二等功或者战时荣获三等功以上奖励军人的子女,一至四级残疾军人的子女,因公牺牲军人的子女,驻国家确定的三类以上艰苦边远地区和西藏自治区、解放军总部划定的二类以上岛屿工作累计满 20 年军人的子女,在国家确定的四类以上艰苦边远地区或者解放军总部划定的特类岛屿工作累计满 10 年军人的子女,在飞或停飞不满1年或达到飞行最高年限的空勤军人的子女,从事舰艇工作满20年军人的子女,在航天和涉核岗位工作累计满15年军人的子女,参加全国统考录取并达到有关高校专业(类)投档要求的,应予以优先录取。

烈士子女、因公牺牲军人的子女、现役军人子女报考军队院校, 投档范围内优先录取。

②退出部队现役的考生、残疾人民警察参加全国统考录取并达到有关高校专业(类)投档要求的, 在与其他考生同等条件下优先录取。

③公安烈士、公安英模、因公牺牲、一级至四级因公伤残公安民警的子女参加全国统考录取的,按照《关于进一步加强和改进公安英烈和因公牺牲伤残公安民警子女教育优待工作的通知》(公政治 C2018J 27 号)的有关规定执行。

③国家综合性消防救援队伍人员及其子女参加全国统考录取的,参照军人有关优待政策执行。

④经共青团中央青年志愿者守 信联合激励系统认定,获得5A级青年志愿者的,达到有关高校专业(类)投档要求的,在与其他考生同等条件下优先录取。

f43213efbba25e9e7f3e98deb60214fd.png

14.军队院校如何录取?

在本科提前批 A 段录取,实行顺序志愿投档,设 1 次集中填报志愿和 1 次征集志愿,每次可填报1所学校, 每所学校设6个专业志愿和1个专业服从调剂选项。

集中填报志愿录取时,根据省军区提供的政治考核、军检合格考生名单, 依据考生志愿和分数,区分性别,按招生计划数量的110 %投档(投档数量按照四舍五入取整)。总分相同的依次比较单科成绩,其中,历史科目组合比较顺序为语文、数学、外语,物理科目组合比较顺序为数学、语文、外语。除同分考生外,不得超比例投档。军队院校根据考生填报的专业志愿,由高分到低分依次确定录取对象。达不到相应专业军检标准的,不得录取到该专业。

集中填报志愿录取结束后,如果没有完成招生任务,将向社会公布缺额计划,公开征集文化成绩达到我省特殊类型招生录取控制分数线且军检合格考生志愿。征集志愿录取时,按照缺额计划数量的100 %向军队院校投档。院校不得拒录符合录取条件的征集志愿考生。

烈士子女、因公牺牲军人的子女、现役军人子女,在投档范围内优先录取。

具体规定以当年军校招生文件为准。

f43213efbba25e9e7f3e98deb60214fd.png

15.公安院校如何录取?

公安院校根据招生层次分别安排在本科提前批A段和专科提前批,均实行顺序志愿投档,设1次集中填报志愿和1次征集志愿,每次可填报1所学校, 每所学校可填报6个专业志愿和1个专业服从调剂选项。

每批次集中填报志愿录取时,将报考该院校的政治考察和面试、体检、体能测评均合格且达到相应批次控制线的考生,依据考生分数,按照招生计划的100 %投档,招生院校负责审查录取。如集中填报志愿未完成招生计划,向社会公布缺额计划,进行征集志愿填报,并按照计划缺额数量的 100 % 向招生院校投放征集志愿档案。招生院校负责对投档但未录取考生的解释及其他遗留问题的处理。

具体规定以当年公安招生文件为准。

f43213efbba25e9e7f3e98deb60214fd.png

16.司法类院校提前录取专业如何录取?

根据教育部、司法部有关文件规定,中国政法大学、西南政法大学、中南财经政法大学、华东政法大学、西北政法大学等学校的侦查学、治安学、边防管理和刑事科学技术等专业和中央司法警官学院各专业在本科提前批 A 段录取,设 1 次 集中填报志愿和 1 次征集志愿,每次可填报 1 所学校,每所学校设 6 个专业志愿和 1 个专业服从调剂选项,实行顺序志愿投档。

我省从集中填报志愿报考该院校的上线考生中,依据考生分数,按照招生院校提出的调档比例投档,招生院校负责审查录取。当集中填报志愿上线考生数量不足时,进行征集志愿投档录取。

如个别司法院校进行政治考察、面试及体能测试,我省从政治考察、面试及体能测试合格且集中填报志愿报考该院校的上线  考生中,依 考生分数,按照招生院校提出的调档比例投档,招生院校负责审查录取。当集中填报志愿上线考生数量不足时,进行征集志愿投档录取。

具体规定以当年司法类院校招生政策为准。

f43213efbba25e9e7f3e98deb60214fd.png

17.艺术类专业如何录取?

艺术类专业招生录取分为本科提前批、专科提前批两个批次。本科提前批分为 A、B、C 三段, 按顺序依次录取;专科提前批分平行志愿和顺序志愿两种模式,考生同一个志愿只能选择其中一种模式填报,不能兼报。

(1)高校录取的艺术类专业考生须相应专业考试合格。部分无须专业考试,直接按高考文化课成绩录取的艺术统考未涉及的校考专业除外。

(2)经教育部批 准的部分独立设置本科艺术院校(含部分艺术类本科专业参照执行的少数高校)可按教育部规定自行划定本校艺术类本科专业高考文化课成绩录取控制分数线。独立设置的本科艺术院校艺术类专科专业及其他高校所有艺术类专业录取的考生,其高考文化课成绩必须达到我省相应批次录取控制分数线。

(3)使用艺 术 校考成绩作为专业成绩录取的艺术类专业。将校考合格且经我省备案(部分无须专业考试,直接按高考文化课成绩录取的艺术统考未涉及的校考专业除外),文化考试成绩达到我省相应批次录取控制分数线的考生(经教育部批准的部分独立设置本科艺术院校及参照执行的高校艺术类本科专业不受我省相应批次录取控制分数线限制)档案全部提供给考生报考高校审阅,由高校择优录取。遗留问题由高校负责解释。其中报考我省统考涉及的校考专业还须相应类别的专业统考合格(美术类本科专业须我省美术类专业统考本科合格) 。

按教育部规定,对于可授予艺术学学士学位的艺术教育、服装设计与工程、风景园林、文化产业管理等4个非艺术学门类专业,高考文化课成绩要求不得低于本校非艺术类专业所在批次录取控制分数线,其中允许开展艺术类专业单独考试招生的高校,对上述专业的高考文化课成绩要求不得低千河北省普通类本科批录取控制分数线。

(4)录取规则。由高校根据学校实际制定并向社会公布。录取时,按照我省投档原则向高校投档后,高校在投档生源范围内按照事先向社会公布的录取规则进行录取。遗留问题由高校负责解释。

具体投档原则见本章第 3 条,录取时间及相关要求按河北省有关文件执行。

f43213efbba25e9e7f3e98deb60214fd.png

18.普通体育类专业如何录取?

普通体育类院校(专业)招生录取分为本科提前批、专科提前批两个批次, 本科提前批分为 A、B两段,按顺序依次录取;本科提前批 B 段、专科提前批,均实行平行志愿投档。投档原则见本章第 3 条。

河北体育学院少数民族传统体育项目安排在本科提前批 A 段录取, 实行顺序志愿投档。

录取规则:由高校根据实际制定并向社会公布。录取时,我省按照河北省投档原则向高校投档后,高校在投档生源范围内按照事先向社会公布的录取规则进行录取。遗留问题由高校负责解释。

f43213efbba25e9e7f3e98deb60214fd.png

19.高水平艺术团招生如何录取?

报考高校高水平艺术团的考生,在本科提前批C段录取,根据考生所报志愿,将具有高水平艺术团招生资格且经教育部“阳光高考“信息平台公示的考生,按照试点高校有关要求,向高校提供档案,由高校审查决定是否录取,在本科批投档前审核录取完毕。高水平艺术团学生要接受所在院系和艺术团双重管理,既要按本专业培养方案完成专业学习任务,也要认真履行参加排练和演出的义务。

f43213efbba25e9e7f3e98deb60214fd.png

20.高水平运动队招生如何录取?

试点高校按照本校发展定位和人才培养要求,合理确定本校运动队录取考生高考文化课成绩最低要求。报考高校高水平运动队的考生,在本科提前 C段录取,根据考生所报志愿,将具有高水平运动队招生资格且经教育部“阳光高考”信息平台公示的考生,按高校要求,向高校提供档案,由高校审查决定是否录取,在本科批投档前审核录取完毕。

试点高校面向一级运动员等组织的部分运动员单独招生工作,由招生院校按教育部有关规定办理,将教育部“阳光高考”信息 平台公示无异议的拟录取名单,报我省办理录取备案手续。

高水平运动队学生要接受所在院系和运动队双重管理,既要按本专业培养方案完成专业学习任务,也要认真履行参加训练和比赛的义务。

f43213efbba25e9e7f3e98deb60214fd.png

21.定向就业招生如何录取?

定向就业招生在同批非定向投档前,按照院校招生计划,在该校非定向本科批同科类普通类专业最低投档线下20分以内、同批控制分数线以上,根据考生志愿从高分到低分投档,由院校审核录取。在本批次非定向投档前审核录取完毕。其中定向西藏的最多降40分,不受 控制线限制。

若院校完不成定向就业招生计划, 则转为非定向计划执行。其中,定向西藏就业招生计划因报考生源不足确需调整计划的,要按程序由各高校主管部门报教育部审批。

f43213efbba25e9e7f3e98deb60214fd.png

22.国家专项计划招生如何录取?

国家专项计划招收具有国家专项计划招生资格的考生,在本科提前批A段和本科提前批B段录取。设1次集中填报志愿和1次征集志愿。

本科提前批A段,实行顺序志愿投档。生源不足时,通过志愿征集和适当降分录取。

本科提前批B段,实行平行志愿投档, 集中填报志愿录取控制分数线按我省招生计划中公布的分数线执行。生源不足时,通过征集志愿和适当降分录取。

 f43213efbba25e9e7f3e98deb60214fd.png

23.高校专项计划招生如何录取?

高校专项计划招收具有高校专项计划招生资格的考生,在本科提前批C段录取, 执行高校规定的相关分数要求,考生须经招生院校考核合格且经教育部“阳光高考信息平台公示,根据考生志愿、文化成绩和招生院校要求投档,由高校审查决定是否录取。

f43213efbba25e9e7f3e98deb60214fd.png

24.地方专项计划招生如何录取?

地方专项计划招收具有地方专项计划招生资格的考生,在本科批录取,执行我省本科批文化控制线,实行平行志愿投档。

f43213efbba25e9e7f3e98deb60214fd.png

25.国家专项计划、高校专项计划、地方专项计划录取考生放弃入学资格或退学有什么后果?

根据教育部规定,从2023年起,往年被国家专项计划、高校专项计划、地方专项计划录取后放弃入学资格或退学的考生,不再具有专项计划报考资格。

f43213efbba25e9e7f3e98deb60214fd.png

26.少数民族本科预科班如何录取?

少数民族本科预科班是根据少数民族学生的特点,采取特殊措施,着重提高文化基础知识,加强基本技能的训练,使学生在德育、智育、体育几个方面都得到进一步发展与提高,为在高校进行专业学习打下良好基础所开设的一种教学班制度。少数民族本科预科班从当年参加统考的少数民族考生中择优录取,在本科批进行, 实行平行志愿投档。录取时,少数民族本科预科班录取分数不低于同批次同科类各有关高校非预科班普通类专业(不含地方专项计划)最低投档分数线下 80 分;征集志愿录取时,少数民族本科预科班录取分数不低于同批次同科类相应高校第一次投档录取时的非预科班普通类专业(不含地方专项计划)最低投档线下 80 分。

f43213efbba25e9e7f3e98deb60214fd.png

27.边防军人子女预科班如何录取?

边防军人子女预科班只招收通过边防军人子女资格审查的考生,在本科批进行,实行平行志愿。其中,边防军人子女预科班录取分数不低于院校所在批次控制线下60分,重庆大学等部分高校的边防军人子女预科班录取分数要求不低于特殊类型招生录取控制分数线。

f43213efbba25e9e7f3e98deb60214fd.png


28.残疾考生报考普通高校录取有什么规定?

各高校开设的专业不同,学生将来所从事的职业不同,对学生身体状况也有不同要求。残疾考生根据自己的身体状况,对照《普通高等学校招生体检工作指导意见》和院校的招生章程,选择适合自身身体状况的专业。

对肢体残疾、生活能够自理、不影响所报专业学习,且考试成绩达到要求的考生,高校不能仅因其残疾而不予录取。

 

f43213efbba25e9e7f3e98deb60214fd.png

29.单独招收残障考生的院校有哪些?

对残障考生招生,有专门的绿色通道。近年经教育部批准,具备招收残障考生资格的院校有北京联合大学、天津理工大学、长春大学、郑州工程技术学院、长沙职业技术学院、贵州盛华职业学院、河南推拿职业学院、滨州医学院、南京特殊教育师范学院、西安美术学院、郑州师范学院、乐山师范学院、山东特殊教育职业学院、浙江特殊教育职业学院、绥化学院、重庆师范大学等高校,招收对象为符合高考报名条件的残障考生,具体招生事宜考生可向招生院校咨询或查阅院校招生简章。

f43213efbba25e9e7f3e98deb60214fd.png

30.控制分数线上考生未被录取的主要原因有哪些?

(1) 填报了无效志愿。如汉族考生填报少数民族本科预科班志愿;不具备资格考生填报国家专项计划志愿;填报志愿时未按要求操作等。

(2)未达到所填报院校专业(类)的录取分数线 。

(3)身体状况或相关科目等不符合院校专业(类)要求。如有的考生身体条件受限,盲 目 选报一些受限院校专业(类);没有参加外语口试的考生报考需要口试的专业(类)等。

(4)教育部规定,根据考生成绩,从高分到低分,按略多于招生计划总数确定控制分数线。既然按略多于招生计划总数确定控制分数线,那么这略多于的部分在录取过程中就有可能落选。

f43213efbba25e9e7f3e98deb60214fd.png

31.如何准确查询录取结果?

招生院校依法依规寄发的录取通知书,是 已录取考生获得录取结果的正式渠道。

查询渠道:(1) 通过省教育考试院网站(网址: http: //www.hebeea.edu.cn) 、省教育考试院微信公众号(hbsksy) 、省教育考试院 APP 客户端(扫描省教育考试院网站的二维码下载并安装Android系统或i0S系统的手机客户端软件)查询本人录取结果 。(2) 通过县(市、区)高考咨询服务站查询本人录取结果。(3)通过院校网站查询。

f43213efbba25e9e7f3e98deb60214fd.png

32.如何识别和防范招生诈骗行为?

高校招生“阳光工程”的实施,有效遏制了各种招生诈骗行为,家长和考生甄别诈骗行为的能力也有所提高,招生诈骗行为大量减少,但仍须提醒考生和家长在高校录取中加以防范。常见招生诈骗行为有以下几种形式:

(1)假冒招生工作人员进行诈骗。一些不法分子假冒高校招生工作人员,吹嘘 自己神通广大,诱骗考生填报志愿,之后又强调录取有难度,同时暗示自己可以帮忙,部分家长为了孩子的前途,只好出钱,从而上当受骗。

(2)故意混淆教育形式进行诈骗。一些不法分子故意混淆网络教育、成人教育、学历文凭教育、自学考试、开放大学等与普通高等教育的区别, 声 称只 要花钱就可以读大学,结果 等考生家长出钱后,得到的不是普通高等教育录取通知书,而是自学考试助学班、成人教育辅导班、远程网络教育班等的录取通知书。这是一种比较普遍的诈骗方式。

(3)声称可以帮助考生“低分高录”进行诈骗。招生骗子自称是高校领导或者是省招办领导的熟人,声称可以帮忙,可以把低层次分数的考生提到高一层次院校录取,从而骗取钱财。

(4)编造所谓“内部指标” 的谎言进行诈骗。招生骗子伪造文件以及高校公章, 声称手中拥有高校招生“内部指标”“机动指标考生想上大学,需要花钱通过“内部指标”录取。

(5)利用“定向招生” 之名实施诈骗。一些招生骗子利用国家定向招生政策欺骗考生家长。

(6)非正当手段搜集录取信息,从而骗取钱财。每年高校录取期间,有些骗子通过非法手段获取考生信息,将考生原本正常投档以及录取,说成是花钱打通关系, 以此向考生家长骗取钱财。

为此,考生和家长要在以下几个方面提高警惕:

(1)普通高校招生录取新生是按照国家政策、院校招生章程、考生的分数和志愿, 在全国统一编制的网上录取系统中进行的,并且经过省教育考试院审核报教育部备案。未经省教育考试 院办理录取手续的考生,教育部将一律不予学籍学历电子注册。

(2)省招生考试机构未授权任何单位或个人从事招生中介服务。

(3)高校招生不会向考生收取与录取挂钩的任何费用。

(4)不存在公布的招生计划以外的所谓的“内部指标”。

(5)考生可通过各级招生考试机构查询自己的录取结果。特别提醒考生:

有的考生委托中学、高校或他人代为填报志愿,将自己的考生号、登录密码等信息透露给委托人,录取时发现与本人设想相去甚远,想修改志愿但密码已被他人篡改,追悔莫及。为维护考生本人利益,请务必妥善保管个人信息,更不要委托他人填报, 否则后果考生自负

f43213efbba25e9e7f3e98deb60214fd.png

33.新生入学报到前应做哪些准备?

接到大学录取通知书,就意味着你将是一名大学新生了,必须从各方面做好成为一名大学生的准备。

(1)各种证件和材料的准备。为了能够按时报到,必须按照随录取通知书一起寄来的“新生须知”上的要求准备好报到所需 的各种证件和材料,如准考证、身份证、户口迁移证明、高考纸介质档案(河北省2023年普通高校录取新生综合信息登记表)、高级中等教育阶段档案(或人事档案)和党、团关系等证件和材料,和录取通知书放在一起准备报到之用。

(2)经济上的准备。准备好报到时需交纳的学费、住宿费等院校要求的各种费用,同时要准备好到校后生活所需的伙食费及零用钱。

(3)学习和生活必需品的准备。尤其是到外地上学的学生,需要准备好四季的衣服、一些生活用品及学习用品。

(4)旅途准备。大多数新生都要到外地上学,入学报到需长途旅行。因此行前要周密安排,买好火车票,凭录取通知书新生可以享受半价,要注意报到时间,及时从家里出发,以免影响报到注册。

(5)思想准备。新生更重要的是做好思想上的准备。许多新生从小到大从未离开过父母,不习惯独立生活,而大学恰好是要培养学生独立生活的能力,所以新生从思想上要做好离开父母自己生活的准备,准备独立面对生活,解决可能出现的问题。同 时,应该做好继续努力学习的准备,认识到迈入大学是担起一副更加沉重的担子。大学的学习内容、学习方法与中学相比都有明 显的不同,要有勇气处理即将遇到的一切困难和问题,对大学中可能遇到的种种难题都要有战胜和克服它们的勇气和信心。

(6)考生凭录取通知书办理户口迁移手续,并按高校规定的时间及有关要求,办理报到等手续。不能按时报到的,应向高校提出书面申请,经校方同意后,方可延期报到。新生在报到期间无故不报到的,将被取消入学资格。

f43213efbba25e9e7f3e98deb60214fd.png

34.考生如何申请国家助学贷款?

家庭经济困难学生可申请办理国家助学贷款,解决学费、住宿费及部分生活费。本专科生每生每学年最高申请金额不超过12000元。贷款学生在校学习期间的助学贷款利息全部由财政补贴,毕业后的利息由贷款学生本人全额支付,还款期限原则上按学制加15 年确定,最长不超过22年。国家助学贷款是信用贷款,学生不需要办理贷款担保或抵押,但需要承诺按期还款,并承担相关法律责任。国家助学贷款包括生源地信用助学贷款和校园地国家助学贷款,我省具体办理方式为:

(1)生源地信用助学贷款(国家开发银行河北省分行、农村信用社承办)。

贷款对象:河北籍家庭经济困难的全日制普通高校本专科生(含高职、第二学士学位、预科)和研究生。

贷款方式:由学生户籍所在地县级学生资助机构或农村信用社县(市、区)联社统一办理。

申请方式:学生在新学期开始前,登录河北省生源地信用助学贷款管理系统网站(网址:http://syd.hee.gov.cn) 或国家开发银行生源地助学贷款学生在线系统网站(网址:http://www.csls.cdb.com.cn), 填写统一的贷款申请表,学生和家长向户籍所在地县级教育部门申请。

(2)校园地国家助学贷款(国家开发银行河北省分行承办,目前仅在省属公办 28 所高校开办)。

贷款对象:家庭经济困难的全日制普通高校本专科生(含高职、第二学士学位、预科)和研究生。

申请方式:每年9月份,学生向所在高校申请。

除以上介绍的国家助学贷款外,国家有关部门、高校、相关部门或团体、组织等,还有其他形式的助学、奖学方式,请考生注意向高校或有关部门咨询。

f43213efbba25e9e7f3e98deb60214fd.png

35.高校如何进行新生入学复查?

按教育部规定,各普通高校在新生入学报到时须进行新生入学复查。依据录取相关材料、准考证、考生档案相关内容,进行身份复核、体检复查等。对特殊类型招生录取的新生,高校将组织专家组开展入学专业复测和复核,对于专业测试不达标、入学前后两次测试成绩差异显著的情况,认真核实确认。对伪造材料取得报考资格者、冒名顶替者和体检舞弊及其他舞弊者,按照有关规定处理。

f43213efbba25e9e7f3e98deb60214fd.png

36.新生入学时如何购买合格的学生公寓床上用品?

河北省高校的学生公寓管理部门将为新生提供学生公寓床上用品代购服务, 进货价格销售。代购产品从河北省教育厅、河北省市场监督管理局共同组织的展销订货会上预定,并已取得省级质检部门检验合格证明。新生购买时,要选择公寓管理部门设置的张贴河北省产品质量监督检验研究院产品质量合格报告的场所,切记不要购买没有检验合格证明的产品。按照教育部和国家市场监督管理总局有关文件要求,为防止学生将劣质床上用品带入校园,公寓管理部门将对学生自购自带床上用品的品牌、生产企业等进行登记造册,如发现涉嫌质量问题产品,市场监管部门将提示学生进行更换,并对销售者进行查处。

在购买、使用过程中遇到问题,可与以下部门进行联系:

河北省学校后勤服务中心电话:0311-66005765, 66005766河北省产品质量监督检验研究院电话:0311-67568286。录取到省外高校的考生,相关问题请向录取高校咨询。

f43213efbba25e9e7f3e98deb60214fd.png

           河北省2023年普通高校

        招生工作新视点

1.强化报名资格审查。对接教育部高校学信息和全省高学籍信息, 实时比对考生学籍信息;联合公安部门对考生户籍和居住证信息逐一核查,严防“ 高考移民”;与卫健、人社、退役军人等部门建立联审机制,强化优惠加分、优先录取等优惠资格审核,严防弄虚作假。

2.严厉打击违规行为。将防范手机等高科技作弊及群体性、有组织舞弊作为重点。考前联合公安、保密、无线电管理、通信管理等部门持续开展考试环境综合治理专项活动。考生入场全部采用人脸识别技术, 严防替考。强化安检措施, 严格执行国家教育考试入场安检规范, 严防手机等作弊器材入场。所有考场、 通道、卫生间等区域实现5G信号屏蔽全覆盖。考试期间强化监考巡考,实现全程监控、全程录像。考后严格执行考场视频回放制度,严查违规违纪行为。

3.严格三个专项计划报考资格。《教育部关于做好2023年普通高校招生工作的通知》(教学[2023]J1号)规定,从2023年起,往年被国家专项计划、高校专项计划、地方专项计划录取后放弃入学资格或退学的考生,不再具有专项计划报考资格。

4.加强涉考培训咨询机构治理。根据教育部部署,各地教育行政部门会同网信、科技、公安、文化和旅游、市场监管、体育等部门,加强对社会培训机构或个人开展涉考培训咨询的规范治理,严厉打击涉及虚假宣传、价格欺诈、组织或参与考试作弊、干扰破坏考试招生秩序等违规违法行为。

5.深化艺术类专业考试招生改革。按照教育部《关于进一步加强和改进普通高等学校艺术类专业考试招生工作的指导意见》精神,我省到2024 年,基本建立以统一高考为基础 、省级专业考试为主体,依据高考文化成绩、专业考试成绩,参考学生综合素质评价,分类考试、综合评价、多元录取的高校艺术类专业考试招生制度,形成促进公平、科学选才、监督有力的艺术人才选拔评价体系。

6.改革高校高水平艺术团招生办法。自2024 年起,高校高水平艺术团不再从高校招生环节选拔,由相关高校从在校生中遴选培养。

7.深化高校高水平运动队考试招生改革。自2024年起,进一步完善和规范高校高水平运动队考试招生工作,通过优化招生项目范围, 严格报考条件和资格审核,改进考试评价方式,提高文化成绩要求,完善招生录取机制,选拔培养德智体美劳全面发展且具有较高体育竞技水平的学生,为奥运会、世界大学生运动会等重大体育比赛和国家竞技体育后备人才培养体系提供人才支撑。


重要日程备忘




































f43213efbba25e9e7f3e98deb60214fd.png


1.如何认定与处理考生的违规行为?

(一)按照《国家教育考试违规处理办法》(教育部令第 33号)规定,按以下情形认定考生的违规行为并进行处理:

(1)考生不遵守考场纪律,不服从考试工作人员的安排与要求,有下列行为之一的,应当认定为考试违纪:

携带规定以外的物品进入考场或者未放在指定位置的;

未在规定的座位参加考试的;

考试开始信号发出前答题或者考试结束信号发出后继续答  题的;

在考试过程中旁窥、交头接耳、互打暗号或者手势的;

在考场或者教育考试机构禁止的范围内,喧哗、吸烟或者实施其他影响考场秩序的行为的;

未经考试工作人员同意在考试过程中擅自离开考场的;

将试卷、答卷(含答题卡、答题纸等,下   同)、草稿纸等考试用纸带出考场的;

用规定以外的笔或者纸答题或者在试卷规定以外的地方书写姓名、考号或者以其他方式在答卷上标记信息的;

其他违反考场规则但尚未构成作弊的行为。

考生有上述第(1)条所列考试违纪行为之一的,取消该科目的考试成绩。

(2)考生违背考试公平、公正原则,在考试过程中有下列行为之一的,应当认定为考试作弊:

携带与考试内容相关的材料或者存储有与考试内容相关资料的电子设备参加考试的;

抄袭或者协助他人抄袭试题答案或者与考试内容相关的资  料的;

抢夺、窃取他人试卷、答卷或胁迫他人为自己抄袭提供方便的;

携带具有发送或者接收信息功能的设备的;

由他人冒名代替参加考试的;

故意销毁试卷、答卷或者考试材料的;

在答卷上填写与本人身份不符的姓名、考号等信息的;

传、接物品或者交换试卷、答卷、草稿纸的;

其他以不正当手段获得或者试图获得试题答案、考试成绩的行为。

(3)教育考试机构、考试工作人员在考试过程中或者在考试结束后发现下列行为之一的,应当认定相关的考生实施了考试作弊行为:

通过伪造证件、证明、档案及其他材料获得考试资格、加分资格和考试成绩的;

评卷过程中被认定为答案雷同的;

考场纪律混乱、考试秩序失控,出现大面积考试作弊现象的;

考试工作人员协助实施作弊行为,事后查实的;

其他应认定为作弊的行为。

考生有上述第 (2) 、(3)条所列考试作弊行为之一的,其所报名参加考试的各阶段、各科成绩无效。

(4)考生及其他人员应当自觉维护考试秩序,服从考试工作人员的管理,不得有下列扰乱考试秩序的行为:

故意扰乱考点、考场、评卷场所等考试工作场所秩序;

拒绝、妨碍考试工作人员履行管理职责;

威胁、侮辱、诽谤、诬陷或者以其他方式侵害考试工作人员、其他考生合法权益的行为;

故意损坏考场设施设备;

其他扰乱考试管理秩序的行为。

考生有上述第(4)条所列行为之一的,应当终止其继续参加本科目考试,其当次报名参加考试的各科成绩无效;考生及其他人员的行为违反《中华人民共和国治安管理处罚法》的,由公安机关进行处理;构成犯罪的,由司法机关依法追究刑事责任。

(5)有下列情形之一的,可以视情节轻重,同时给予暂停参加该项考试 1 至 3 年的处理;情节特别严重的,可以同时给予暂停参加各种国家教育考试 1 至 3 年的处理:

组织团伙作弊的;

向考场外发送、传递试题信息的;

使用相关设备接收信息实施作弊的;

伪造、变造身份证、准考证及其他证明材料, 由他人代替或者代替考生参加考试的。

(6)考生以作弊行为获得的考试成绩并由此取得相应的学位

证书、学历证书及其他学业证书、资格资质证书或者入学资格的,由证书颁发机关宣布证书无效,责令收回证书或者予以没收;已经被录取或者入学的,由录取学校取消录取资格或者其学籍。

(7) 在校学生、在职教师有下列情形之一的,教育考试机构应当通报其所在学校,由学校根据有关规定严肃处理,直至开除学籍或者予以解聘:

代替考生或者由他人代替参加考试的;

组织团伙作弊的;

为作弊组织者提供试题信息、答案及相应设备等参与团伙作弊行为的。

(二)按照《普通高等学校招生违规行为处理暂行办法》(教育部令第 36 号)规定,考生有下列情形之一的,应当如实记入其考试诚信档案。下列行为在报名阶段发现的,取消报考资格;在入学前发现的,取消入学资格;入学后发现的,取消录取资格或者学籍;毕业后发现的,由教育行政部门宣布学历、学位证书无效,责令收回或者予以没收;涉嫌犯罪的,依法移送司法机关处理。

(1 ) 提供虚假姓名、年龄、民族、户籍等个人信息,伪造、非法获得证件、成绩证明、荣誉证书等,骗取报名资格、享受优惠政策的;

(2)在综合素质评价、相关申请材料中提供虚假材料、影响录取结果的;

(3)冒名顶替入学,由他人替考入学或者取得优惠资格的;

(4)其他严重违反高校招生规定的弄虚作假行为。

违反国家教育考试规定、情节严重受到停考处罚,在处罚结束后继续报名参加国家教育考试的,由学校决定是否予以录取。

教育部规定:教育考试机构应当建立国家教育考试考生诚信档案,记录、保留在国家教育考试中作弊人员的相关信息。国家教育考试考生诚信档案可以依申请接受社会有关方面的查询,并应当及时向招生学校或单位提供相关信息,作为招生参考条件。

f43213efbba25e9e7f3e98deb60214fd.png

2.考试工作人员违纪舞弊应受何种处理?

(一)按照《国家教育考试违规处理办法》(教育部令第33号)规定,按以下情形认定考试工作人员的违规行为并进行处理:

(1)考试工作人员应当认真履行工作职责,在考试管理、组织及评卷等工作过程中,有下列行为之一的,应当停止其参加当年及下一年度的国家教育考试工作,并由教育考试机构或者建议其所在单位视情节轻重分别给予相应的行政处分:

应回避考试工作却隐瞒不报的;

擅自变更考试时间、地点或者考试安排的;

提示或暗示考生答题的;

擅自将试题、答卷或者有关内容带出考场或者传递给他人的;

未认真履行职责,造成所负责考场出现秩序混乱、作弊严重或者视频录像资料损毁、视频考试系统不能正常工作的;

在评卷、统分中严重失职,造成明显的错评、漏评或者积分差错的;

在评卷中擅自更改评分细则或者不按评分细则进行评  卷的;

因未认真履行职责,造成所负责考场出现雷同卷的;

擅自泄露评卷、统分等应予保密的情况的; 的其他违反监考、评卷等管理规定的行为。

(2)考试工作人员有下列作弊行为之一的,应当停止其参加国家教育考试工作,由教育考试机构或者其所在单位视情节轻重分别给予相应的行政处分,并调离考试工作岗位;情节严重,构成犯罪的,由司法机关依法追究刑事责任:

少为不具备参加国家教育考试条件的人员提供假证明、证件、档案,使其取得考试资格或者考试工作人员资格的;

因玩忽职守,致使考生未能如期参加考试的或者使考试工作遭受重大损失的;

利用监考或者从事考试工作之便,为考生作弊提供条  件的;

伪造、变造考生档案(含电子档案)的;

在场外组织答卷、为考生提供答案的;

指使、纵容或者伙同他人作弊的;

偷换、涂改考生答卷、考试成绩或者考场原始记录材  料的;

擅自更改或者编造、虚报考试数据、信息的;

利用考试工作便利,索贿、受贿、以权徇私的; 吵诬陷、打击报复考生的。

(3)对出现大规模作弊情况的考场、考点的相关责任人、负责人及所属考区的负责人,有关部门应当分别给予相应的行政处分;情节严重,构成犯罪的,由司法机关依法追究刑事责任。

(4)违反保密规定,造成国家教育考试的试题、答案及评分参考(包括副题及其答案及评分参考,下同)丢失、损毁、泄密,或者使考生答卷在保密期限内发生重大事故的,由有关部门视情节轻重,分别给予责任人和有关负责人行政处分;构成犯罪的,由司法机关依法追究刑事责任。

盗窃、损毁、传播在保密期限内的国家教育考试试题、答案及评分参考、考生答卷、考试成绩的,由有关部门依法追究有关  人员的责任;构成犯罪的,由司法机关依法追究刑事责任。

(5)有下列行为之一的,由教育考试机构建议行为人所在单位给予行政处分;   违反《中华人民共和国治安管理处罚法》的,由公安机关依法处理;构成犯罪的, 由 司法机关依法追究刑事责任:

指使、纵容、授意考试工作人员放松考试纪律,致使考场秩序混乱、作弊严重的;

代替考生或者由他人代替参加国家教育考试的;

组织或者参与团伙作弊的;

利用职权,包庇、掩盖作弊行为或者胁迫他人作弊的;

以打击、报复、诬陷、威胁等手段侵犯考试工作人员、考生人身权利的;

向考试工作人员行贿的;

故意损坏考试设施的;

扰乱、妨害考场、评卷点及有关考试工作场所秩序后果严重的。

国家工作人员有第(5)条所列行为的,教育考试机构应当建议有关纪检、监察部门,根据有关规定从重处理。

(二)按照《普通高等学校招生违规行为处理暂行办法》(教育部令第 36 号)规定,招生工作人员有下列情形之一的,其所在单位应当立即责令暂停其负责的招生工作,由有关部门视情节轻重依法给予相应处分或者其他处理;涉嫌犯罪的,依法移送司法机关处理。

(1)违规更改考生报名、志愿、资格、分数、录取等信息的;

(2)对已录取考生违规 变更录取学校或者专业的;

(3)在特殊类型招生中泄露面试考核考官名单或者利用职务便利请托考核评价的教师,照顾特定考生的;

(4)泄露尚未公布的考生成绩、考生志愿、录取分数线等可能影响录取公正信息的,或者对外泄露、倒卖考生个人信息的;

(5)为考生获得相关招生资格弄虚作假、徇私舞弊的;

(6)违反回避制度, 应当回避而没有回避的;

(7)索取或收受考生及家长财物,接受宴请等可能影响公正履职活动安排的;

(8)参与社会中介机构或者个人非法招生活动的;

(9)其他影响高校招生公平、公正的行为。

f43213efbba25e9e7f3e98deb60214fd.png


3.对教育考试机构作出的违规处理决定不服的如何申诉?

考生或者考试工作人员对教育考试机构作出的违规处理决定不服的,可以在收到处理决定之日起15日内,向其上一级教育考试机构提出复核申请;对省级教育考试机构或者承办国家教育考试的机构作出的处理决定不服的,也可以向省级教育行政部门或者授权承担国家教育考试的主管部门提出复核申请。

受理复核申请的教育考试机构、教育行政部门应对处理决定所认定的违规事实和适用的依据等进行审查,并在受理后30日内,按照下列规定作出复核决定:

(1)处理决定认定事实清楚、证据确凿,适用依据正确, 程序合法,内容适当的,决定维持。

(2)处理决定有下列情况之一的,决定撤销或者变更:

违规事实认定不清、证据不足的;

适用依据错误的;

违反本办法规定的处理程序的。

作出决定的教育考试机构对因错误的处理决定给考生造成的损失, 应当予以补救。

申请人对复核决定或者处理决定不服的,可以依法申请行政复议或者提起行政诉讼。

f43213efbba25e9e7f3e98deb60214fd.png

4.发现考生违规行为如何举报?

在报名、体检、考试、录取期间,对违规行为可通过招生考试机构设置的举报电话、邮箱、举报箱等进行举报,也可以直接到当地招生考试机构或教育行政部门举报。

f43213efbba25e9e7f3e98deb60214fd.png

5.违规招生的学校应受何种处理?

按照《普通高等学校招生违规行为处理暂行办法》(教育部令第 36 号)规定, 高校违反国家招生管理规定,有下列情形之一的,由主管教育行政部门责令限期改正,给予警告或者通报批评;情节严重的,给予减少招生计划、暂停特殊类型招生试点项目或者依法给予停止招生的处理。对直接负责的主管人员和其他 直接责任人员,视情节轻重依法给予相应处分;涉嫌犯罪的,依法移送司法机关处理。

(1)发布违反国家规定的招生简章,或者进行虚假宣传、骗取钱财的;

(2)未按照信息公开的规定公开招生信息的;

(3)超出核定办学规模招生或者擅自调整招生计划的;

(4)违反规定降低标准录取考生或者拒绝录取符合 条件的考生的;

(5)在特殊类型招生中出台违反国家规定的报考条件,或者弄虚作假、徇私舞弊,录取不符合条件的考生的;

(6)违规委托中介机构进行招生录取, 或者以承诺录取为名向考生收取费用的;

(7)其他违反国家招生管理规定的行为。

f43213efbba25e9e7f3e98deb60214fd.png

6.在艺术、体育、对口升学和高职单招等各类单招中违规的考生如何处理?

艺术、体育、对口升学和高职单招等各类单招考试是高考的组成部分,依据教育部《国家教育考试违规处理办法》(教育部令第33号),考生在上述考试中被认定为违纪的,取消其该科目的考试成绩;被认定为作弊的,其当年高考报名参加考试的各阶段、各科成绩无效。

f43213efbba25e9e7f3e98deb60214fd.png

7.《中华人民共和国刑法》涉及考试作弊入刑的主要内容是什么?

(1)在刑法第二百八十条之一后增加一条, 第二百八十条之二:

盗用、冒用他人身份,顶替他人取得的高等学历教育入学资格、公务员录用资格、就业安置待遇的,处三年以下有期徒刑、拘役或者管制,并处罚金

组织、指使他人实施前款行为的,依照前款的规定从重处罚。

国家工作人员有前两款行为,又构成其他犯罪的,依照数罪并罚 的规定处罚。

(2)在刑法第二百八十四条后增加一条,作为第二百八十四条之一:

在法律规定的国家考试中,组织作弊的,处三年以下有期徒刑或者拘役,并处或者单处罚金;情节严重的,处三年以上七年以下有期徒刑,并处罚金。

为他人实施前款犯罪提供作弊器材或者其他帮助的,依照前款的规定处罚。

为实施考试作弊行为, 向他人非法出售或者提供第一款规定的考试的试题、答案的,依照第一款的规定处罚。

代替他人或者让他人代替自己参加第一款规定的考试的,处拘役或者管制,并 处或者单处罚金。

f43213efbba25e9e7f3e98deb60214fd.png

8.《最高人民法院最高人民检察院关于办理组织考试作弊等刑事案件适用法律若干问题的解释》涉及高考作弊入刑的主要内容是什么?

普通高等学校招生考试及其特殊类型招生、特殊技能测试、面试等考试,均属“法律规定的国家考试”。

在法律规定的国家考试中,组织作弊,具有下列情形之一的,应当认定为刑法第二百八十四条之一第一款规定的“情节严重”:

在普通高等学校招生考试中组织考试作弊的;导致考试推迟、取消或者启用备用试题的;考试工作人员组织考试作弊的;组织考生跨省、自治区、直辖市作弊的;多次组织考试作弊的; 组织三十人次以上作弊的;提供作弊器材五十件以上的;违 法所得三十万元以上的;其他情节严重的 情形

具有避开或者突破考场防范作弊的安全管理措施,获取、记录、传递、接收、存储考试试题、答案等功能的程序、工具,以及专门设计用千作弊的程序、工具,应当认定为刑法第二百八十四条之一第二款规定的作弊器材”。

是否属于刑法第二百八十四条之一第二款规定的作弊器材难以确定的,依据省级以上公安机关或者考试主管部门出具的报告,结合其他证据作出认定;涉及专用间谍器材、窃听、窃照专用器材、伪基站” 等器材的,依照相关规定作出认定。

组织考试作弊,在考试开始之前被查获,但已经非法获取考试试题、答案或者具有其他严重扰乱考试秩序情形的,应当认定为组织考试作弊罪既遂。

为实施考试作弊行为,非法出售或者提供法律规定的国家考试的试题、答案, 具有下列情形之一的,应当认定为刑法第二百八十四条之一第三款规定的“情节严重”:

非法出售或者提供普通高等学校招生考试的试题、答案的;导致考试推迟、取消或者启用备用试题的;考试工作人员非法出售或者提供试题、答案的;多次非法出售或者提供试题、答案的;向三十人次以上非法出售或者提供试题、答案的;违法所得三十万元以上的;其他情节严重的清形。

为实施考试作弊行为,向他人非法出售或者提供法律规定的国家考试的试题、答案,试题不完整或者答案与标准答案不完全一致的,不影响非法出售、提供试题、答案罪的认定。

代替他人或者让他人代替自己参加法律规定的国家考试的, 应当依照刑法第二百八十四条之一第四款的规定,以代替考试罪定罪处罚。

对于行为人犯罪情节较轻,确有悔罪表现,综合考虑行为人替考情况以及考试类型等因素,认为符合缓刑适用条件的,可以宣告缓刑。

单位实施组织考试作弊、非法出售、提供试题、答案等行为的,依照本解释规定的相应定罪量刑标准,追究组织者、策划者、实施者的刑事责任。以窃取、刺探、收买方法非法获取法律规定的国家考试的试题、答案,又组织考试作弊或者非法出售、提供试题、答案,分别符合刑法第二百八十二条和刑法第二百八十四条之一规定的,以非法获取国家秘密罪和组织考试作弊罪或者非法出售、提供试题、答案罪数罪并罚。

设立用实施考试作弊的网站、通讯群组或者发布有关考试作弊的信息,情节严重的,应当依照刑法第二百八十七条之一的规定,以非法利用信息网络罪定罪处罚;同时构成组织考试作弊罪,非法出售、提供试题、答案罪,非法获取国家秘密罪等其他犯罪的,依照处罚较重的规定定罪处罚。

f43213efbba25e9e7f3e98deb60214fd.png

9.《中国共产党纪律处分条例》涉及考试招生违规处理的主要内容是什么?

第一百二十一条  在考试、录取工作中,有泄露试题、考场舞弊、涂改考卷、违规录取等违反有关规定行为的,给予警告或者严重警告处分;情节较重的,给予撤销党内职务或者留党察看处分;情节严重的,给予开除党籍处分。

f43213efbba25e9e7f3e98deb60214fd.png

10.《中华人民共和国教育法》涉及考试招生违规处理的主要内容是什么?

第七十九条  考生在国家教育考试中有下列行为之一的,由组织考试的教育考试机构工作人员在考试现场采取必要措施予以制止并终止其继续参加考试;组织考试的教育考试机构可以取消其相关考试资格或者考试成绩;情节严重的,由教育行政部门责令停止参加相关国家教育考试一年以上三年以下;构成违反治安管理行为的,由公安机关依法给予治安管理处罚;构成犯罪的,依法追究刑事责任:

(一)非法获取考试试题或者答案的;

(二)携带或者使用考试作弊器材、资料的;

(三)抄袭他人答案的;

(四)让他人代替自己参加考试的;

(五)其他以不正当手段获得考试成绩的作弊行为。

第八十条    任何组织或者个人在国家教育考试中有下列行为之一, 有违法所得的, 由公安机关没收违法所得,并处违法所得一倍以上五倍以下罚款;情节严重的,处五日以上十五日以下拘留;构成犯罪的,依法追究刑事责任;属于国家机关工作人员的,还应当依法给予处分:

(一)组织作弊的;

(二)通过提供考试作弊器材等方式为作弊提供帮助或者便利的;

(三)代替他人参加考试的;

(四)在考试结束前泄露、传播考试试题或者答案的;

(五)其他扰乱考试秩序的行为。

第八十一条举办国家教育考试,教育行政部门、教育考试机构疏于管理,造成考场秩序混乱、作弊情况严重的,对直接负责的主管人员和其他直接责任人员,依法给予处分;构成犯罪的,依法追究刑事责任。

f43213efbba25e9e7f3e98deb60214fd.png